Você está na página 1de 194

?

undamentos de
[ \ ;f a /
L \\ / I (fí i f ^ ^^ o w
A ' Elementar
n -F4
,O C L
« r

Samuel Hazzan
SAMUEL HAZZAN

FUNDAMENTOS DE

MATEMÁTICA
ELEMENTAR 5
COMBINATÓRIA PROBABILIDADE

ATUAL
EDÍTORA
SAMUEL HAZZAN

FUNDAMENTOS DE

MATEMÁTICA
ELEMENTAR 5
COMBINATÓRIA PROBABILIDADE

43 exercícios resolvidos
4 39 exercícios propostos com resposta
155 testes de vestibulares com resposta

7? edição
10? reimpressão

ATUAL
EDITORA
€ ; Samuel Hazzan
Copyright desta edição:
SARAIVA S.A. Livreiros Editores, São Paulo, 2011.
Rua Henrique Schaumann, 270 — Pinheiros
05413-010 São Paulo — SP
PABX: (11 >3613-3000
SAC: 0800-0117875
www.editorasaraiva.com.br
Todos os direitos reservados.

Dados Internacionais de Catalogação na Publicação (CIP)


(Câmara Brasileira do Livro, SP, Brasil)

Hazzan. Samuel. 1946-


Pundamentos de matemática elementar. 5 :
combinatória, probabilidade : 43 exercícios resolvi­
dos, 439 ex e rc ício s propostos com resposta,
155 testes de vestibular com resposta / Samuel
Hazzan. — 7. ed. São Paulo : Atual. 2004.

ISBN 978-85-357-0461-7

I. Matemática (Ensino médio) 2. Matemática


(Ensino médio) Problem as, exercícios, etc.
3. Matemática (V estibular) Testes 1. Titulo.
II. Título : Combinatória. probabilidade.

CDD-510.7

índices para catálogo sistemático:


I. Matemática : Ensino médio 510.7

Fundamentos de Matemática Elementar — vol. 5

Gerente editorial: Lauri Ccricato


Editoras: Bárbara Ferreira Arena
Teresa Christina W. P. de M. Dias
Editor de cam po: Valdir Montanari
Coordenadora editorial: Sandra Lucia Abrano
Assistentes editoriais: Mônica Rodrigues de Lima
Teresa Cristina Duarte
Revisão de texto: Pedro Cunha Jr. (Coord.)
C hefe de prepararão d e texto e revisão: Noé G. Ribeiro
C oordenadora de revisão; Maria Lucia Xavier Souto
Revisores: Alice Kobavashi
Magna Reitnberg Teobaldo
Vera Lúcia Pereira Delia Rosa
Gerente de arte: Nair de Medeiros Barbosa
Chefe d e arte: Zildo Braz
Supervisor d e arte: José Maria de Oliveira
Coordenadora de arte: Thais de ü. F. Motta
Editor de arte: Z.ildo Braz
Assistentes de arte: Lu Bevilacqua Ghion
Ricardo Yorio
Rosi Meire Martins Ortega
Gerente d e produ ção: Antonio Cabello Q. Filho
Assistente d e produ ção: Graee Alves
D iagram ação: Setup Bureau Editoração Eletrônica
C oorden ação eletrônica: Silvia Regina E. Almeida
Produção gráfica: José Rogério L. de Simone
Maurício T. de Moraes
Colaboradores
Revisão técnica: Irene Torrano Filisetti
Projeto gráfico (miolo); Thais de B. F. Motta
(capa): Ettore Bottini
Imagem de ca p a : Milton Ribeiro
Foto li to: Pri scon/B inhos
C om posição e arte-final: Diarte Ed. e Coml. de Livros Ltda.

Visite nosso site: www.atualeditora.com.br


Central de atendimento ao professor: (Oxxl I ) 3613-3030
Apresentação

Fundamentos de M atemática Elementar é uma coleção elaborada com o objeti­


vo de oferecer ao estudante uma visão global da Matemática, no ensino médio. De­
senvolvendo os programas em geral adotados nas escolas, a coleção dirige-se aos
vestibulandos, aos universitários que necessitam rever a Matemática elementar e tam­
bém, como é óbvio, àqueles alunos de ensino médio cujo interesse focaliza-se em ad­
quirir uma formação mais consistente na área de Matemática.
No desenvolvimento dos capítulos dos livros de Fundamentos procuramos seguir
uma ordem lógica na apresentação de conceitos e propriedades. Salvo algumas exce­
ções bem conhecidas da Matemática elementar, as proposições e os teoremas estão
sempre acompanhados das respectivas demonstrações.
Na estruturação das séries de exercícios, buscamos sempre uma ordenação crescen­
te de dificuldade. Partimos de problemas simples e tentamos chegar a questões que en­
volvem outros assuntos já vistos, levando o estudante a uma revisão. A seqüência do tex­
to sugere uma dosagem para teoria e exercícios. Os exercícios resolvidos, apresentados
em meio aos propostos, pretendem sempre dar explicação sobre alguma novidade que
aparece. No final de cada volume, o aluno pode encontrar as respostas para os problemas
propostos e assim ter seu reforço positivò ou partir à procura do erro cometido.
A última parte de cada volume íé Constituída por testes de vestibulares,
selecionados dos melhores vestibulares do país e com respostas. Esses testes podem ser
usados para uma revisão da matéria estudada.
Aproveitamos a oportunidade para agradecer ao professor dr. Hygino H.
Domingues, autor dos textos de história d^ Matemática que contribuem muito para o
enriquecimento da obra.
Neste volume, abordamos o estudo da análise combinatória e do cálculo de pro­
babilidades. Em análise combinatória, a ênfase maior é dada ao princípio fundamental
da contagem que, sem dúvida, é de grande utilidade. Convém lembrar que fica impossí­
vel desenvolver o cálculo de probabilidades sem antes ter tratado exaustivamente de
análise combinatória. O estudo do teorema de Newton para desenvolver potências ^e
binômios tem importância menor, não devendo ocupar tempo exagerado no curso. \
Finalmente, como há sempre uma certa distância entre o anseio dos autores e o
valor de sua obra, gostaríamos de receber dos colegas professores uma apreciação so­
bre este trabalho, notadamente os comentários críticos, os quais agradecemos.

Os autores
Sumário

CAPÍTULO I — AN ÁLISE COMBINATÓRIA ,


I. Introdução ........................................................................................ 1
II. Princípio fundamental da contagem ....................................... 2
III. Conseqüências do princípio fundamental da contagem .... 15
IV. Arranjos com repetição................................................................ 15
V. Arranjos ............................................................................................ 16
VI. Permutações .................................................................................... 18
VII. Fatorial ............................................................................................. 19
V III. Combinações ................................... 33
IX . Permutações com elementos repetidos......... .......................... 44
X . Complementos .............. 49
Leitura: Cardano: o intelectual jo g a d o r............................................. 56
CAPÍTULO II — BINÔMIO DE NEWTON 5g
I. Introdução .......................... >.......................................................... 58
II. Teorema binom ial.......................................................................... 61
III. O bservações....................................................................................... 63
IV. Triângulo aritmético de Pascal (ou de T artaglia)................. 70
V. Expansão m ultinom ial................ . ................................................ 80
Leitura: Pascal e a teoria das probabilidades................................... 83
Leitura: Os irmãos Jacques e Jean B ernou lli................................... 85
CAPÍTULO III — PRO BA BILID A D E g9
I. Experimentos aleató rio s.............................................................. 89
II. Espaço am o strai............................................................................. 90
III. E v en to ................................................................................................. 92
IV. Combinações de eventos.............................................................. 93
V. Freqüência relativ a......................................................................... 96
VI. Definição de probabilidade.......................................................... 97
VIL Teoremas sobre probabilidades em espaço amostrai Finito 100
VIII. Espaços amostrais equiprováveis................................................ 106
IX. Probabilidade de um evento num espaço equiprovável......... 107
X. Probabilidade condicional............................................................. 118
XI. Teorema da multiplicação............................................................... 123
XII. Teorema da probabilidade to tal.— ................................................ 126
X III. Independência de dois eventos....................................................... 133
XIV. Independência de três ou mais eventos....................................... 135
XV. Lei binominal da probabilidade.................................................... 139
Leitura: Laplace: a probabilidade chega aos c é u s .............................. 146

R ESPO STA S DOS E X E R C ÍC IO S ......................................................... 148


T E S T E S DE V E S T IB U L A R E S .............................................................. 158
R ESPO STA S DOS T E S T E S ..................................................................... 182
CAPITULO I

Análise
Combinatória

I. Introdução

1. A A nálise C om b in ató ria visa desenvolver m étodos que perm itam con tar
o núm ero de elem entos de um co n ju n to , sendo estes elem entos ag ru p am en tos
fo r m a d o s s o b certas co n d iç õ e s .
À prim eira vista pode parecer desnecessária a existência desses m étodos.
Isto de fa to é verdade, se o núm ero de elem entos que querem os co n ta r fo r pe­
qu eno. E n tretan to , se o núm ero de elem entos a serem co n tad o s fo r grande, es­
se trab alh o to rn a-se quase im possível sem o uso de m étodos especiais.
V ejam o s alguns exem plos. U sarem os a n o tação # A f p ara indicar o nú­
m ero de elem entos de um co n ju n to M .

2. Exemplos

1?) A é o co n ju n to de núm eros de dois algarism os distintos form ados a


partir dos dígitos 7, 2 e 5 .
A = [12, 13, 2 1 , 2 3 , 31, 32) e # A = 6

2 ?) B é o co n ju n to das d iagonais de um heptágono


b = [p ^ , p ^ , v ji, p ^ , iy v p j v p ã ,
p 2p 7> p 3p 5) p 3p 6) p 3p 7, p 4p 6, p 4p 7, p 5p 7j
e # B = 14. P6 P5

l
ANÁLISE COMBINATÓRIA

3 ?) C é o co n ju n to das seqüências de letras que se o b têm , m udando a o r­


dem das letras da palavra A R I (anagram as da palavra A R I).
C = (A R I, A IR , IR A , IA R , R A I, R IA ) e # C = 6

4 ? ) D é o co n ju n to de núm eros de três algarism os, tod os distintos, fo r­


m ados a partir dos dígitos 7, 2 , 5 , 4 , 5 , 6 , 7, 8.
D = (123, 124, 125, . . . , 87 5 , 876)

P od e-se perceber que é trab alh o so obter tod os os elem entos (agrupam en­
tos) desse co n ju n to e depois co n tá-lo s. C orre-se o risco de haver om issão ou
repetição de agrupam entos. U sando técnicas que irem os estudar ad iante, vere­
m os que # D = 336.

II. Princípio fundamental da contagem


3. T a l princípio co n sta de duas partes (A e B ) ligeiram ente diferentes. Antes
de enunciar e dem onstrar este princípio, vam os provar dois lem as (teorem as
au xiliares).

4. Lema 1

C onsiderem os os co n ju n to s A = [a, a 2, am\ e B = ( b „ b 2, . . . , b n).


Pod em os form ar m • n pares ordenados ( a if bj) em que a, E A e bj E B.

D em o n stra çã o
Fixem os o prim eiro elem ento do par e façam o s variar o segundo.
T erem os:

(a ,, b ,), (a „ b 2), . . . , (a ,, b n) — n pares


m J (a2, b ,), (a2, b2), . . . , (a2, b n) — n pares
linhas
J a m, b ,), (a m, b 2), . .. , (am, b n) — ^ n pares

O núm ero de pares ordenados é então + n + n + ... + n\ = m • n.

m vezes

U m a o u tra form a de visualisarm os os pares ordenados é através do dia­


g ram a ab aixo , conhecid o co m o d ia g ra m a seq ü en cia ou d ia g ra m a d a árv ore.

2
ANÁLISE COMBINATÓRIA

b, (a ,, b,)
b2 (a „ b 2)

b„ (a „ b„)
b, (a2, b,)
b2 (a 2, b2)

b„ (a2, b n)

b, (am, b,)
b2 (am) b 2)

b,

pares

5. Exemplos

1?) T em os três cidades X 9 Y e Z . E xistem qu atro rodovias que ligam X


com Y e cinco que ligam Y com Z . P artin d o de X e passando por Y , de quantas
form as podem os chegar até Z ?

S ejam :
A o co n ju n to das rodovias que ligam X com Y e
B o co n ju n to das rodovias que ligam Y com Z:
A = |a,, a2, a 3, a4| e B = jb ,, b2, b3, b4, b 5j.

C ada m odo de efetu ar a viagem de X até Z pode ser consid erad o com o
um par de estradas (aiy b j) em que a { E A e bj G B.
L o g o , o núm ero de pares ordenados (ou de m odos de v ia ja r de X até Z ) é
4 • 5 = 20.

3
ANÁLISE COMBINATÔRIA

2 ?) Q uantos núm eros de dois algarism os (distintos ou não) podem ser fo r­


m ados, usando os dígitos 1, 2, 3, 4, 5, 6, 7, 81
C ad a núm ero pode ser considerado um par de dígitos (<a , b ) em que
a G [7 , 2 , 5 , 8} e b E [7 , 2 , 5 , 8 ).
L o g o , o resultado p rocurad o é:
8 • 8 = 64.

6- Lema 2

O núm ero de pares ordenados ( a it dj) tais que


a , G A = [a lf a 2, a m}9 Oj G A = [ aJf a 2f a j e a, a} (p ara / * j ) é
m (m — 7 ).

D em o n stra çã o
Fixem os o prim eiro elem ento do par, e façam o s variar o segundo.

4
ANÁLISE COMBINATÓRIA

T erem o s:

(a„ az), (a„ a3), .... (a„ a,») (m - 1) pares


m J (a2, a,), (a2, a3), (a2> a j (m — 1) pares
linhas ] : : :
^l)> (^m> ^2)» 0*m> ^m-I) (m - 1) pares

O núm ero de pares é:


(m — 1) + (m — 1) + ... + (m — 1) = m • (m — 1).

1 V
J
m vezes

Exemplo

7. Q uantos núm eros com dois algarism os distintos podem os fo rm a r co m os


dígitos 7, 2 , 5 , 4 y 5 , 6, 7 e 81
C ad a núm ero pode ser consid erad o um par de dígitos (a, b) em que
a E {1, 2 , 3, . . . , 8}, b E (1, 2 , 3, . . . , 8) e a * b.

E n tão o resultado procurad o será 8 • 7 = 56.


O bservem os que o d iagram a da árvore pode ser usado p ara obterm o s os
núm eros form ad o s, notand o apenas qu e, um a vez tom ad o um elem ento n a 1?
etapa do d iagram a, ele não pod erá aparecer na 2? etapa.

O princípio fundamental da contagem (parte A)

C onsiderem os r co n ju n to s

A = (a,, a2, •••> ®n,j # A = n,


B = [bj, b2, . . . . b j # B = n2

Z — (z„ z2, # Z = nr

en tão , o núm ero de r-uplas ordenadas (seqüências de r elem entos) do tipo


b j, •••> Zp)
em que n, E A , bj E B ... zp E Z é
n, • n2 • ... • n ,.

5
ANÁUSE COMBINATÓRIA

D em o n stra çã o (P rin cíp io d a in d u ção fin ita )


Se r = 2 , é im ediato, pois caím os no lem a 1 já visto.
Suponham os que a fórm u la seja válida para o inteiro (r - 1) e provem os
que ela tam bém é válida para o inteiro r.
P ara (r - 7 ), tom em os as seqüências de (r — 7 ) elementos (aJy bj9 ..., wk).
P o r hipótese de ind u ção, existem
nj n2 • ... • n^j seqüências e nr elem entos pertencentes ao co n ju n to Z.
C ada seqüência (aif bj9 . . . , wk, zp) consiste de uma seqüência ( ait bjf ...,
wk) e um elem ento zp G Z .
P o rta n to , pelo lem a 1, o núm ero de seqüências do tipo (aif bjt . . . , wkt
zp) é
(n i • n2 . .. nr_ j) • nr = n, • n2 • • nr_| • nr.

D ecorre então que o teorem a é válido V r E IN e r ^ 2.

9. Exemplo
U m a m oeda é lançad a 3 vezes. Q ual o núm ero de seqüências possíveis
de ca ra e c o r o a i
Indiquem os por K o resultado ca ra e por C o resultado c o r o a .
Q uerem os o núm ero de triplas ordenadas (a t b , c ) , em que
a G (K, CJ b G (K, C) e c G (K , C ).

L o g o , o resultado procurad o é:
2 • 2 • 2 = 8.

As seqüências podem ser o btid as através de um diagram a da árvore.

resultados

(£7 k T£)
(K , K , C )

(K , C , K)

(K , C , C)

(C , K , K)

(C , K , C)

(C , C , K)

(C , C , C )

6
ANÁLISE COMBINATÓRIA

10. O princípio fundamental da contagem (parte B)

C onsiderem os um co n ju n to A com m (m ^ 2 ) elem entos. E n tã o o núm e­


ro de r-uplas ordenadas (seqüências com r elem entos) form ad as com elem entos
distintos dois a dois de A é:
m • (m - 1) • (m - 2) • ... • [m - (r - 1)].
v
r fatores

Ou se ja , se A [a„ a 2, . . . , a m}, o núm ero de seqüências do tipo

r elementos

\ E A Vi G (1, 2 , . .. , m] é
com
ai 5* ap para i 5* p

m - (m - 1) • . .. • [m - (r - l)]j

r fatores

A d em onstração é feita por indução fin ita , de m odo an álog o à feita na


parte A .

11. Exemplos
1?) Q u atro atletas participam de uma co rrid a. Q uantos resultados exis­
tem para o 1?, 2? e 3? lugares?

C ad a resultado co n sta de uma tripla ordenada (a , b, c ) , em que a repre­


senta o atleta que chegou em 1? lugar, b o que chegou em segundo, e c o que
chegou em terceiro.

a , b e c pertencem ao co n ju n to dos atletas Q a ^ b , a ^ c c b ^ c .

L o g o , o núm ero de resultados possíveis é:


4 • 3 • 2 = 24.

7
ANÁLISE COMBINATÓRIA

2 ?) D e qu antos m odos três pessoas podem ficar em fila indiana?


C ada m odo corresponde a uma tripla ordenada de pessoas. L o g o , o re­
sultado procurad o é:
3-2-1 =6.

Se cham arm os de A, B e C as pessoas, os m odos podem ser obtid os a tra ­


vés do diagram a de árvore.

m odos

© (A , B, C)

© (A , C , B)

© (B , A , C)

© (B , C , A)

B) (C , A , B)

® (C , B , A)

12. Observação

A lgum as vezes, o co n ju n to cu jo s elem entos querem os co n ta r consta de


seqüências de tam anhos diferentes (isto é, o núm ero de elem entos das seqüên-
cias consideradas é d iferente), o que impede o uso do princípio fundam ental
da contagem . E n tretan to , usando o diagram a de árvore, podem os saber fa cil­
m ente quantas são as seqüências.

13. Exemplo

U m a pessoa lança uma m oeda sucessivam ente até que o co rram duas ca ­
ras consecu tivas, ou qu atro lançam entos sejam feitos, o que prim eiro ocorrer.
Q uais as seqüências de resultados possíveis?

8
A N Á L IS E C O M B IN A T Ó R IA

Tem os:

O s re sultados possíveis são:

(K , K ); (K , C , K , K ); (K , C , K , C ); (K , C , C , K ); (K , C , C , C ); (C , K , K );
(C , K , C , K ); (C , K , C , C ); (C , C , K , K ); (C , C , K , C ); (C , C , C , K ); (C , C , C , C );

e o n ú m e r o de seqüências é 12.

EXERCÍCIOS
1. Um homem vai a um restaurante disposto a comer um só prato de carne e uma
só sobremesa. O cardápio oferece oito pratos distintos de carne e cinco pratos di­
ferentes de sobremesa. De quantas formas pode o homem fazer sua refeição?

2 . Uma moça possui 5 blusas e 6 saias. De quantas formas ela pode vestir uma blusa
e uma saia?

9
ANÁLISE COMBINATÓRIA

3. Num banco de automóvel o assento pode ocupar 6 posições diferentes e o encosto


5 posições, independentemente da posição do assento. Combinando assento e en­
costo, quantas posições diferentes esse banco pode assumir?

4. Numa festa existem 80 homens e 90 mulheres. Quantos casais diferentes podem


ser formados?

5. Um edifício tem 8 portas. De quantas formas uma pessoa poderá entrar no edifí­
cio e sair por uma porta diferente da que usou para entrar?

6. Num concurso com 12 participantes, se nenhum puder ganhar mais que um prê­
mio, de quantas maneiras poderão ser distribuídos um primeiro e um segundo
prêmios?

7. Um homem possui 10 ternos, 12 camisas e 5 pares de sapatos. De quantas formas


poderá ele vestir um terno, uma camisa e um par de sapatos?

8. Um automóvel é oferecido pelo fabricante em 7 cores diferentes, podendo o com­


prador optar entre os motores 2 000 cc e 4 000 cc. Sabendo-se que os automóveis
são fabricados nas versões ‘‘standard” , “ luxo” e “ superluxo” , quantas são as al­
ternativas do comprador?

9. De quantas formas podemos responder a 12 perguntas de um questionário, cujas


respostas para cada pergunta são: sim ou n ãol

Solução

Cada resposta do questionário todo consta de uma seqüência

(a i» a2» •••> a 12)


em que cada <z, vale S (sim) ou N (não). Além disso:

a, E A, = (S, N)
a2 E A2 = [S, NJ

a»2 ê A l2 = (S, N)

Logo, pelo princípio fundamental da contagem, o número de seqüências do


tipo acima é:

[2 •2 • ... • 2 j = 2 12
V
/2 vezes

10. Uma prova consta de 20 testes do tipo verdadeiro ou fa lso . De quantas formas
uma pessoa poderá responder aos 20 testes?

10
ANÁLISE COMBINATÓRIA

I I * Uma loteria (semelhante à loteria esportiva) apresenta 10 jogos, cada um com 4


possíveis resultados. Usando a aproximação 2 W = 10\ qual é o número total de
resultados possíveis? r

12. Em um computador digital, um bit é um dos algarismos 0 ou 1 e uma palavra é


uma sucessão de bits. Qual é o número de palavras distintas de 32 b itsl

13. Uma sala tem 10 portas. De quantas maneiras diferentes essa sala pode ser aberta?

14. De quantas maneiras diferentes um professor poderá escolher um ou mais estu­


dantes de um grupo de 6 estudantes?

15. De um grupo de 5 pessoas, de quantas maneiras distintas posso convidar uma ou


mais para jantar?

16. Quantos anagramas podemos formar, batendo ao acaso em 6 teclas (escolhidas entre
as 26 existentes) numa máquina de escrever? Entre eles consta o anagrama TECTEC?

17. Num concurso para preenchimento de uma cátedra, apresentam-se 3 candidatos.


A comissão julgadora é constituída de 5 membros, devendo cada examinador es­
colher exatamente um candidato. De quantos modos os votos desses examinadores
podem ser dados?

18. Quantos números de 3 algarismos (iguais ou distintos) podemos formar com os


dígitos /, 2, 3, 7, 81

19. Temos um conjunto de 10 nomes e outro de 20 sobrenomes. Quantas pessoas po­


dem receber um nome e um sobrenome, com esses elementos?

20. Um mágico se apresenta em público vestindo calça e paletó de cores diferentes.


Para que ele possa se apresentar em 24 sessões com conjuntos diferentes, qual é
o número mínimo de peças (número de paletós mais número de calças) de que ele
precisa?

21. Seis dados são lançados simultaneamente. Quantas seqüências de resultados são
possíveis, se considerarmos cada elemento da seqüência como o número obtido em
cada dado?

22. O sistema telefônico de São Paulo utiliza sete (7 ) dígitos para designar os diversos
telefones. Supondo que o primeiro dígito seja sempre dois (2 ) e que o dígito zero
(0 ) não seja utilizado para designar estações (2? e 3? dígitos), quantos números
de telefones diferentes poderemos ter?

23. As letras em código morse são formadas por seqüências de traços (—) e pontos ( •),
sendo permitidas repetições. Por exemplo: (—; •; —; —; -I •)•
Quantas letras podem ser representadas:
a) usando exatamente 3 símbolos?
b) usando no máximo 8 símbolos?

11
ANÁLISE COMBINATÓRIA

2 4 . Quantos números telefônicos com 7 dígitos podem ser formados, se usarmos os


dígitos de 0 a 91

Solução

Cada número telefônico consiste em uma sequência de 7 dígitos do tipo:


(a„ a2........a*,, a7) em que a, G A, = (0, 1, 2, .... 9)
a2 G A2 = |0, 1, 2, .... 9)

av G A7 = (0, 1, 2 ....... 9j
Logo, pelo princípio fundamental da contagem, o número de seqüências é:
.10 • 10 • . . . • 10. = 107 = 10 000 000
V--------------- v ---------------
7 vezes

2 5 . Existem apenas dois modos de atingir uma cidade X partindo de uma outra A.
Uma delas é ir até uma cidade intermediária B e de lá atingir X; a outra é ir até
C e de lá chegar a X. (Veja esquema.) Existem 10 estradas ligando A e B\ 12 ligan­
do B a X ; 5 ligando A a C; 8 ligando C a
X ; nenhuma ligação direta entre B e C e
nenhuma ligação direta entre A e X. Qual
o número de percursos diferentes que po­
dem ser feitos para, partindo de A , atin­
gir X pela primeira vez?
C
%
2 6 . Um homem encontra-se na origem de um sistema cartesiano ortogonal de eixos
Ox e Oy. Ele pode dar um passo de cada vez, para norte (N) ou para leste (L).
Quantas trajetórias ele pode percorrer, se der exatamente 4 passos?

Solução

Notemos que cada trajetória consiste em y-


uma quádrupla ordenada (ah a * ap a4)
em que N
a, E (N, L), a2 E (N, L), a3 E [N, L) e L N
a4 e (N, L).
N
Por exemplo, (N, L , N, N ) corresponde
0 X
graficamente a:
Logo, pelo princípio fundamental da contagem, o número de trajetórias (quá­
druplas ordenadas) é 2 • 2 • 2 • 2 = 16.

12
ANÁLISE COMBINATÓRIA

27. Caminhando sempre para a direita ou pa­


ra cima, sobre a rede da figura, de quan­
tas maneiras se pode ir do ponto A até
a reta BC1

28. Resolva o problema anterior, se o homem der exatamente 6 passos. Dê o gráfico


de 3 trajetórias possíveis.

29. Quantos divisores positivos tem o número 38 8 8 = 2* •351

Solução

Cada divisor é um número do tipo 2 a>• 3a*, em que or7 E [0, 1, 2, 3, 4}


cl2 E{0, /, 2, 3, 4, 5).
Exemplo: 23 • 35; 2o • 33; 22 • 3o, etc.
Portanto, o número de divisores é o número de pares ordenados ( a 7, a 2),
que, pelo princípio fundamental da contagem, é:
5 - 6 = 30.

30. Quantos divisores positivos tem o número N = 2a • 3b • 5C• 7*?

31. Cada pedra de dominó é constituída de 2 números. As peças são simétricas, de


sorte que o par de números não é ordenado. Exemplo:

é o mesmo que

Quantas peças diferentes podem ser formadas, se usarmos os números 0, 1, 2, 3 ,


4, 5 e 6 ?

32. Quantas peças diferentes podem ser formadas num jogo de dominó se usarmos
os números 0, 1, 2, 3, ...» n?

33. A t B são conjuntos tais que #/4 = n e # £ = r. Quantas funções f : A -*■ B


existem?

13
ANÁLISE COMBINATÓR1A

34. Em um baralho de 52 cartas, cinco cartas são escolhidas sucessivamente. Quantas


são as seqüências de resultados possíveis:
a) se a escolha for feita com reposição?
b) se a escolha for feita sem reposição?

Solução

a) Seja A o conjunto das cartas do baralho. Temos #v4 = 52.


Cada escolha consta de uma seqüência do tipo
0*1» a 2» a 3» a 4> a s )

em que aj E A, a2 E A, a3 E A, a4 E A, a5 E A (pois a escolha foi


feita com reposição). Logo, pelo princípio fundamental da contagem (parte
A), o número de seqüências é:
|52 • 52 •52 • 52 •52j = 525
V--------------v/------------- '
5 vezes

b) Se a escolha é feita sem reposição, então cada seqüência (a]t a2, a3,
a5) é tal que cada elemento pertence slA e são todos elementos distintos.
Logo, pelo princípio fundamental da contagem (parte B), o número de se­
qüências é:
[52 • 51 •50 •49 •48j = 311 875 200
' V

5 fatores

3 5 . Duas pessoas, Antônio e Benedito, praticam um jogo no qual em cada partida há


um único vencedor. O jogo é praticado até que um deles ganhe 2 partidas consecu­
tivas ou 4 partidas tenham sido jogadas, o que ocorrer primeiro. Quais as seqüên­
cias possíveis de ganhadores?
(Sugestão: Construa o diagrama da árvore.)

3 6 . Uma urna tem 10 bolinhas numeradas 7, 2, 3, ..., 10. Três bolinhas são extraídas
sucessivamente, sem reposição. De quantas formas os números das bolinhas for­
mam uma P .A . na ordem em que foram extraídas?
(Sugestão: Construa o diagrama da árvore.)

3 7 . Uma moto tem combustível suficiente para somente três voltas num circuito. Pe­
dro, Manoel e Antônio disputam, por meio do lançamento de uma moeda, a opor­
tunidade de dar cada volta, do seguinte modo:
I. o lançamento da moeda é efetuado antes de cada volta;
II. se coroa, a vez é de Manoel;
III. se cara, a vez é de Pedro;
IV. se a mesma face ocorrer consecutivamente, a vez é de Antônio.
Se a primeira volta for dada por Pedro, quantas voltas poderá dar Antônio?

14
ANÁLISE COMB1NATÓR1A

3 8 . Suponha que no início de um jogo você tenha R$2 000,00 e que só possa jogar
enquanto tiver dinheiro. Supondo que em cada jogada você perde ou ganha
R$ 1 000,00 , quais são os possíveis resultados ao final de três jogadas?

3 9 . Um homem tem oportunidade de jogar no máximo 5 vezes na roleta. Em cada jo ­


gada, ele ganha ou perde R I 1 000,00. Começará com R$ I 000,00 e parará de
jogar antes de cinco vezes, se perder todo seu dinheiro ou se ganhar R$3000,00 ,
isto é, se tiver R $4 000,00. De quantas maneiras o jogo poderá se desenrolar?

40 . Em um baile há r rapazes e m moças. Um rapaz dança com 5 moças, um segundo


rapaz dança com 6 moças, e assim sucessivamente. O último rapaz dança com to­
das as moças. Qual é a relação entre m e r?

III. Conseqüências do princípio fundamental da


contagem

O princípio fundam ental da contagem nos forn ece o instrum ento básico
para a Análise C o m b in ató ria; en tretan to , sua ap licação direta na resolução de
problem as pode às vezes to rn ar-se trabalh o sa. Irem os então definir os vários
modos de form ar agrupam entos e, usando sím bolos sim plificativos, deduzir fó r­
mulas que perm itam a contagem dos m esm os, em cad a caso particular a ser
estudado.

IV. Arranjos com repetição


1

14. S e ja M um co n ju n to com m elem entos, isto é, M = j a„ a 2, ..., a m\. C ha­


mamos a rra n jo c o m r e p etiçã o dos m elem entos, tom ados r a r , tod a r-upla o r­
denada (seqüência de tam anho r) form ada com elem entos de M não necessa­
riam ente distintos.

15. Exemplo
U m a urna contém uma bola verm elha ( V ), uma branca ( B ) e um a azul
(/4). U m a bola é extraíd a, observada sua co r e reposta na urna. Em seguida
outra bola é extraída e observada sua co r. Q uantas são as possíveis seqüências
de cores observadas?

15
ANÁLISE COMBINATÓRIA

T em os:
* C ada seqüência é um par ordenado de cores (x, y ) em que x , y E M =
[ V, B , A ). L o g o , pelo princípio fundam ental da contagem (parte A ), o núm ero
de pares é

3*3 = 9.

16. Fórmula do número de arranjos com repetição

S e ja M = [a„ a 2, a mJ e indiquem os por ( A R ) m r o núm ero de arran ­


jo s com repetição de m elem entos tom ados r a r .
C ad a a rra n jo com repetição é um a seqüência de r elem entos, em que ca ­
da elem ento pertence a M.

i(“ » ” , ” , •••» ” )i

r elementos *

P elo princípio fundam ental da contagem (parte A ), o núm ero de arran -


jo s ( A R ) „ r será:

(AR)mr = |in •m •... 3


ii
l ?

r vezes

O bservem os que, se r = 7, ( A R ) m , = m e a fórm ula acim a con tin u a


válida V r E IN*.

V. Arranjos

1 7 . S e ja M um co n ju n to com m elem entos, isto é, M = ja„ a2, a mJ. C ha­


m am os de a rra n jo dos m elem entos tom ados r a r ( / ^ r ^ m ) a qualquer
r-upla (seqüência de r elem entos) form ada com elem entos de M , to d o s distintos.

18. Exemplo

M = (a, b, c , dj.
Os a rran jo s dos qu atro elem entos de A/, tom ad os dois a d ois, são os pa­
res ordenados ( x , y ) form ad os com elem entos distintos de M .

16
ANÁLISE COMBINATÓRIA

P elo princípio fundam ental da contagem (parte B ), o núm ero de pares


ordenados é:
4 • 3 = 12.

19. Fórmula do número de arranjos


S e ja Aí = j a„ a 2, . . . , a m\ e indiquem os por A m r o núm ero de a rra n jo s
dos m elem entos tom ados r a r .
C ada a rra n jo é uma seqüência de r elem entos, em que cad a elem ento per­
tence a A í , e são todos distintos.

i(~ ~ V
-)i
r elementos

Pelo princípio fundam ental da contagem (parte B ), o núm ero de a rra n jo s


Am r será:

E m p articular, se r = /, é fácil perceber que A m , = m .


N otem os ainda que, de aco rd o com a d efin ição que dem os de a rra n jo ,
temos necessariam ente / ^ r ^ m .

20. Exemplo

D e um baralh o de 52 cartas, 3 cartas são retiradas sucessivam ente e sem


reposição. Q uantas seqüências de cartas é possível ob ter?
N otem os que cad a resultado é uma tripla ordenada de cartas ( x , y , z ),
cm que J t é a i . fl ca rta extraíd a, y a 2? e z a 3 ? . O bservem os que x, y, z são
todas d istintas, visto que a ex tra çã o é f e i t a sem rep o siçã o .
L o g o , o núm ero que querem os é A 52. j , isto é:
A 52í3 = (52 • 51 • 50j = 132 600
V------------------V------------------y

3 fatores

17
ANÁLISE COMB1NATÓRIA

VI. Permutações

21. S e ja M um co n ju n to com m elem entos, isto é, M = [a„ a 2, ..., am\. C ha­


m am os de perm utação dos m elem entos a tod o a rra n jo em que r = m .

22. Exemplo

S e ja M = [a, b , c).
As perm utações dos elem entos de M são todos os a rra n jo s constituídos
de 3 elem entos.
S ã o eles:
(a, b, c) (b, a, c) (c, a , b) (a, c, b) (b , c , a) (c, b , a).

2 3 . Fórmula do número de permutações

S e ja M o co n ju n to M = [a„ a 2, amj e indiquem os por P m o núm ero


de perm utações dos m elem entos de M .
T em os:

4P m = 1A*m , m

logo: P m = m(m - 1) • (m - 2) • ... • [m - (m - 1)]

Pm = m *(m - 1) •(m - 2) • •3 •2 •1

E m particular, se m = /, é fácil perceber que P , = 1.

24. Exemplo

D e qu antas form as podem 5 pessoas fica r em fila indiana?


N otem os que cad a fo rm a de ficar em fila indiana é um a perm utação das
5 pessoas. O núm ero de perm utações (m odos de fica r em fila indiana) será:
P 5 = 5 •4 •3 - 2 - 1 = 120.

18
ANÁLISE COMBINATÓRIA

VII. Fatorial

25. A fim de sim p lificar as fórm ulas do núm ero de a rra n jo s e do núm ero de
perm utações, bem co m o outras que irem os estudar, vam os d efin ir o sím bolo
fatorial.
S e ja m um número inteiro não negativo (m E IN). D efinim os fa to r ia l d e m
(e indicam os por m !) por m eio da relação:
m ! = m • (m - 1) • (m - 2) • . .. • 3 • 2 - 1 para m ^ 2
1! = 1
0! = 1

As definições 1! e 01 serão ju stificad as posteriorm ente.

26. Exemplo

3! = 3 - 2 - 1 = 6
4! = 4 • 3 • 2 • 1 = 24
5! = 5 - 4 - 3 - 2 1 = 120.

27. O cálculo de m f 9 diretam ente, torna-se trabalh oso à m edida que aum enta.
(10! = 3 628 800)

E n tretan to , m uitos cálcu los podem ser sim plificados se n o tarm o s que:
(n + 1)! = (n + 1) • n - ( n - 1 ) ■ ... ■ 3 • 2 ■ lj = ( n + 1 ) - n!

28. Exemplos
10!
1?) C alcular
9!
~ 10! 10 -J>T 1A
T em os: —^ = — r f : — = 10.
9!

10!
2 ?) C alcu lar
8!
10! 10 • 9 -JMT
T em os: = 90.
8! M

19
ANÁLISE COMBINATÓRIA

12!
3?) C alcu lar
9! 3!
T em os:
12! = 12 • 11 • 10 M 12 • 11 • 10
220.
9! 3! ^9T3! 3-2-1

29. As fórm ulas do núm ero de a rra n jo s e do núm ero de perm utações tam ­
bém podem ser sim plificad as com a no tação fa to ria l.
D e fa to :

Pm = m • ( m - 1 ) • ... • 3 • 2 • 1 = m!
r = m • ( m - 1 ) • . .. • ( m - r + 1 ) =
(m - r) - (m - r - 1) 3*2-1
= m •(m - 1) • . .. • ( m - r + 1) •
(m - r) •(m - r - 1) 3 •2 • 1

m!
^m , r
(m -r)!

E m particular
P, = 1
1! = 1
e a fórm ula P m = m ! é válida V m E IN*

e ainda:

rA m , = m V m G IN*
em particular 1 m!
= m V m E IN*,
(m -1 )!

ml
e a fórm ula A m r = é válida V m E IN*, V r E IN* com r ^ m .
(m-r)!

EXERCÍCIOS
4 1 . Usando o diagrama da árvore, obtenha todos os arranjos dos elementos de
M = [a, b, c, d ) tomados dois a dois.

20
ANÁLISE COMB1NATÓR1A

4 2 . Calcule:
a) A *,3 b) A ,0i4 c) A2oj d) A|2f2

4 3 . Em um campeonato de futebol, participam 20 times. Quantos resultados são pos­


síveis para os três primeiros lugares?

4 4 . Dispomos de seis cores diferentes. Cada face de um cubo será pintada com uma
cor diferente, de forma que as seis cores sejam utilizadas. De quantas maneiras
diferentes isso pode ser feito, se uma maneira é considerada idêntica a outra, desde
que possa ser obtida a partir desta por rotação do cubo?

4 5 . Em um torneio (de dois turnos) do qual participam seis times, quantos jogos são
disputados?

4 6 . Dispomos de 8 cores e queremos pintar uma bandeira de 5 listras, cada listra com
uma cor. De quantas formas isso pode ser feito?

Solução

Cada maneira de pintar a bandeira consiste de uma seqüência de cinco cores


distintas (seqüência, porque as listras da bandeira estão numa ordem) esco­
lhidas entre as oito existentes. Logo, o número de seqüências procurado é:
A8,5 = [8 • 7 • 6 • 5 • 4j = 6720.
V '
n fatores

4 7 . Uma bandeira é formada de 7 listras, que devem ser pintadas de 3 cores diferentes.
De quantas maneiras distintas será possível pintá-la de modo que duas listras adja­
centes nunca estejam pintadas da mesma cor?

4 8 . Uma linha ferroviária tem 16 estações. Quantos tipos de bilhetes devem ser im­
pressos, se cada tipo deve assinalar a estação de partida e de chegada, respecti­
vamente?

4 9 . Designando-se seis cidades por A, B , C, D, E c F , determine o número de maneiras


que permitem a ida de A até F , passando por todas as demais cidades.

50. As 5 finalistas do concurso para Miss Universo são: Miss Japão, Miss Brasil, Miss
Finlândia, Miss Argentina e Miss Noruega. De quantas formas os juizes poderão
escolher o primeiro, o segundo e o terceiro lugares nesse concurso?

51. Um cofre possui um disco marcado com os dígitos 0, /, 2, ..., 9. O segredo do


cofre é formado por uma seqüência de 3 dígitos. Se uma pessoa tentar abrir o co­
fre, quantas tentativas deverá fazer (no máximo) para conseguir abri-lo. (Suponha
que a pessoa sabe que o segredo é formado por dígitos distintos.)

21
ANÁLISE COMBINATÓRIA

5 2 . De quantas maneiras um técnico de futebol pode formar um quadro de 11 jogado­


res, escolhidos entre 22, dos quais 3 são goleiros e só o goleiro tem posição fixa?

5 3 . No jogo de loto, de uma urna contendo 90 pedras numeradas de 1 a 90, quatro


pedras são retiradas sucessivamente ; qual é o número de extrações possíveis, tal
que a terceira pedra seja 801

5 4 . Existem 10 cadeiras numeradas de / a 10. De quantas formas duas pessoas podem


se sentar, devendo haver ao menos uma cadeira entre elas.

Solução

Inicialmente notemos que cada maneira de elas se sentarem corresponde a


um par ordenado de números distintos escolhidos entre 7, 2, ..., 10.
ía pessoa A se senta na cadeira 2
Exemplo: (2, 6)
[a pessoa B se senta na cadeira 6
[ a pessoa A se senta na cadeira 6
(6 , 2)
[a pessoa B se senta na cadeira 2
fa pessoa A se senta na cadeira 3
( 3 ,4 )
(a pessoa B se senta na cadeira 4
Inicialmente, calculemos o total de pares ordenados, que é igual a
A l02 = 10 9 = 90.
Agora temos que excluir os pares ordenados cujos elementos sejam números
consecutivos. São eles:
(1, 2) (2, 3) (3, 4) ... (9, 10) : 9 pares
(2, 1) (3, 2) (4, 3) ... (10, 9) : 9 pares
Ao todo, devemos excluir 9 + 9 = 18 pares.
Logo, o número de maneiras de as pessoas se sentarem, havendo ao menos
uma cadeira entre elas, é 90 —1 8 = 72.
É bastante importante o leitor notar a razão pela qual cada maneira é um
par ordenado.
(-------- , ----------)
t t
senta-se senta-se
A B

5 5 . Uma uma contém m bolas numeradas de 1 até m; r (r < m ) bolas são extraídas
sucessivamente. Qual o número de seqüências de resultados possíveis se a extração
for:
a) com reposição de cada bola após a extração?
b) sem reposição de cada bola após a extração?

22
ANÁLISE COMBINATÓR1A

56. Uma uma I contém 5 bolas numeradas de 7 a 5. Outra uma II contém 3 bolas
numeradas de 7 a 3. Qual o número de seqüências numéricas que podemos obter
se extrairmos, sem reposição, 3 bolas da uma I e, em seguida, 2 bolas da urna IJ.

57. Existem duas urnas. A 1? com 4 bolas numeradas de 7 a 4 e a 2? com 3 bolas nu­
meradas de 7 a 9. Duas bolas são extraídas da 7.fl urna, sucessivamente e sem repo­
sição, e em seguida 2 bolas são extraídas da 2? uma, sucessivamente e sem reposição.
Quantos números (de 4 algarismos) é possível formar nessas condições?

58. Se A e B são conjuntos e #A = n e # B - r, quantas funções f : A -» B, injeto-


ras, existem? (7 < n ^ r)

Notemos que, s e / é injetora, então / ( a ,) * /(<?,) para todo at * a}.


Por outro lado, cada função vai ser definida por uma /t-upla de imagens,
em que todos os elementos da /z-upla devem ser distintos, pois a função é
injetora. ,
Por exemplo, uma das funções é definida pela n-upla de imagens.
(b„ b2, .. ., bfc, bk+i> bn)
t t t t t
f(a,) f(a2) f(ak) f(ak+I) f<a„)
Outra função é definida pela n-upla:!
(b„, V i, . •• b|c+1» bk» ^2» b.)
t t t t t t
f(a,) f(a2) f<Vk> f(V k+i> f<an)
Logo, o número de funções é o número de arranjos dos r elementos de B ,
tomados n a n , isto é, A r n = —----—— .
(r-n )!

59. Qual é o número de funções injetoras definidas em A = (7, 2, 3} com valores em


B = (0, 7, 2, 3, 4}1

60. Sejam A um conjunto finito com m elementos e /„ = (7, 2, ..., n}. Qual é o nú­
mero de todas as funções definidas em In com valores em A ?

23
ANÁLISE COMB1NATÓRIA

6 1 . Sejam A e B dois conjuntos tais que #>4 = #7? = n > 0. Quantas funções
f: A B bijetoras existem?

6 2 . Com os algarismos 7, 2 , 3, 4, 5, 6, 7, 8 c 9, quantos números de 3 algarismos dis­


tintos podemos formar?

6 3 . Qual é a quantidade de números de 3 algarismos que têm pelo menos 2 algarismos


repetidos?

6 4 . Quantos números pares de 3 algarismos distintos podemos formar com os algaris­


mos 7, 3, 6, 7, 8, 9?

Solução

Cada número será uma tripla ordenada de algarismos escolhidos entre os da­
dos. Como estamos interessados nos números pares, então nos interessam
as triplas do tipo:
(-, 6) Q
OU

(-, 8) ©
O número de triplas do tipo (7) é As>2 = 20 e o de triplas do tipo ©
é A „ = 20.
Logo, o resultado procurado é 20 + 20 = 40.

6 5 . Há placas de automóveis que são formadas por duas letras seguidas de 4 algaris­
mos. Quantas placas podem ser formadas com as letras A e B e os algarismos pa­
res, sem repetir nenhum algarismo?

6 6 . Com os algarismos 7, 2, 3, 4, 5, 6, 7, 8 e 9, quantos números com algarismos dis­


tintos existem entre 500 e 7 0001

6 7 . Com os algarismos 7, 2, 3 ,4 , 5, quantos números de 3 algarismos (iguais ou distin­


tos) existem?

6 8 . Com os algarismos 7, 2, 3 , ..., 9, quantos números de quatro algarismos existem,


em que pelo menos dois algarismos são iguais?

6 9 . Quantos números formados por 3 algarismos distintos escolhidos entre 2, 4, 6, 8,


9 contêm o 2 e não contêm o 61 (Lembre-se que o 2 pode ocupar a 1?, 2? ou a
3? posição.)

7 0 . Com os dígitos 1, 2, 3, 4, 5, 6, quantos arranjos desses dígitos tomados 4 a 4 têm


o dígito 7 antes do 41

24
ANÁLISE COMB1NATÓR1A

71. Com os algarismos 1, 2, 3, 4, 5, 6, quantos números pares de 3 algarismos distin­


tos podemos formar?

72. Quantos números ímpares de 4 algarismos, sem repetição, podem ser formados
com os dígitos /, 2, 3, 4, 5 e 61

73. Com os dígitos 2, 5, 6, 7, quantos números formados por 3 dígitos distintos ou


não são divisíveis por 5?

74. Com os algarismos /, 2, 3, 4, 5 e 6 são formados números de 4 algarismos distin­


tos. Dentre eles, quantos são divisíveis por 5?

75. Qual é o total de números múltiplos de 4 , com quatro algarismos distintos, que
podem ser formados com os algarismos 1, 2, 3, 4, 5 z 6 !

76. Formados e dispostos em ordem crescente todos os números que se obtêm permu-
tando os algarismos /, 2, 4, 6 , 8 , que lugar ocupa o número 684121

Solução

Esse número é precedido pelos números da forma:

(1, - ) que são em número de P4 = 4!

(2, - , - , - ) que são em número de P4 = 4!

(4, - , - , - , - ) que são em número de P4 = 4!

(6, 1, - , —, - ) que são em número de P 3 = 3!

(6, 2,—) que são em número de P3 = 3!

(6, 4, - , - ) que são em número de P3 = 3!

(6, 8, 1, —, - ) que são em número de P2 = 2!

Tjí) (6, 8, 2, - ) que são em número de P2 = 2!

De o © ....... (v m ) concluímos que 68 412 é precedido por um to-

tal de 41 + 41 + 4! + 3! + 3! + 3! + 2/ + 21 = 94 números. Portanto, a posi­


ção de 68412 é a 95 f.

25
ANÁLISE COMBINATÓRIA

7 7 . Com os algarismos 1, 2, 3, 4 e 5 e sem repetição, pode-se escrever x números maio­


res que 2 500. Qual é o valor de x l

7 8 . Com os algarismos 0, 1 ,2 , 5 e 6 , sem os repetir, quantos números compreendidos


entre 100 e 1 000 poderemos formar?

7 9 . Formados e dispostos em ordem crescente os números que se obtêm permutando


os algarismos 2, 3, 4, 8 e 9, que lugar ocupa o número 43 8921

8 0 . De quantas formas podemos preencher um cartão da loteria esportiva, com um


único prognóstico duplo e todos os outros, simples?

8 1 . Uma peça para ser fabricada deve passar por 7 máquinas, sendo que a operação
de cada máquina independe das outras. De quantas formas as máquinas podem
ser dispostas para montar a peça?

8 2 . Consideremos m elementos distintos. Destaquemos k dentre eles. Quantos arran­


jos simples daqueles m elementos, tomados n a n (A m n), podemos formar, de mo­
do que em cada arranjo haja sempre, contíguos e em qualquer ordem de coloca­
ção, r (r < n) dos k elementos destacados?

8 3 . Com relação à palavra TEO RIA :


a) Quantos anagramas existem?
b) Quantos anagramas começam por TI
c) Quantos anagramas começam por T e terminam com A l
d) Quantos anagramas começam por vogal?
e) Quantos anagramas têm as vogais juntas?

Solução

a) Cada anagrama é uma permutação das letras T , E , O, R, I, A. Logo,


o número procurado é:
P6 = 6! = 720.

b) T _____________
Neste caso femos somente que permutar as letras E, O, R, I, A. Logo,
o número procurado é:
P5 = 5! = 120.

c) T __________ A
Neste caso temos somente que permutar as letras E , O, R, I. Logo, o nú­
mero procurado é:
P4 = 4! = 24.

26
ANÁLISE COMBINATÓRIA

d) Temos as seguintes possibilidades:


A _______________ 5! = 120 anagramas
E ________________5! = 120 anagramas
I ________________5! = 120 ramas
O ________________5! = 120 anagramas
Logo, ao todo teremos: 120 + 120 + 120 + 120 = 480 anagramas.

e) Se as vogais A, E , I, O devem estar juntas, então elas funcionam como


“ uma letra” que deve ser permutada com T e R . Logo, o número de per­
mutações é:

P3 = 3! = 6.

Todavia, em cada uma dessas permutações, as vogais podem se permutar


entre si, de 4! - 24 formas.

Logo, o número de anagramas nessas condições é:

6 • 24 = 144.

8 4 . Quantos anagramas da palavra FIL TR O começam por consoantes?

8 5 . Calcule o número total de inteiros positivos que podem ser formados com os alga­
rismos /, 2, 3 e 4, se nenhum algarismo é repetido em nenhum inteiro.

8 6 . Uma palavra é formada por N vogais e N consoantes. De quantos modos distintos


podem ser permutadas as letras dessa palavra, de modo que não apareçam juntas
duas vogais ou duas consoantes? 1

8 7 . Quantas palavras distintas podemos formar com a palavra PERNAMBUCO? Quan­
tas começam com a sílaba PER?

8 8 . Quantos anagramas da palavra PA STEL começam e terminam por consoante?

8 9 . Calcule o número de anagramas da palavra R EPÚ BLIC A , nos quais as vogais se


mantêm nas respectivas posições.

9 0 . De quantas formas podemos colocar 8 torres num tabuleiro de xadrez de modo


que nenhuma torre possa “ comer” outra?

9 1 . Em um “ horário especial” , um diretor de televisão dispõe de 7 intervalos para anún­


cios comerciais. Se existirem 7 diferentes tipos de anúncios, de quantas formas o
diretor poderá colocar os 7 nos intervalos destinados a eles?

27
92. Dez pessoas, entre elas Antônio e Beatriz, devem ficar em fila. D c quantas formas
isso pode ser feito se Antônio e Beatriz devem ficar sempre juntos?

Solução

Se Antônio e Beatriz devem ficar juntos, eles funcionam como “ uma única
pessoa” , que junto com as outras 8 devem ser permutadas, dando um total
de 9! permutações.

Entretanto, em cada uma dessas permutações, Antônio e Beatriz podem ser


permutados entre si (A B ou BA ) de 21 = 2 formas.

Logo, o total de permutações em que eles aparecem juntos (A B ou BA) é:


2 • 9!

9 3 . De quantas formas 4 homens e 5 mulheres podem ficar em fila, se:


a) os homens devem ficar juntos;
b) os homens devem ficar juntos e as mulheres também?
i

9 4 . Temos 5 meninos e 5 meninas. De quantas formas eles podem ficar em fila se me­
ninos e meninas ficam em posições alternadas?

9 5 . Considere um teste de múltipla escolha, com 5 alternativas distintas, sendo uma


única correta. De quantos modos distintos podemos ordenar as alternativas, de ma­
neira que a única correta não seja nem a primeira nem a última?

9 6 . De quantas maneiras três casais podem ocupar 6 cadeiras, dispostas em fila, de


tal forma que as duas das extremidades sejam ocupadas por homens?

9 7 . De quantas formas 6 pessoas podem se sentar numa fileira de 6 cadeiras se duas


delas (Geraldo e Francisco) se recusam a sentar um ao lado do outro?

9 8 . As placas dos automóveis são formadas por três letras seguidas de quatro algaris­
mos. Quantas placas podem ser formadas com as letras A, B e C junto com os
algarismos pares, sem haver repetição de letras ou de algarismos?

9 9 . No sistema decimal, quantos números de cinco algarismos (sem repetição) pode­


mos escrever, de modo que os algarismos 0 (zero), 2 (dois) e 4 (quatro) apareçam
agrupados?
Obs.: Considere somente números de 5 algarismos em que o primeiro algarismo
é diferente de zero.

28
1 0 0 . De quantas formas 4 pessoas podem se sentar ao redor de uma mesa circular?

Solução

Quando elementos são dispostos ao redor de um círculo, a cada disposição


possível chamamos perm utação circular. Além disso, duas permutações cir­
culares são consideradas idênticas se, e somente se, quando percorremos
a circunferência no sentido anti-horário a partir de um mesmo elemento
das duas permutações, encontramos elementos que formam sequências
iguais.
Por exemplo, consideremos as permutações circulares:

Tomando o elemento A , a seqüência


encontrada é (A, C, D, B ).

D
B

Tomando o elemento A f a seqüência


encontrada é ( A , C, D, B).

Logo, as duas permutações circulares são iguais. A igualdade das duas


perm utações circulares acima podería ser observada, tomando-se outro ele­
mento diferente de A . Por exemplo, D. Em (1) encontraremos a seqüência
( A B , A, C ) e em (2) encontraremos a seqüência ( A B , A , C ). Para resol­
vermos o exercício proposto, chamcmoç de x o número de permutações cir­
culares. Notemos que a cada permutáção circular de A , B t C, D correspon­
dem 4 permutações de A f B t C, D.
Por exemplo: à perm utação circular do exemplo correspondem as permu­
tações: I
(A, C, D, B),
(C, D, B, A),
(D, B, A, C),
(B , A, C, D).
Por outro lado, no conjunto das permutações, a cada quatro permutações
corresponde uma única permutação circular. Por exemplo:
(A, B, D, C)
(B, D, C, A)
(D, C, A, B)
(C, A, B, D)

29
ANÀI.ISt COMBINATÓR1A

9 2 . Dez pessoas, entre elas Antônio e Beatriz, devem ficar em fila. De quantas formas
isso pode ser feito se Antônio e Beatriz devem ficar sempre juntos?

Solução

Se Antônio e Beatriz devem ficar juntos, eles funcionam como “ uma única
pessoa” , que junto com as outras 8 devem ser permutadas, dando um total
de 9! permutações.

Entretanto, em cada uma dessas permutações, Antônio e Beatriz podem ser


permutados entre si (A B ou B A ) de 2! = 2 formas.

Logo, o total de permutações em que eles aparecem juntos (A B ou B A ) é:


2 • 9!

9 3 . De quantas formas 4 homens e 5 mulheres podem ficar em fila, se:


a) os homens devem ficar juntos;
b) os homens devem ficar juntos e as mulheres também?

9 4 . Temos 5 meninos e 5 meninas. De quantas formas eles podem ficar em fila se me­
ninos e meninas ficam em posições alternadas?

9 5 . Considere um teste de múltipla escolha, com 5 alternativas distintas, sendo uma


única correta. De quantos modos distintos podemos ordenar as alternativas, de ma­
neira que a única correta não seja nem a primeira nem a última?

9 6 . De quantas maneiras três casais podem ocupar 6 cadeiras, dispostas em fila, de


tal forma que as duas das extremidades sejam ocupadas por homens?

9 7 . De quantas formas 6 pessoas podem se sentar numa fileira de 6 cadeiras se duas


delas (Geraldo e Francisco) se recusam a sentar um ao lado do outro?

9 8 . As placas dos automóveis são formadas por três letras seguidas de quatro algaris­
mos. Quantas placas podem ser formadas com as letras A, B e C junto com os
algarismos pares, sem haver repetição de letras ou de algarismos?

9 9 . No sistema decimal, quantos números de cinco algarismos (sem repetição) pode­


mos escrever, de modo que os algarismos 0 (zero), 2 (dois) e 4 (quatro) apareçam
agrupados?
O bs.: Considere somente números de 5 algarismos em que o primeiro algarismo
é diferente de zero.

28
/
ANÁLISE COMBINATÓRIA

100. De quantas formas 4 pessoas podem se sentar ao redor de uma mesa circular?

Solução

Quando elementos são dispostos ao redor de um círculo, a cada disposição


possível chamamos perm utação circular. Além disso, duas permutações cir­
culares são consideradas idênticas se, e somente se, quando percorremos
a circunferência no sentido anti-horário a partir de um mesmo elemento
das duas permutações, encontramos elementos que formam sequências
iguais.
Por exemplo, consideremos as permutações circulares:

Tomando o elemento A , a seqüência


encontrada é (A , Ç, D, B ).

D
B

Tomando o elemento A , a seqüência


encontrada é ( A , C, D , B ).

Logo, as duas permutações circulares são iguais. A igualdade das duas


permutações circulares acima podèria ser observada, tomando-se outro ele­
mento diferente de A . Por exemplo; Z>: Em (1) encontraremos a seqüência
(D, B, A , C ) e em (2) encontraremos a seqüência (D , B, A , C ). Para resol­
vermos o exercício proposto, chamemos de x o número de perm utações cir­
culares. Notemos que a cada permutação circular de A , B, C, D correspon­
dem 4 permutações de A , B y C, D. I
Por exemplo: à perm utação circular do exemplo correspondem as permu­
tações:
(A, C, D, B),
(C, D, B, A),
(D, B, A, C),
(B , A, C, D).
Por outro lado, no conjunto das permutações, a cada quatro permutações
corresponde uma única permutação circular. Por exemplo:
(A, B, D, C)
(B, D, C, A)
(D, C, A, B)
(C, A, B, D)

29
ANÁLISE COMBINATÓRIA

correspondem à permutação circular:

O '
A cada conjunto de 4 permutações que definem uma determinada permu­
tação circular chamamos classe.
Como temos x permutações circulares, teremos x classes.
Observemos que a interseção de duas classes distintas é o conjunto vazio.
Logo, o número de permutações de A, B, C, D pode ser calculado de dois
modos:
1?) P4 = 4!
2?) existem x classes, cada qual com 4 permutações; logo, 0 total de per­
mutações é 4 •x.
Portanto:

4 •x = 4! => x = = 3! = 6.
4

Observação
Com raciocínio análogo ao anterior, podemos calcular o número de per­
mutações circulares de n(n ^ 2 ) elementos, da seguinte forma:
1) existem n! permutações dos n elementos;
2) existem x perm utações circulares em que a cada uma correspondem n
permutações.

Logo: n •x = n! X = = (n - 1)!

que é o número de permutações circulares de n elementos.

1 0 1 . De quantas formas 12 crianças podem formar uma roda?

1 0 2 . Quantos colares podemos formar usando quatro contas, todas diferentes?

1 0 3 . Temos m meninos e m meninas. De quantas formas eles podem formar uma ro­
da, de modo que os meninos e as meninas se alternem?

Sugestão: Suponha m = 3 e forme primeiro a roda só com meninos. Depois


que o leitor “ sentir” o problema para m = 5, deve resolver para m qualquer.

30
ANÁLISE COMBINATÓRIA

1 0 4 . Mostre que:
a) 5! + 7! * 12!
b) 8! - 3! * 5!
c) 2 • (5!) * (2 • 5)!

1 0 5 . Resolva a equação: A n 4 = 12 •A n 2.

Solução

Observemos que a equação só tem solução para n ^ 4.

Temos:
n(n - 1) (n - 2) (n — 3) = 12 • n • (n — 1).
Como n (n - 1) ^ 0 , resulta:
(n - 2) (n - 3) = 12
n2 — 5n + 6 = 12

—1 (não convém)

o conjunto solução é (6).

106. Obtenha m , sabendo que: — = 4.


M a

107. Se y4/> 1,3 = — , calcule a.


A *.3 4

108. Resolva a equação: Am 3 = 50 m.

109. Obtenha m na equação (m + 2 ) ! = 72 • m !

110. Resolva a equação (n - 6 ) ! = 720.

111. Calcule n , sabendo que 2 A n 2 + 50 ~ A 2„'2‘

112. Prove que, v n E N com n ^ 2, n! — (n - 2 ) ! = ( n2 — n — 1) (n — 2 ) !

113. Prove que:


n
n! (n + 1)! (n + 1)!
b) (m!)2 = [(m + 1)! — m!] • (m - 1)!

31
ANÁLISE COMBINATÓRIA

114. Exprima mediante fatoriais 2 • 4 • 6 • 8 • ... • (2 • n).

Solução

2 •4 •6 • ... • (2n) = (2 • 1) •(2 • 2) • (2 • 3) • ... • (2 • n) =


= p • 2 • 2 • ... •2)j (1 • 2 • 3 • ... • n) = 2n • n!
y/
/i fatores

115. Exprima mediante fatoriais:


a) 1 •3 •5 • ... - (2n - 1)
b) l 2 • 22 •32 • ... •n2.

(k !Y
116. Simplifique a expressão
[(k — l) ! } 2 9

(n - r + /)/
117. Simplifique a expressão
(n-f-/)/ *

118. Simplifique a expressão [( u h -2)/ - (m + l)!)m !

1 1 9 . Simplifique a expressão 1 • // + 2 • 21 + ... + m • m/, sabendo que


m • m/ = (m + /)/ - m/

1 2 0 . Simplifique a expressão n2 • (n - 2)/ para n ^ 2.

1 2 1 . Prove que:

y — i— = i --------- 1—
i-e , (i + 1)! (n + 1)!

Sugestão: Desenvolva a somatória e use a identidade:

1 _ 1 _ i
i! (i + D! 0 + 1)!

1 2 2 . Mostre que:
_L . 1 = n+ 2
n! (n + 1)! (n + 1)!

( n + 2)/ + (rt + /)•(/! - 1)!


1 2 3 . Mostre que é um quadrado perfeito.
(n + 1) •(/i 7)/

32
ANÁLISE COMBINATÓR1A

VIII. Combinações

30. Seja M um conjunto com m elementos, isto é, M = \ah a2, am\. Cha­
mamos de combinações dos m elementos, tomados r a r , aos subconjuntos de
M constituídos de r elementos.

3 1 . Exemplo

M = (a, b, c, d}.
As combinações dos 4 elementos, tomados dois a dois, são os subcon­
juntos:
(a, b] [b, c) (c, d)
[a, cj (b, d)
ja, dj

32. Notemos que [a, b\ = {b, a) pois, conforme definimos, combinação é


um conjunto, portanto não depende da ordem dos elementos.
É importante notar a diferença entre uma combinação (conjunto) e uma
seqüência, pois numa combinação não importa a ordem dos elementos, ao passo
que numa seqüência importa a ordem dos elementos.
A própria natureza do problema a ser resolvido nos dirá se os agrupa­
mentos a serem formados dependem ou não da ordem em que figuram os ele­
mentos.

33. Cálculo do número de combinações

Seja M = (ah a2, am) e indiquemos por Cmr ou (?) o número de


combinações dos m elementos tomados r a r .
Tomemos uma combinação, digamos esta: E, = [a„ a2, a3t ..., ar}. Se
permutarmos os elementos de E h obteremos r! arranjos.
Se tomarmos outra combinação, digamos E2 = (a2t a3> ..., a„ ar+,\, per-
mutando os elementos de E 2, obteremos outros r! arranjos.
Chamemos de x o número de combinações, isto é, x = Cmr e suponha­
mos formadas todas as combinações dos m elementos tomados r a r . São elas:
E „ E 2, E 3, . . . , E x.

Cada combinação E t dá origem a r! arranjos. Chamemos de Et o conjun­


to dos arranjos gerados pelos elementos de Et.

33
ANÁLISE COMBINATÓRIA

Temos então a seguinte correspondência:


E, — F,
E2 — F2

Ex — Fx.
Verifiquemos que:

(7 ) Fj H Fj = 0 para / ^ j

(íj) F, U F 2 U F 3 U ... U F x = F, em que F é o número de arranjos dos


m elementos de M tomados r a r.

Temos:
(T) Se Fj O Fj * 0 (para / ^ j ) , então existiría um arranjo que pertence­
ría a Fj e Fj simultaneamente.
Tomando os elementos desse arranjo obteríamos que coincidiría com F,
e Ej e, portanto, E-t = E j. Isto é absurdo, pois quando construímos todas as
combinações: Zs, 5* E} (para 1 ^ j ).
Logo, F, C\ Fj = 0 .

^j) Para provarmos que F, U F2 U ... U Fx = F, provemos que:


[F, U F2 U ... U F x C F e
(F C F, U F2 U ... U Fx.

a) Seja a um arranjo tal que


a E F, U F2 U ... U F x,
então a E F, (para algum / E [1, 2, ... *)) e, evidentemente, a E F;
logo:
F, U F2 U F3 U ... U Fx C F.

b) Seja agora a um arranjo tal que a E F. Se tomarmos os elementos


desse arranjo a, obteremos uma das combinações, digamos E,. Ora,
como E, gera o conjunto dos arranjos Fh então a E F) e, portanto,
a E F, U F2 U ... U Fj U ... U F x.
Então:
F C F, U F2 U ... U F x.

34
ANÁLISE COMBINATÓRIA

De (a) a (b) resulta que:


F, U F2 U ... U Fx = F.

Sabemos ainda que, se x conjuntos são disjuntos dois a dois, o número


de elementos da união deles é a soma do número de elementos de cada um.
Isto é,
#(F, U F 2 U ... U Fx) = #F #F, + #F, + + #F„ = #F
m! m!
r! + r! + ... r! = r!f =
(m - r)! (m - r)!
Logo:

m!
x
(m - r)! r!

Como x indica Cmr (ou (7)), temos a fórmula do número de combi­


nações:

V m, r E IN*
r < m

34. Casos particulares

I ? caso: my /* E IN* e r = m
I v-'m, m= 11
(C
-------—------- = 1
^ m!(m - m):

2? caso: m E IN* e r = 0
Cm.o = 1 (o único subconjunto com 0 elemento é o vazio)
------m!------ = j
0 !(m - 0)!

3? caso: m = 0 e r = 0
C0o = 1 (o único subconjunto do conjunto vazio é o próprio vazio)
Í — 51—

0 !(0 - 0)!
= 1

35
ANÁLISE COMBINATÓRIA

E m virtude da análise dos casos particulares, conclu ím os que a fórm ula

m!
Cm.r = (?) =
r !( m - r ) !

é válida V m , r G IN com r < m .

3 5 . Exemplos

1?) D eseja-se form ar um a com issão de três m em bros e dispõe-se de dez


fu n cio n ário s. Q uantas com issões podem ser form adas?
N otem os que cad a co m issão é um su b co n ju n to de três elem entos (pois
em cad a com issão não im porta a ordem dos elem entos). L o g o o núm ero de
com issões é:
10!
120.
3! 7!

2 ? ) T em os 7 cadeiras num eradas de 7 a 7 e d esejam os escolher 4 lugares


entre os existentes. D e qu antas form as isso pode ser feito ?
C ada escolha de 4 lugares correspond e a um a co m b in ação dos 7 elem en­
to s, tom ad os 4 a 4 , pois a ordem dos núm eros escolhidos n ão interessa (esco­
lher o s lugares 7, 2 , 4> 7 é o m esm o que escolher os lugares 7, 2 , 4 , 7 ). L o go ,
o resultado procurad o é:
7!
C 7.4 = 35
4 ! 3!

EXERCÍCIOS
124. Calcule os números:

125. Obtenha todas as combinações dos elementos de M = (7, 8, 9, 0], tomados dois
a dois.

36
ANÁLISE COMBINATÓRIA

1 2 6 . Um conjunto A possui n elementos, sendo n ^ 4. Determine o número de sub­


conjuntos de A com 4 elementos.

1 2 7 . O conjunto A tem 45 subconjuntos de 2 elementos. Qual é o número de elemfcn-


tos de A ?

Q
128. Sabendo que — *i £ ± L = 2, determine o valor de p.
Ca.p+i

1 2 9 . Calcule p , sabendo que Am p = Cmp v m c 0 ^ p < m .

1 3 0 . Calcule Am sabendo que Cm 3 = 84.

1 3 1 . Se | ^ j = 28, determine n.

1 3 2 . Determine x na equação Ax 3 — 6 • Cx 2 = 0.

1 3 3 . Determine n> sabendo que An+J 4 = 20 • Cn 2-

1 3 4 . Qual é o número m de objetos de uma coleção que satisfaz a igualdade


Am,3 ~ Cm. 3 = 25 Qn.m -/.

1 3 5 . Seja a, a ^ 6, a solução da equação An+2 7 = 10080 Cn+l 7. Então, sendo


f ( x ) = x 2 - 3x + /, calcule f ( a ) .

1 3 6 . Determine m, sabendo que A m 5 = 180 Cm 3.

1 3 9 . Prove que o produto de m fatores inteiros positivos e consecutivos é divisível por m l


Sugestão: Procure relacionar o produto dado com alguma fórmula conhecida.

1 4 0 . Uma prova consta de 15 questões, das quais o aluno deve resolver 10. De quantas
formas ele poderá escolher as 10 questões?

Solução
Notemos que a ordem em que o aluno escolher as 10 questões não interes­
sa. Por exemplo, resolver as questões:
7, 2, 5, 4, 5, 6, 7, 8, 9, 10 é o mesmo que resolver as questões: 10, 9, 8,
7, 6, 5, 4, 3, 2, 7.

37
ANÁLISE COMBINATÓRIA

Logo, cada maneira de escolher 10 questões é uma combinação das 15 ques­


tões, tomadas 10 a 10, isto é:
15!
3 003.
10! 5!

1 4 1 . De um baralho de 52 cartas, são extraídas 4 cartas sucessivamente e sem reposição.


Qual o número de resultados possíveis, se não levarmos em conta a ordem das
cartas extraídas?

1 4 2 . Em uma reunião social, cada pessoa cumprimentou todas as outras, havendo ao


todo 45 apertos de mão. Quantas pessoas havia na reunião?

1 4 3 . Quantos produtos podemos obter se tomarmos 3 fatores distintos escolhidos en­


tre 2, 3, 5, 7 e //?

1 4 4 . Um grupo tem 10 pessoas. Quantas comissões de no mínimo 4 pessoas podem


ser formadas, com as disponíveis?

1 4 5 . Um salão tem 10 portas. De quantas maneiras diferentes este salão poderá ser
aberto?

1 4 6 . Dez clubes de futebol disputaram um campeonato em dois turnos. No final, dois


clubes empataram na primeira colocação, havendo mais um jogo de desempate.
Quantos jogos foram disputados?

1 4 7 . De quantas formas podemos escolher 4 cartas de um baralho de 52 cartas, sem


levar em conta a ordem delas, de modo que em cada escolha haja pelo menos
um rei?

Solução

Como não levamos em conta a ordem das cartas, cada escolha é uma com­

binação. O número total de combinações é O número de combinações

em que não comparece o rei é Logo a diferença é o

número de combinações em que comparece ao menos um rei.

1 4 8 . O sr. Moreira, dirigindo-se ao trabalho, vai encontrando seus amigos e levando-


os juntos no seu carro. Ao todo, leva 5 amigos, dos quais apenas 3 são conheci­
dos entre si. Feitas as apresentações, os que não se conheciam, apertam-se as mãos
dois a dois. Qual é o total de apertos de mão?

38
ANÁLISE COMBINATÓRIA

1 4 9 . Existem 10 jogadores de futebol de salão, entre eles João , que por sinal é o único
que joga como goleiro. Nessas condições, quantos times de 5 pessoas podem ser
escalados?

1 5 0 . Um time de futebol de salão deve ser escalado a partir de um conjunto de 10 jo ­


gadores (entre eles Ari e Arnaldo). De quantas formas isso pode ser feito, se Ari
e Arnaldo devem necessariamente ser escalados?

1 5 1 . Um professor conta exatamente 3 piadas no seu curso anual. Ele tem por norma
nunca contar num ano as mesmas 3 piadas que ele contou em qualquer outro ano.
Qual é o número mínimo de piadas diferentes que ele pode contar em 35 anos?

1 5 2 . Uma equipe brasileira de automobilismo tem 4 pilotos de diferentes nacionalida­


des, sendo um único brasileiro. Ela dispõe de 4 carros, de cores distintas, dos
quais somente um foi fabricado no Brasil. Sabendo que obrigatoriamente ela de­
ve inscrever, em cada corrida, pelo menos um piloto ou carro brasileiros, qual
é o número de inscrições diferentes que ela pode fazer, para uma corrida onde
irá participar com 3 carros?

1 5 3 . Um químico possui 10 (dez) tipos de substâncias. De quantos modos possíveis


poderá associar 6 (seis) dessas substâncias se, entre as dez, duas somente não po­
dem ser juntadas porque produzem mistura explosiva?

1 5 4 . Um grupo consta de 20 pessoas, das quais 5 matemáticos. De quantas formas


podemos formar comissões de 10 pessoas de modo que:
a) nenhum membro seja matemático?
b) todos os matemáticos participem da comissão?
c) haja exatamente um matemático na comissão?
d) pelo menos um membro da comissão seja matemático?

1 5 5 . De um grupo de 10 pessoas deseja-se formar uma comissão com 5 membros. De


quantas formas isso pode ser feito, se duas pessoas (A e B) ou fazem parte da
comissão, ou não?

1 5 6 . Uma organização dispõe de 10 economistas e 6 administradores. Quantas comis­


sões de 6 pessoas podem ser formadas de modo que cada comissão tenha no mí­
nimo 3 administradores?

1 5 7 . Uma empresa tem 3 diretores e 5 gerentes. Quantas comissões de 5 pessoas po­


dem ser formadas, contendo no mínimo um diretor?

1 5 8 . Numa classe de 10 estudantes, um grupo de 4 será selecionado para uma excur­


são. De quantas maneiras o grupo poderá ser formado se dois dos dez são mari­
do e mulher e só irão juntos?

1 5 9 . Um homem possui 8 pares de meias (todos distintos). De quantas formas ele po­
de selecionar 2 meias, sem que elas sejam do mesmo par?

39
ANÁLISE COMBINATÓRIA

1 6 0 . Temos 10 homens e 10 mulheres. Quantas comissões de 5 pessoas podemos for­


mar se em cada uma deve haver 3 homens e 2 mulheres?

Solução

Podemos escolher 3 homens entre 10 de = 120 formas e podemos

escolher 2 mulheres entre 10 de = 45 formas.

Cada grupo de 3 homens pode se juntar com um dos 45 grupos de mulhe­


res, formando uma comissão. Como existem 120 grupos de homens, tere­
mos ao todo 120 • 45 = 5 400 comissões.

1 6 1 . Temos 5 homens e 6 mulheres. De quantas formas:


a) podemos formar uma comissão de 3 pessoas?
b) podemos formar uma comissão de 3 pessoas de modo que haja 2 homens e
uma mulher, na mesma?

1 6 2 . Um lote contém 50 peças boas e 10 defeituosas. Extraindo-se 8 peças (sem reposi­


ção), não levando em conta a ordem das mesmas, de quantas formas podemos
obter 4 peças boas e 4 defeituosas?

1 6 3 . Em uma urna existem 12 bolas, das quais 7 são pretas e 5 brancas. De quantos
modos podemos tirar 6 bolas da urna, das quais 2 são brancas?

1 6 4 . Quantos subconjuntos de 5 cartas contendo exatamente 3 ases podem ser forma­


dos de um baralho de 52 cartas?

1 6 5 . Uma urna contém 3 bolas vermelhas e 5 brancas. De quantas formas podemos


extrair 2 bolas, sem reposição e sem levar em conta a ordem na extração, de mo­
do que:
a) as duas sejam vermelhas?
b) as duas sejam brancas?
c) uma seja vermelha e a outra branca?

1 6 6 . Uma urna contém 10 bolas brancas e 6 pretas. De quantos modos é possível tirar
7 bolas, das quais pelo menos 4 sejam pretas?

1 6 7 . A diretoria de uma firma é constituída por 7 diretores brasileiros e 4 japoneses.


Quantas comissões de 3 brasileiros e 3 japoneses podem ser formadas?

1 6 8 . Deve ser formada uma comissão constituída de 3 estatísticos e 3 economistas, es­


colhidos entre 7 estatísticos e 6 economistas. De quantas maneiras diferentes po­
derão ser formadas essas comissões?

40
ANÁLISE COMBINATÓRIA

1 6 9 . Em um congresso há 30 professores de Matemática e 12 de Física. Quantas co­


missões poderiamos organizar compostas de 3 professores de Matemática e 2 de
Física?

1 7 0 . Quer-se criar uma comissão constituída de um presidente e mais 3 membros. Sa­


bendo que as escolhas devem ser feitas dentre um grupo de 8 pessoas, quantas
comissões diferentes podem ser formadas com essa estrutura?

171. Em um grupo de 15 pessoas existem 5 médicos, 7 engenheiros e 3 advogados. Quan­


tas comissões de 5 pessoas podemos formar, cada qual constituída de 2 médicos,
2 engenheiros e 1 advogado?

17 2 . Os ingleses têm o costume de dar alguns nomes para as crianças. Qual é o núme­
ro de maneiras diferentes de chamar uma criança, se existem 300 nomes diferen­
tes e se uma criança não pode ter mais do que 3 nomes, todos diferentes entre
si, e não se leva em conta sua ordem?

173. Em uma sala há 8 cadeiras e 4 pessoas. De quantos modos distintos essas pessoas
poderão ocupar as cadeiras?

174. Existem 7 voluntários para exercer 4 funções distintas. Qualquer um deles está
habilitado para exercer qualquer uma dessas funções. Portanto, podem ser esco­
lhidos quaisquer 4 dentre os 7 voluntários e atribuir a cada um deles uma das
4 funções. Quantas possibilidades existem para essa atribuição?

175. Existem 5 pontos, entre os quais não existem 3 colineares. Quantas retas eles de­
terminam?

176. Quantos planos são determinados por quatro pontos distintos e não coplanares?

177. Quantos triângulos são determinados por n pontos distintos do ponto, e não ali­
nhados 3 a 31

178. Há 12 pontos A, B y C, ... dados num plano a , sendo que 3 desses pontos nunca
pertencem a uma mesma reta. Qual é o número de triângulos que podemos for­
mar, utilizando os 12 pontos e tendo o ponto A como um dos vértices?

179. Num plano existem 20 pontos, dos quais 3 nunca são colineares, exceto 6 que
estão sobre uma mesma reta. Encontre o número de retas que esses pontos de­
terminam.

180. Numa circunferência são tomados 8 pontos distintos.


a) Ligando-se 2 desses pontos, quantas cordas podem ser traçadas?
b) Ligando-se 3 desses pontos, quantos triângulos podem ser formados?
c) Ligando-se 6 desses pontos, quantos hexágonos podem ser formados?

41
ANÁLISE COMB1NATÓRIA

181. Quantas diagonais tem um polígono regular de n lados?

Solução

O polígono tem n vértices: A h A2, ...» A„. Cada segmento é determinado


por um par não ordenado de dois vértices (A ,A2 = AyA,y por exemplo).

O número total de segmentos determinados será então Entre esses


segmentos estão incluídos os lados e as diagonais. Como existem n lados,
0 número de diagonais será:

/n \ n! n(n - 1) n2 - n - 2n
12 ) " n ~ (n - 2)! 2 n “ 2 2~ “
i

_ n2 — 3n _ n » (n — 3)
2 2

182 . Quantas diagonais, não das faces, tem:


a) um cubo? b) um octaedro?

183. Sabe-se que o número total de vértices de um dodecaedro regular é 20 e que as


faces são pentágonos. Quantas retas ligam dois vértices do dodecaedro, não per­
tencentes à mesma face?

184 . Quantas diagonais, não das faces, tem um prisma cuja base é um polígono de
n lados?

185 . No espaço existem 7 pontos, entre os quais não existem 4 pontos coplanares. Quan­
tos planos eles determinam?

186 . No espaço existem n pontos, entre os quais não existem 4 coplanares, com exce­
ção de 5 que estão num mesmo plano. Quantos planos os n pontos determinam?

187 . Num plano são dados 19 pontos, entre os quais não se encontram 3 alinhados,
nem 4 situados sobre uma mesma circunferência. Fora do plano, é dado mais
um ponto. Quantas superfícies esféricas existem, cada uma passando por 4 dos
20 pontos dados?

188. São dados 12 pontos em um plano, dos quais 5 e somente 5 estão alinhados. Quan­
tos triângulos distintos podem ser formados com vértices em 3 quaisquer dos 12
pontos?

42
ANÁLISE COMBINATÓRIA

Solução

Cada combinação de 3 pontos, entre os 12 existentes, dá origem a um triân­


gulo, com exceção das combinações de 3 pontos, tomados entre os 5 ali­
nhados; logo, o número de triângulos que podem ser formados é:

= 210 .

1 8 9 . São dadas 2 retas paralelas. Marcam-se 10 pontos distintos sobre uma e 8 pontos
distintos sobre a outra. Quantos triângulos podemos formar ligando 3 quaisquer
desses 18 pontos?

1 9 0 . Seja P o conjunto dos pontos de p retas ( p ^ 2), duas a duas paralelas, de um


plano. Seja Q o conjunto dos pontos de q(q ^ 2) retas, duas a duas paralelas
do mesmo plano, concorrentes com as p primeiras. Calcule o número total de
paralelogramos de vértices pertencentes ao conjunto P D Q e de lados contidos
no conjunto P U Q.

1 9 1 . Com as letras a , e, /, o, b, c, d, f , g, quantas palavras (com ou sem sentido) de


6 letras distintas podem ser formadas, usando-se 3 vogais e 3 consoantes?

1 9 2 . De quantas maneiras diferentes podemos colocar os 4 cavalos de um jogo de xa­


drez (2 brancos iguais e 2 pretos iguais) no tabuleiro do mesmo jogo (64 casas)?

1 9 3 . Obtenha o número de maneiras que nove 0*s e seis l*s podem ser colocados em
seqüência de modo que dois l*s não compareçam juntos.

Solução

- o - o - o —0 - 0 - 0 - 0 - ( M 0 -

Dispostos os nove zeros; existem 10 posições que os 1 *s podem ocupar (ver


figura acima).

Como existem 6 l ’s, precisamos escolher 6 lugares entre os 10 existentes.

194. De quantas formas podemos alinhar em seqüência p sinais “ + ” e q sinais


de modo que 2 sinais ” nunca fiquem juntos?
(Observação: E dado que p + 1 ^ q.)

43
ANÁLISE COMBINATÓRIA

1 9 5 . Considere as combinações de p elementos tomados m a m. Qual é a razão entre


o número de combinações em que figura um certo elemento e o número de com­
binações em que esse elemento não figura?

1 9 6 . Calcule o número de combinações de 8 elementos, 3 a 5, que contêm um determi­


nado elemento.

1 9 7 . Qual é o número de combinações de n elementos p a p que contêm k elementos


determinados?

IX. Permutações com elementos repetidos

36. C onsiderem os a palavra A N A e procurem os seus anagram as. V am os in­


d icar por A * o segundo A . T erem o s, en tão:
A N A *, A A * N , N A A * , N A * A , A * N A , A * A N
(D (2) (3) (4) (5) ( 6)

N otem os que as perm utações:

(1) e (5) são iguais


( 2 ) e ( 6) são iguais
(3) e (4) são iguais.

N a verdade, não tem os 3 ! = 6 perm utações distintas, m as apenas 3 , que


são:
AN A, AAN , N A A.

E ssa dim inuição do núm ero de perm utações decorreu do fa to de term os


duas letras iguais, A z A , n o co n ju n to das letras a serem perm utadas. É in tu iti­
vo perceber que o fato de existirem letras repetidas para serem perm utadas acar­
reta um a dim inuição do núm ero de perm utações, em relação ao núm ero que
teríam o s, se tod as fossem distintas.

3 7 . Vamos calcular o número de permutações que podemos formar quando


alguns elementos a serem permutados são iguais.

1? ca so

C onsiderem os n elem entos dos quais n, são iguais a a , e os restantes são


todos distintos entre si e distintos de a ,.

44
ANÁLISE COMBINATÓRIA

Indiquem os por P n‘ o núm ero de perm utações nessas cond ições e calcu ­
lem os esse núm ero.
C ada perm utação dos n elem entos é uma n -upla ordenada de elem entos
em que devem figurar n, elem entos iguais a a, e os restantes n - n, elem entos
distintos

n elementos

F açam o s o seguinte racio cín io . Das n posições que existem na perm uta­
çã o , vam os escolher n - n, p osições, para co lo ca r os elem entos tod os distin­
tos de a ,.

Existem _ m odos de escolher essas posições.

P ara cada escolha de (n — rij) posições, existem (n - n ,)! m odos em que


os (n - n ,) elem entos podem ser perm utados. L o g o , existem ao tod o

form as de disporm os os elem entos distintos de a ,, na perm utação.

U m a vez co lo cad o s esses elem entos d istintos, a posição dos elem entos re­
petidos a , fica determ inada (de uma só fo rm a) pelos lugares restantes.
tl ^
L o g o , existem — perm utações com n, elem entos iguais a a ,. Isto é:
rt//

E x em p lo

Q uantos anagram as existem da palavra P A R A G U A I?

r A , A , A — ► elem entos repetidos


P
T em os ) R
I G
U

n = 8 e n, = 5 , logo: P^ = - | ~ = 8 - 7 « 6 - 5 - 4 = 6 720.

45
ANÁLISE COMBINATÓRIA

2? ca so
C onsiderem os n elem entos dos quais n, são iguais a a ,: a ,, a ,, ...,
v V
n2 são iguais a a 2: n,

°2
e os restantes são tod os distintos entre si e distintos de a , e de a 2. Indiquem os
por P n,,n- o núm ero de perm utações, nessas cond ições.

C ada perm utação dos n elem entos é uma n -upla ordenada de elem entos
em que devem figurar n, elem entos iguais a a ,, n2 elem entos iguais a a 2 e os
n - n , - n2 elem entos restantes.

Façam os o seguinte racio cín io . D as n posições que existem na perm uta­


çã o , vam os escolher n - n2 lugares para co lo ca r todos os elem entos, com ex­

ceção dos iguais a a 2. E xistem ^ j m odos de escolher esses lugares. Para


cad a uma dessas escolhas, existirão P nLn m odos em que os n - n2 elem entos
podem ser perm utados (lem brem os que, dos elementos a serem perm utados ago­
ra, existem n, iguais a a ,) . A o tod o existirão

/ n \ p „( n! (n ~ n2)l = n!
\n-nj n' n^ ( n - n 2) ! n 2! ‘ n ,! n ,! n2!

form as de a rra n ja r na perm utação tod os os elem entos, com exceção de a 2.

U m a vez arran jad o s esses elem entos na perm utação, as posições dos ele­
m entos repetidos a 2 ficam determ inadas (de uma única form a) pelos lugares
. . ^ nt
restantes. L o g o , existirão — f — perm utações com n , elem entos iguais a a,
n i- n2-
e n2 elem entos iguais a a 2. Isto é:

P ni•
n
n2 __ n!

38 . Exemplos

1?) Q uantos anagram as existem da palavra A N A L ÍT IC A ?

46
ANÁLISE COMBINATÓRIA

r A, A, A elem entos repetidos


I, I elem entos repetidos
N
Tem os <
L
T
C
L

n = 9, n, = 3, n2

L ogo:

p^ 2 = = 30240.

2 ?) Existem 6 bandeiras (de m esm o fo rm ato ), sendo 3 vermelhas e 3 bran­


cas. D ispondo-as ordenadam ente num m astro, quantos sinais diferentes podem
ser em itidos com elas?
T em os:
C ada sinal em itido co n sta de uma perm utação de 6 band eiras, sendo 3
iguais a V (verm elhas) e 3 iguais a B (bran cas).
Isto é, n = 6, rij - 3 , n2 = 3 .

P o rta n to , existem :
r»3.3 6! '
P6 = - jT 3 7 = 2 0 sinais-

39. Caso geral

C onsiderem os n elem entos, dos quais:


n, são iguais a a ,
n2 são iguais a a2

nr são iguais a ar
U sando racio cín io an álog o ao do 1? edo 2? ca so , poderem os calcu lar
o núm ero de perm utações nessas cond ições (indicado por P nr n?..... n ) através
da fórm ula:

47
ANÁLISE COMBINATÓRIA

É fácil ver que no caso particular de n, = n2 nr 1 obterem os:


pi.
-*■ n 1 = n!
que é o núm ero de perm utações de n elem entos distintos.

EXERCÍCIOS
1 9 8 . De quantas formas 8 sinais “ + ” e 4 sinais podem ser colocados em uma
seqüência?

1 9 9 . Quantos números de 6 algarismos podemos formar permutando os algarismos


2, 2, 3, 3, 3, 51

2 0 0 . Sobre uma mesa são colocadas em linha 6 moedas. Quantos são os modos possí­
veis de colocar 2 caras e 4 coroas voltadas para cima?

2 0 1 . Quantos anagramas existem da palavra AM ARILIS?

2 0 2 . Se uma pessoa gasta exatamente um minuto para escrever cada anagrama da pa­
lavra ESTA TÍSTIC A , quanto tempo levará para escrever todos, se não deve pa­
rar nenhum instante para descansar?

2 0 3 . Uma moeda é lançada 20 vezes. Quantas seqüências de caras e coroas existem,


com 10 caras e 10 coroas?

2 0 4 . Quantos números de 7 algarismos existem nos quais comparecem uma só vez os


algarismos 5, 4t 5 e quatro vezes o algarismo 91

2 0 5 . Uma urna contém 3 bolas vermelhas e 2 amarelas. Elas são extraídas uma a uma
sem reposição. Quantas seqüências de cores podemos observar?

2 0 6 . Um homem encontra-se na origem de um sistema cartesiano ortogonal, como mos­


tra a figura. Ele só pode dar um passo de cada vez, para norte (TV) ou para leste
( L ). Quantas trajetórias (caminhos) existem da origem ao ponto P(7, J ) ?

Solução

Notemos inicialmente que o homem terá que dar, ao todo, 7 + 5 = 12 pas­


sos (7 para leste e 5 para norte).
Cada trajetória possível é, então, uma seqüência de 12 elementos, sendo
7 L*s e 5 N's.

48
ANÁLISE COMBINATÓRIA

A trajetória da figura é:

(L, N, N, L , L, L , N, L, N, N, L, L).

Se quisermos o número de trajetórias,


teremos que calcular então o número
de permutações com repetição de 12
elementos, sendo 7 L ’s e 5 N 9s. Por­
tanto, o número de trajetórias é:

_ 12!
p 7, 5
p '2 ' 7! 5!

2 0 7 . Uma cidade é formada por 16 quarteirões


dispostos segundo a figura ao lado. Uma
pessoa sai do ponto P e dirige-se para o
ponto Q pelo caminho mais curto, isto
é, movendo-se da esquerda para direita,
ou de baixo para cima. Nessas condições,
quantos caminhos diferentes ele poderá
fazer?

2 0 8 . Um homem encontra-se na origem de um sistema cartesiano ortogonal. Ele só


pode dar um passo de cada vez, para norte ( N ) ou para leste ( L ) . Partindo da
origem e passando pelo ponto A(3, /), quantas trajetórias existem até o ponto
B(5t 4)1
r *v

2 0 9 . Com os dígitos /, 2, 3 , 4, 5, 6, 7, de quantas formas podemos permutá-los de


modo que os números ímpares fiquem sempre em ordem crescente?

2 1 0 . Uma classe tem a meninas e b meninos. De quantas formas eles podem ficar em
fila, se as meninas devem ficar em ordem crescente de peso, e os meninos tam­
bém? (Suponha que 2 pessoas quaisquer não tenham o mesmo peso.)

X. Complementos
Partições ordenadas

40. Considerem os um co n ju n to A e K su bco n ju n to s de A n ão vazios A „ A 2,


A Ki tais que:

a) Aj H A j = 0 (para i 5* j )
b) A , U A 2 U . . . U A K = A

49
ANÁLISE COMBINATÓR1A

C ham arem os de uma p artição ordenada do co n ju n to A à seqüência de


co n ju n to s.

(A ,, A 2, A k).

41- Exemplos

S e ja A = [1, 2, 3, 4, 5, 6] e considerem os as seqüências de co n ju n to s

1) ( ( 1 ,2 ) ; (3, 4); (5, 6))


2) ((1, 2 , 3, 4); (5); (6))
3) ( 0 ; (1, 2, 3, 4); [5, 6))
4) ((1, 2, 3); (3, 4 , 5); (6))
5) ([1, 2); (3, 4 , 5)).

Nos exem plos (1) e (2) tem os p artições ordenadas de A , ao passo que em
(3) não tem os, pois 0 faz parte da seqüência. Em (4) não tem os uma partição
ord enad a, pois {/, 2, 5 ) f l (5, 4, 5) ^ 0 e finalm ente, em (5 ), (/, 2} U
(5, 4, 5 ) * A .
O bservem os que a p artição ord enad a ([/, 2 ) ; [3, 4}\ (5, 6 )) é diferente
da p artição ordenada ((5, 6 ); (5, 4 }; (/, 2 )), pois cad a p artição , sendo uma se­
qüência de co n ju n to s, d e p e n d e d a o r d e m d o s m esm o s .

42. Podem os resolver alguns problem as co m b in ató rios com auxílio do co n ­


ceito de partição ordenada.

E x em p lo
D e quantas m aneiras podem os co lo car 10 pessoas em três salas, A , B e
C , de m odo que em A fiquem 4 pessoas, em B fiquem 3 pessoas e em C tam ­
bém 3 pessoas?
N otem os que cad a m odo de d istribuir as 10 pessoas corresponde a uma
p artição ordenada das m esmas do tip o:

(í » ” ]> ( » )» í » »| ))
pessoas na pessoas na pessoas na
sala A sala B sala C

P ara calcu larm os o núm ero de partições ordenadas, façam o s o seguinte


racio cín io:
E scolhem os 4 entre 10 pessoas para ficarem em A . Isto pode ser feito de

50
ANÁLISE COMBINATÓRIA

E m seguida, entre as 6 pessoas restantes, escolhem os 5 , para ficarem em


B. Isto pode ser feito de m aneiras.

As 3 pessoas restantes podem ser escolhidas de J = 1 m an eira (isto é,


as pessoas da sala C ficam determ inadas).

O ra, cad a co m b in ação de pessoas em A gera í ^ j m aneiras de dispor 3


pessoas em B .
L o g o , o núm ero to tal de p artições é:

(10 \ . /6\ 10! 6! 10!


\ 4 / \3/ 6! 4! 3! 3! 4! 3! 3!
Isto é, existem 4 2 0 0 m odos de disporm os as 10 pessoas nas 3 salas.

Partições não ordenadas

43. C onsiderem os um co n ju n to A e K su bco n ju n to s de A não vazios


A „ A 2t . .. , A k , tais que:
a) Aj f l Aj = 0 (p ara i * j)
b) A , U A 2 U . .. U A k = A .

C ham arem os de um a p artição não ordenada de A à fam ília:

44. Exemplo

S e ja o co n ju n to : A = (/, 2, 5 , \4, 5, 6\ e considerem os as fam ílias:


1) [(1, 2, 3), [4, 5, 6}j é um a p artição ,
2) ((1, 2); (3, 4, 5, 6j) é um a p artição ,
3) ((1, 2); (3j) não é um a partição ,
4) ((1, 2, 3, 4); [3, 4, 5, 6}) não é uma p artição .

45. Alguns problem as com binatórios podem ser resolvidos com este co n ceito .

E x em p lo
De qu antos m odos 12 pessoas podem ser repartidas em 3 g rupos, tendo
cada grupo 4 pessoas?
C onsiderem os, para fixar idéias, 3 grupos, A ,, A 2 e A 3.

51
ANÁLISE COMBINATÓRIA

N otem os que a ordem em que figuram pode ser qualquer um a que tere­
m os a m esm a d istribu ição em 3 grupos. E stam o s en tão interessados no núm e­
ro de partições não ordenadas do tipo:

íí > > ]> 1 * * ]* í > * 11


grupo A , grupo A 2 grupo A }

P a ra calcu larm os o núm ero de p artições não ordenadas, façam os o se­


guinte raciocín io:

1?) C alculem os o núm ero de p artições ordenadas

({-, -); K -); K - -))


qu e, com o racio cín io do exem plo an terio r, sabem os ser:

42)-(5)•U) -
2 ? ) C ada grupo de 3 ! = 6 partições ordenadas dá origem à m esm a parti­
ção não ordenada.
3 ?) L o g o , o núm ero de p artições não ordenadas será:

34 650 _ 5775
6

Soluções inteiras não negativas de uma equação linear

46. C onsiderem os a equação linear x + y = 7 e encontrem os o núm ero de


soluções inteiras não negativas da m esm a.
P o r tentativas, en co n tram o s:

(0, 7); (1 , 6); (2, 5); (3, 4 ); (4, 3); (5, 2 ); ( 6 , 1); (7, 0).

A o tod o tem os 8 soluções inteiras não negativas.

4 7 . A gora, se tiverm os a eq u ação x + y + z = 7, resolvendo por ten tati­


vas, o trabalh o será m uito grande, e correm os o risco de “ esquecer” alguma
solu ção.
U m racio cín io alternativo seria o seguinte:
T em os que dividir 7 unidades em 3 partes ordenadas, de m odo que fique
em cad a parte um núm ero m aior ou igual a zero.
Indiquem os cad a unidade por um p on to. E n tã o , elas serão representadas
por:

52
ANÁLISE COMBINATÓRIA

• • • • • • •

C om o querem os dividir as 7 unidades em 3 partes, vam os usar duas b a r­


ras para fazer a sep aração.
C ad a m odo de disporm os os pontos e as barras dará origem a um a solu ­
çã o . P o r exem plo:

3 2

O ra, com o tem os 9 s ím b o lo s J e


l2 *
o núm ero de perm utações desses sím bolos será:

7,2 9!
P9 36
7! 2!

que é o núm ero de soluções inteiras não negativas da equação x + y + z = 7.


T al racio cín io pode ser generalizado pelo:

48. Teorema

O núm ero de soluções inteiras não negativas da eq u ação x, + x2 +


+ ... + xn = r é:

(n + r — 1 )!
r! (n - 1 )!

53
ANÁLISE COMBINATÓRIA

D em o n stra çã o
D e fa to , cad a solu ção da eq u ação é uma perm utação de r sím bolos • e
{n ~~ /) sím bolos I (precisam os de (n - 1) barras para dividir r pontos em n
partes).

O núm ero de perm utações (soluções da equação) será:

p (n -l), r
(n + r - 1 )!
r n + r- 1
r! (n - 1 )!

49. Exemplo de aplicação


Um bar vende 3 tipos de refrigerantes: g u aran á , s o d a e tôn ica. D e qu an ­
tas form as uma pessoa pode com p rar 5 g arrafas de refrigerantes?
S e ja :
x o núm ero de g arrafas de guaraná
y o núm ero de g arrafas de soda
z o núm ero de garrafas de tô n ica

É claro que x, y, z E IN e x +y + z = 5.
T rata-se então de ach ar o núm ero de soluções inteiras não negativas da
equação
x + y + z = 5

(5 + 3 — D !
que é, en tão :
5! (3 - 1)!

EXERCÍCIOS
211 . Um grupo de 10 viajantes pára para dormir num hotel. Só havia 2 quartos com
5 lugares cada um. De quantas formas eles puderam se distribuir para dormir
naquela noite?

54
ANÁLISE COMBINATÓRIA

2 1 2 . De quantos modos 8 pessoas podem ocupar duas salas distintas, devendo cada
sala conter pelo menos 3 pessoas?

2 1 3 . Dez alunos devem ser distribuídos em 2 classes, de 7 e 3 lugares respectivamente.


De quantas maneiras distintas pode ser feita essa distribuição?

2 1 4 . Separam-se os números inteiros de 1 a 10 em dois conjuntos de 5 elementos, de


modo que 1 e 8 não estejam no mesmo conjunto. Isso pode ser feito de n modos
distintos. Qual é o valor de n ?

2 1 5 . Dentre 6 números positivos e 6 números negativos, de quantos modos podemos


escolher quatro números cujo produto seja positivo?

2 1 6 . De quantas formas 12 estudantes podem ser divididos e colocados em 3 salas,


sendo 4 na primeira, 5 na segunda e 3 na terceira?

2 1 7 . De quantas maneiras podemos atribuir os nomes de Paulo, Antônio e José a 11


meninos, com a condição de que 3 deles se chamem Paulo, 2 Antônio e 6 José?

2 1 8 . Um baralho tem 52 cartas. De quantos modos podemos distribuí-las entre 4 joga­


dores, de modo que cada um receba 13 cartas?

2 1 9 . De quantas formas 20 alunos podem ser colocados em 4 classes, A, B, C, D, fi­


cando 5 alunos por classe?

2 2 0 . De quantas formas podemos distribuir 10 bolinhas, numeradas de 1 a 10, em 2


urnas, A c B (podendo eventualmente uma ficar vazia)?

2 2 1 . De quantas formas podemos repartir 9 pessoas em 3 grupos, ficando 3 pessoas


em cada grupo?

2 2 2 . Com 10 pessoas, de quantas formas podemos formar dois times de bola ao cesto?
\
2 2 3 . De quantas formas 15 pessoas podem ser divididas em 3 times, com 5 pessoas
por time?

2 2 4 . Quantas soluções inteirasnão negativas têm as equações:


a) x + y +z = 6
b) x + y +z + t= 10
c) x + y +z + t + w = 10

2 2 5 . Quantas soluções inteiras tem a equação: x, + x2 + x3 + x4 + x5 = 20 , se ca­


da Xj é tal que xt ^ 3 V i E (7, 2, 3, 4, 5 ) ?

2 2 6 . Uma pastelaria vende pastéis de carne, queijo e palmito. De quantas formas uma
pessoa pode comer 5 pastéis?

55
ANÁI isi combinatôria

2 2 7 . IJmn mercearia tem em seu estoque pacotes de café de 6 marcas diferentes. Uma
pessoa deseja comprar 8 pacotes de café. De quantas formas pode fazê-lo?

2 2 8 . Uma confeitaria vende 5 tipos de doces. Uma pessoa deseja comprar 3 doces.
De quantas formas isso pode ser feito?

2 2 9 . Temos duas urnas, A e B. De quantas formas podemos colocar 5 bolas indistin­


guíveis, podendo eventualmente uma das urnas ficar vazia?

LEITURA

Cardano: o Intelectual Jogador


H ygino H . D om ingues

O perfil b iog ráfico que dele traçaram vários historiadores o c o ­


locaria folgadam ente na galeria dos célebres crápulas da história. C on ­
tudo há uma tendência h o je a se consid erar que tais historiad ores fo ­
ram dem asiado severos para com ele ou pelo m enos que n ão levaram
na devida conta todo o co n ju n to de circunstâncias (e in fo rtú n io s) de
sua vida. M as num ponto há unanim idade: G iro lam o C ardano
(1501-1576) merece figurar, por vários m otivos, na com panhia dos gran­
des m atem áticos de todos os tem pos.
O próprio ato de nascim ento
de C ard ano , em P ávia (Itá lia ), po­
de ter sido seu prim eiro in fo rtú n io .
Segundo parece, seus pais, que não
eram casad os, fizeram de tudo pa­
ra que ele não nascesse. P elo menos
para o bem da m atem ática, não fo ­
ram felizes nesse intento.
O pai de G iro lam o era um in­
telectual de certa p ro jeçã o que se
dedicava à m edicina, à ad vocacia,
à m atem ática e ... às ciências o cu l­
tas. Ó filho tam bém enveredou pe­
la m edicina (depois de Versalius foi
o m édico mais renom ado da E u ro ­
pa em seu tem po) e pela m atem áti­
ca . M as não ficou só n isso ...

56
U m dos “ p ecad o s” atribu íd os a C ard ano foi o vício do jo g o . D e
fa to , em sua a u to b io g rafia, D e p r ó p r ia vita, ele co n fessa ter jo g a d o
diariam ente: xadrez por m ais de qu arenta anos e dados por m ais de r
vinte e cin co . D eve-se levar em co n ta porém que n o século X V I o jo g o
era o passatem po d om inante. E , co m o se jo g a v a a d inheiro, iniciou-se
nessa atividade aind a estudante universitário para prover sua m anu­
ten ção .
O utro “ estigm a” de C ardano foi sua condição de astrólogo. O co r­
re porém que naquele tem po a astrolo gia ocupava uma posição m uito
d iferente no p anoram a cu ltural. H a ja vista que m uitos governantes
m antinham astrólogos em suas cortes e que m uitos p rofessores univer­
sitários faziam predições baseadas na astrologia. M ais ain d a, era co n ­
siderado norm al que m atem áticos e astrônom os se dedicassem a essa
pseudociência. O próprio Jo h a n n Kepler (1571 -1 6 3 1 ) às vezes reco rria
a ela para com p lem entar seus ganhos.
A o b ra m atem ática m ais conhecida de C ard ano é a A rs m ag n a
( A a rte m a io r ), onde aparecem im pressas pela prim eira vez as solu ­
ções gerais das equações cú bica e q u ártica. M as ironicam ente, co m o
quase tudo em sua vida, um pequeno m anual do jo g a d o r, intitulad o
L ib e r d e lu d o a le a e ( O livro d o s j o g o s d e a z a r), que ele sequer co n si­
derava digno de p u blicação, pode ter sido sua co n trib u ição m aior à
m atem ática.
Na parte técnica do livro, C ard ano discutiu eqüiprobabilid ad e,
esperança (o m ontante co rreto da ap osta a ser feita por um jo g a d o r
que tem p robabilidad e p de ganhar a im p ortân cia s ), estabeleceu a lei
p n = P n* Que dá a probabilidad e de que um evento de p robabilidad e
p o co rra independentem ente n sucessivas vezes, achou tábuas de pro­
babilidades para dados e usou a cham ad a lei dos grandes núm eros (de
m odo rudim entar) — questões em que fo i o pioneiro.
É verdade tam bém que C ard ano ensinava no livro a trap acear
no jo g o . M as o que im porta isso em face do vanguardismo de sua o b ra?

57
CAPÍTULO II

Binômio de Newton
I. Introdução
50. V am os usar as técnicas que estudam os em A nálise C om b in ató ria para ter
um resultado im portante em Á lgebra, que consiste em obter o desenvolvim en­
to do binôm io ( jc + a ) n para n E IN e jc, a E IR.
J á nos são fam iliares os casos particulares:
(x + a)° = 1
(x + a )1 = x + a
(x + a )2 = x 2 + 2 xa + a 2
(x + a )3 = x 3 + 3x2a + 3xa 2 + a3.

P ara tod o n inteiro, positivo, podem os calcu lar:

(x + a)n = [(x + a) - (x + a) * ... * (x + a) j

n fatores

usando a propriedade distributiva da m u ltiplicação.

51. O procedim ento é o seguinte:

1?) D e cad a fato r ( x + a ) selecionam os exatam ente um term o , que po­


derá ser x ou a, m ultiplicando-os em seguida.
2 ? ) C ontinu am os o processo até esgotar todas as seleções possíveis de
um term o de cad a fa to r.
3?) T om am os todos os produtos obtidos e calculam os sua som a (que con ­
siste em reduzir os term os sem elhantes).
4?) E ssa som a obtid a é o resultado do desenvolvim ento de (x + a ) n.

58
BINOMIO DF. NF.WTON

52 . Exemplo 1

(x 4- a )2 = (x + a) • (x + a)

U sam os o diagram a da árvore para as seleções dos term os.

1? fa to r 2? fa to r Produto

S o m a: x -x + x - a + a - x + a - a = x 2 + 2ax 4- a2.
P o rta n to , (x 4- a )2 = x 2 4- 2ax 4- a 2.

53. Exemplo 2

(x 4- a )1 = (x + a) • (x 4- a) • (x 4- a).

1? fa to r 2? fa to r 3? fato r P rodu to

x •x •x

x •x •a

x •a •x

x •a •a

a •x •x

a •x •a

a •a •x

a •a •a

So m a: x * x * x 4- x - x - a 4- x * a - x 4- x - a - a 4- a - x * x 4- a - x * a 4-
4 - a * a - x 4 - a - a * a = x 3 4- 3x 2a 4- 3xa 2 4- a 3.

P o rta n to , (x 4- a )3 = x 3 4- 3x2a 4- 3ax 2 4- a 3.

59
BINÔMIO DE NEWTON

54. O problem a que surge é o seguinte. Será que podem os o b te r os term os


do desenvolvim ento de ( x + a ) n sem recorrer ao diagram a da árvore?
A resposta é positiva. V am os m ostrar co m o isso é possível por meio de
um exem plo p articular, e em seguida vam os generalizar o resultado obtid o.

E x em p lo
(x + a)3 = (x + a) •(x + a) •(x + a).
Se escolherm os um term o de cada fa to r, obterem os três term os, que de­
vem ser m ultiplicados entre si.
O s tipos de produtos que podem os ob ter são: x 3; x 2 • a\ x • a 2; a 3.
A gora vejam os quantos aparecem de cada tipo.

1?)
Só existe uma m aneira de obter o produto x 3 = x • x • jc, que é escolhen­
do som ente o term o “ j c ” de cada fato r. L ogo, o coeficiente de x 3 no desenvol­
vim ento do binôm io é / ou l^ j.

2 ?) x 2 • a
A quantidade de produtos do tipo x 2 • a é igual ao núm ero de seqüên-
cias de três letras em que duas são iguais a “ x ” e uma é igual a “ a ” . Isto é:

L o g o , o co eficien te de x 2 • a é \

3 ?) * • a 2
A quantidade de produtos do tipo x • a 2 è igual ao núm ero de seqüên-
cias de três letras em que um a é igual a “ x ” e duas são iguais a “ a ” . Isto é:
1.2 3!
P3
1! 2 !

L o go , o co eficien te de x • a 2 é (^ j.

4?) a 3
Só existe uma m aneira de obter o produto a 3 = a a a , que é escolhen­
do som ente o term o “ a ” de cad a fato r. L o go , o co eficien te de a 3 no desen­
volvim ento do binôm io é:

1 ou

60
BINÔMIO DE NEWTON

Em resum o:

(x + a )3 x2 •a + x • a2 + a3

II. Teorema binomial

O desenvolvim ento de ( x + a )" para n G IN e x, a & IR é dado por:

(x + a)" = ( q | • x" + ( " ) x "-1 • a' + ( 2 ) ’ x"-2 ' 32 + — + ••• +

+ (p| • x "- p • ap + ... + ( " ) • a"

D em o n stra çã o
(x + a )n = i(x + a) • (x + a) ■ . .. • (x + a) 1
V------------------------------------------- y ------------------------------------------ J

Pela propriedade distributiva da m u ltiplicação e tendo em vista os exem ­


plos precedentes, conclu ím os que os d iferentes tipos de term os que podem ser
obtidos na m ultiplicação são:
xn; x " -1 • a; x n- 2 • a2; ...; x n- p • a p; ...; a n.

V ejam os agora a quantidade de cada um desses diferentes tipos de term os.

1?)
O produto x n só pode oco rrer de Uma form a: pr•x •*•...• xj e, portanto,
V

n fatores

o co eficien te de x n é I ou j^ j.

2 ?) x n~l • a
O produto x n~l • a pode ocorrer de tantas form as quantas pudermos per-
inutar (n - 1) letras “ x ” e uma letra “ tf” . Isto é:

p n -u = n! = /n\
n (n - 1 )! 1 ! \l/“

P o rta n to , o co eficien te de x n~l • a é í nt .

61
BINÔMIO DE NEWTON

3 ?) x n~2 • a 2
O produto x"~2 • a 2 pode o co rrer de tan tas form as qu an tas pudermos
perm utar ( n - 2 ) letras “ x ” e duas letras “ tf” . Isto é:

p n -2 .2 = n! = /n\
" (n - 2 )! 2 ! \2 /'

P o rta n to , o co eficien te de x n~2 a2é

4 ? ) x n~p • a p
G enericam ente, o produto x n~p • a p pode ocorrer de tan tas form as quan­
tas puderm os perm utar ( n - p ) letras “ x ” e p letras “ tf” . Isto é:

pn-p, p = ------- 1±1------


(n - p)! p!

P o rta n to , o co eficien te de x n~p • a p é

5?) a n
F inalm ente, o prod uto a n só pode o co rrer de um a fo rm a , que é:
an = |a • a • a • • aj

n fatores

P o rta n to , o co eficien te de a n é / ou

D as considerações feitas acim a, conclu ím os que:

(x + a )n = •xn + • x n- ‘ • a 1 + . .. + • xn- p • a p + ... +

que é o que queríam os d em onstrar.

55. Exemplo
D esenvolver (3 x 2 + a ) 4.
T em os:

i4 =
(3x 2 + a )4 (3x 2)4 + m • (3x 2)3
0 \ 2)

+ Ç J ■ (3x2) • a 3 + ( J j • a4.

(3x 2 + a )4 = 81x8 + 108x6a + 54x 4a 2 + 12x 2a 3 + a4.

62
BINÔMIO DE NEWTON

EXERCÍCIOS
2 3 0 . Desenvolva, usando o teorema binomial:
a) (x + 3b )3 b) (1 - x 2)5 c) (Jx - Jy )4 d) (sen 0 + cos d)4 e) (3 - y)5

2 3 1 . Desenvolva, usando o teorema binomial


/ i v1 / 1 V
m + — - — m — —-
\ m/ \ ml
2 3 2 . Desenvolva ( jc + a ) 7.

2 3 3 . Calcule a e b y sabendo que (a + b )3 = 64 e que

a5 - ( J ) a 4 •b + • a 3 •b2 - • a 2 ■ b 3 + ( J ) ab< - b5 = -3 2 .

2 3 4 . Quantos termos tem o desenvolvimento de:


a) (x + y)7? b) (x + y)10? c) (x + y)n?

2 3 5 . a) Quantos termos tem o desenvolvimento de ( jc + a )50?


b) Escreva os 4 primeiros termos, sem os coeficientes, em potências de expoentes
decrescentes de x.

2 3 6 . No desenvolvimento de ( jc + y ) 1000, qual o centésimo termo, se o desenvolvimen­


to for feito em potências de expoentes decrescentes de jc ?

2 3 7 . Quais os 3 primeiros termos do desenvolvimento de ( jc + y ) ,0° segundo as po­


tências de expoentes decrescentes de x l

III. Observações

5 6 . Os números:

são cham ados co eficien tes bin om iais. No coeficiente binom ial l n \f n é cham ado
num erador e p , d enom inador.

63
BINÔMIO DE NEWTON

5 7 . O teorema binomial é válido para (x — a)n, pois basta escrevermos


( x - a)n como [x + (-a )]rt e aplicarmos o teorema.

E x em p lo
(x - 2 y)4 = [x + (—2 y )]4 =

= ( jj) x 4 + ( * j x 3( - 2 y ) ‘ + ( 2 ) x 2(—2y )2 + ( 3 ) X' •(—2y )3 + ( J ) (~ 2 y )4 =

= x 4 — 8x3y + 24x 2y2 — 32xy 3 + lóy 4.

EXERCÍCIOS
2 3 8 . Sabendo que:

a 5 + |j j a4b + a 3b 2 + ( 3 ) a 2b 3 + ( 4 ) a^4 + b 5 = 1 024

calcule o valor de (a + b )2.

2 3 9 . Determine o valor da expressão:


995 + 5(99)4 + 10(99)3 + 10(99)2 + 5(99) + 1

2 4 0 . Calcule o valor numérico do polinômio:


1 + J6 yfó- l
x4 - 4x3y + óx^ 2 - 4xy 3 + y4 para x = e
fs

2 4 1 . Calcule o valor de S = 2 + j 22 + ..

2 4 2 . Calcule o valor da expressão ( I - J s f - ( I + J5 )5.

2 4 3 . Calcule o valor numérico da expressão:

xn + x" -1 y + ^2 ) x""2 y 2 + ••• + yn> Para X = y = 1.

2 4 4 . Calcule o valor de S, sabendo que:


S = (x3 - l )4 + 4(x 3 - l )3 + 6(x 3 - l )2 + 4(x 3 - 1) + 1

2 4 5 . Qual é o valor de ( 2 )x (3 )n~x?

64
BINÔMIO DE NEWTON

58. Term o geral

J á vim os que:

x n_I • a + ... + xn-p . a p + ... +

O term o:

é cham ad o geral, pois fazendo p = 0, 1, 2, n obtem os tod os os term os do


desenvolvim ento.
N otem os aind a qu e, V p , a som a dos expoentes de x e a é sem pre n . Além
disso, o expoente de x é igual à d iferen ça entre o num erador e o d enom inador
do co eficien te binom ial correspond ente.

E x em p lo s
1?) N o desenvolvim ento de ( x 2 + 7 )6, qual o co eficien te de x 8l
T em os:

O term o geral do desenvolvim ento é: (x2)6- p • l p = x ,2_2p.

C om o querem os o term o que possua x 8, devemos im por que 12—2 p = 8 ,


isto é, p = 2 .
L o g o , o term o que possui x 8 é:

P a ra que ele independa de x> devem os ter 8 - 2 p = 0, isto é, p = 4.


L o g o , o term o procurad o é:

65
BINÔMIO DE NEWTON

3?) Desenvolvendo ( x + y)'° em potências de expoentes decrescentes de


x , qual é o 6 ? term o?
N otem os que:
o 1 ? term o co n terá x'°
o 2 ? term o co n terá x 9

o 6 ? term o co n terá x5.

P o rtan to , o term o procurad o é:

252 x 5y5.

Um o u tro m od o de en contrarm os o term o d esejado seria n o tar que, de­


senvolvendo o binôm io em potências de expoentes decrescentes de x , os co e fi­
cientes seriam :

1? term o 2? term o 3? term o (p 4- 1) term o

E , com o querem os o 6 ? term o , devemos tom ar o co eficien te binom ial

que no nosso caso é P o rta n to , o term o d esejado é x 5y 5 =


= 252 x 5y 5.

EXERCÍCIOS
246. Qual o coeficiente de x 2 no desenvolvimento de ( / - 2x)6t}.
247. Desenvolvendo (x + 3y)9y qual o termo que contém x 4l
248. No desenvolvimento de ( / - 2x2)5y qual o coeficiente de x 8?
249. Qual o coeficiente de x 6 no desenvolvimento de (x 2 + x '3)8!

2 5 0 . Qual o termo em x 3 no desenvolvimento de

2 5 1 . Qual o termo em x 3 no desenvolvimento de |Jx -

252. Determine o coeficiente numérico do termo de 4? grau do desenvolvimento do


binômio de Newton ( x - 2 )7.

66
BINÔMIO DE NEWTON

2 5 3 . Qual é o coeficiente do termo que contém o fator y 4 no desenvolvimento bino-


/ 10
mial de ^ — x 2 - yj ?

2 5 4 . Qual é o coeficiente numérico do termo de grau l em x, no desenvolvimento


de [x + - ^ ] ?

2 5 5 . Determine o coeficiente de x 5 no desenvolvimento binomial de - -y x j.

2 5 6 . Obtenha o coeficiente do termo em x 3 no desenvolvimento de fxx +

' 2y ? 12
2 5 7 . Qual é o coeficiente do termo em x 2 de [— -----
3 2x)
2 5 8 . No desenvolvimento de (x + a )/0°, qual o coeficiente do termo que contém x 60?

2 5 9 . Qual é o coeficiente do termo médio de (x J + y 2)/0?

2 6 0 . Qual o termo independente de y no desenvolvimento de + -y j ?

2 6 1 . Qual o termo independente de x no desenvolvimento de |x + j ?

2 6 2 . Qual o termo independente de x no desenvolvimento de í - j t + — ?

2 6 3 . Calcule o termo independente de x no desenvolvimento de [ y y - .


\\ y X
2 6 4 . Obtenha o termo independente de x no desenvolvimento do binômio + ■— -j .

2 6 5 . Um dos termos no desenvolvimento de (x + 3a)5 é 360x3. Sabendo que a não


depende de x, determine o valor dç a.

2 6 6 . Determine o valor de a , de modo que um dos termos do desenvolvimento de


(x + a )5 seja 270x2.
I I \517?
2 6 7 . Qual é o termo independente de x no desenvolvimento de íx — —)

2 6 8 . Que posição ocupa o termo independente de x no desenvolvimento de


(5 + 6x 2)n , se o desenvolvimento for em potências de expoentes decrescentes
de x?

2 6 9 . Qual é a condição que n deve satisfazer para que o desenvolvimento de íx + -Ç )


tenha um termo independente de x ? \ x j
I ]\ 2n+l
2 7 0 . No desenvolvimento de (x + — j , n G IN*, pela fórmula do binômio de
Newton, existe um termo que não depende de x ?

67
minAmio de: newton

271. Sabendo que o quarto termo do desenvolvimento de (2x - 3y)n é - 1 080x2y 3, cal­
cule o terceiro termo desse desenvolvimento.

2 7 2 . Os três primeiros coeficientes do desenvolvimento de x2 + estão em pro­


gressão aritmética. Determine o valor de n.

2 7 3 . Os coeficientes do quinto, sexto e sétimo termos do desenvolvimento de (/ + x)n


estão em progressão aritmética. Se n ^ 10, calcule o valor de ( 2n - /).

274. No desenvolvimento do binômio (a + b )n+\ ordenado segundo as potências de­


crescentes de a, o quociente entre o termo que ocupa a (n + J)-ésim a posição
2 b2 T 2 b2
por aquele que ocupa a (n + /)-ésima é - isto é: . Determine
3* Tn+J 3az
o valor de n.

275. Qual é o produto do terceiro pelo antepenúltimo termo do desenvolvimento


d e i x + 4 -"?

2 7 6 . Qual o coeficiente de Jtn+/ no desenvolvimento de ( x + 2)n • x 3l

2 7 7 . Determine o coeficiente de a n+,~pb p no produto de

ak + a k~l b + ... + a k~pbP + ... + M jx>r (a + b ), para k = n.

278. Qual o valor do termo independente de x no desenvolvimento de

2 7 9 . Quantos termos racionais tem o desenvolvimento de (yj2 + l13 )y°°?

2 8 0 . Qual é o número de termos racionais no desenvolvimento de (2^3 + J S) /0?

2 8 1 . Calcule aproximadamente ( 1,002)20f usando o teorema binomial.

Solução

Vamos mostrar que (/ + x )n s / + nx para nx pequeno.

De fato, pelo teorema binomial:

d + X)" = 1 + ( j ) X + (5 ) X2 +

n ( n - 1) n ( n - 1 ) (n —2)
(1 + x)n = 1 + nx + xz + x-* + xn
3!

68
BINÔMIO DE NEWTON

porém
n ( n - 1 ) x2 n2x 2
------- 2------- < 2
n ( n - 1 ) ( n - 2) x 3 3V3
nJx
3!
etc.
Se nx é pequeno (próximo de zero), então n 2x 2, n3x 3, etc. são muito pe­
quenos, comparados com nx. Desprezando os termos do desenvolvimento
a partir do 3? termo, teremos:
(1 + x)n = 1 + n • x.
No nosso exemplo: (1,002)2° = (1 + 0,002 )20 = 1 + 20 •0,002 = 1,04.
Se calcularmos ( / y002)2° sem a aproximação, obteremos 1,0408.

2 8 2 . Calcule aproximadamente:
a) (1,002)'° b) (0,997 )20

2 8 3 . Usando o binômio de Newton, determine a aproximação, a menos de um centési­


mo, de ( 1,003)2°.

2 8 4 . Qual a soma dos coeficientes dos termos do desenvolvimento de (2x + 3y)4t}.

Solução

(2x + 3y)4 = (2x )4 + 4 •(2x )3•(3y) + 6 •(2x )2 •(3y)2+4 •(2x) •(3y)3 + (3y)4
Essa igualdade vale v x, y reais; s^ fizermos x = 1 e y = /, teremos:
1? membro: (2 • 1 + 3 • l )4 = 54 = 625
2? membro: 24 + 4 . 2 3 . 3 + 6 • 2 2 . 32 + 4 •2 • 33 + 34
que é exatamente a soma dos coeficientes. Logo, a soma dos coeficientes
é 625.

2 8 5 . Qual a soma dos coeficientes dos termos do desenvolvimento de:


a) (3x + 2y)10? b) (5x + y)8?

286. Indique a soma dos coeficientes de (4x + 3y)4 sem efetuar o desenvolvimento.

2 8 7 . Qual a soma dos coeficientes dos termos do desenvolvimento de:


a) (x - y)5? b) (3x - y)4?

69
BINÔMIO DE NEWTON

2 8 8 . Quando você desenvolve ( 5 jc + 2yY pelo binômio de Newton, aparecem coefi­


cientes numéricos e potências de x e y. Determine a soma dos coeficientes nu­
méricos.

2 8 9 . Determine /?, sabendo que a soma dos coeficientes numéricos do desenvolvimen­


to de ( jc + a ) p é igual a 512.

5
2 9 0 . ( 2 x - y )4 = a ,x 4 + a^x3y + ajx 2y 2 + ajxy3 + a y 4. Calcule o valor de at.

2 9 1 . A soma dos coeficientes dos termos de ordem ímpar de ( jc - y Y é 256. Deter­


mine n.

2 9 2 . Sendo 1024 a soma dos coeficientes do desenvolvimento de (3x + /)m, cal­


cule m.

2 9 3 . Sabendo que a soma dos coeficientes de (a + b )m é 256, calcule o número de


permutações de - y - elementos.

IV. Triângulo aritmético de Pascal (ou de Tartaglia)

59. É um a tab ela onde podem os dispor ordenadam ente os co eficien tes bino-
m iais: ( " ) .

70
BINÔMIO DE NEWTON

Isto é:
A 1? linha contém o co eficien te binom ial com n = 0
A 2? linha contém os co eficien tes binom iais com n = 1
A 3? linha contém os co eficien tes binom iais com n = 2

A k? linha contém os coeficien tes binom iais com n = k


etc.

60 . Pod em os tam bém escrever o triângulo de P a sca l substituindo cad a co e fi­


cien te binom ial pelo seu valor, isto é:

1
1 1
1 2 1
1 3 3 1
1 4 6 4 1
1 5 10 10 5 1

61. N otem os que:

A 1 ? linha do triângulo contém os coeficien tes do d esenvolvim ento de


(x + a)°.
A 2? linha do triângulo con tém os co eficien tes do d esenvolvim ento de
(x + a )'. (
A 3? linha do triângulo con tém os coeficien tes do d esenvolvim ento de
(x + ay.
E assim por diante.

(x + a)°

(x + a )1

(x + a y

(x + a y

71
biblio teca
\ I A*
BINÔMIO DE NEWTON

62. Observação

Na co n stru ção do triângulo de P ascal, não é necessário calcu lar os co e fi­


cientes binom iais um a um. B asta usarm os algum as de suas propriedades.

63. Propriedades do triângulo de Pascal

1?) E m cad a linha do triângulo, o prim eiro elem ento vale /, pois, qu al­

quer que seja a linha, o prim eiro elem ento é = /, v n E IN.

m
m ■
□ 2 ■
[T ] 3 3 1

JT j 4 6 4 1

2 ?) E m cad a linha do triâng u lo, o últim o elem ento vale /, pois, qualquer

que seja a linha, o últim o elem ento é f 72 = /, V n G IN.

m
■ m
1 2 H

1 3 3 tu

1 4 6 4 [T ]

3?) A partir da 3? linha, cad a elem ento (com exceção do prim eiro e do
ú ltim o) é a som a dos elem entos da linha an terio r, im ediatam ente acim a dele.

72
BINÔMIO DE NEWTON

E sta propriedade é conhecida co m o relação de Stifel e a firm a que:

n ^ 2

1 4 1
1 « ò"10*® 5' 1
1 6 ^ 15- 20 15 6 1

A d em onstração desta propriedade está na parte de exercícios resolvidos.

4 ? ) Numa linha, dois co eficien tes binom iais eqüidistantes dos extrem os
são iguais.
Isto equivale a dem onstrar que:
n
n - p
O que é im ed iato, pois:

M n!
\P/ p! (n - p)!

í n 1 - n!
©
\n - p } ( n , - p)! p! j

64. Exemplos
1 1
1 2 1
1
0 0 1
i 0 6
E
1 5
0 5

0 1
5 20 15
h
7 21 35 35 21

73
BINÔMIO DE NEWTON

EXERCÍCIOS
2 9 4 . Assinale com V as sentenças verdadeiras e com F as falsas.

295. Demonstre que:

Solução

Vamos desenvolver (/ + /)" pelo teorem a b inom ial.


T em os:

in+ (!j i“- ‘ •i ' + . . . + f ' in-‘ -1* + ... + (“ ) ■i°


l 1i/ \ii/ \n/

-(s)♦(:)+ (”)•
296. Calcule:

297. Calcule o determinante:

298. Calcule:

299. Calcule m, sabendo que: 1023.

74
BINÔMIO DE NEWTON

3 0 0 . Calcule £ ( ” )•
p=J

3 0 1 . Calcule £ (//)•

3 0 2 . Sejam n E IN*, p E IN, onde IN = ( 0, 1, 2, ...J e IN* = (/, 2, 3 , ...J. Calcule


n . .
o valor de £ ( - l ) p ( - l ) n~p j.
p =o

n / .
3 0 3 . Determine o valor de ^ (2 p3 n~p - 4 P), para todo n > 0.
p =o

3 0 4 . Prove que, se um conjunto tem n elementos, entào o número de subconjuntos


de A é 2".

3 0 5 . Quantos subconjuntos não vazios possui o conjunto A com n elementos?

3 0 6 . Calcule o valor da expressão:

, + /l\n + V* / n \ / ± r k I3_)k
41 Lj \k/ \4/ \4
k= I

3 0 7 . Demonstre que V n E IN3*

3 0 8 . Se p > 0 y q > 0 yp + q = / e (p + q ) n = ^ (^ J p ‘q n ', n > 0y demonstre

que ( .) p lq n 1 é sempre menor do que /.

3 0 9 . Verifique que, quando n é ímpar,

2n_l = ? ) + ( ? ) + ín + ... +

Sugestão:

s++ U.HCM.-2) = 2n

75
BINÔMIO DE NEWTON

310. Prove que:

Solução

Sabemos que:

«, + *>F- ( ! ) + (í)* + (;)*,+ - + (n)


Derivando membro a membro em relação a temos:

* • <! + x>n' ‘ = ( ? ) + 2 ( ? ) x + 3 ( ? ) x2 + - + n ( ? ) x"~'


Fazendo x = I nesta igualdade, resulta:

, que é o que queríamos

demonstrar.

3 1 1 . Prove que:

2 ■ 1 ( j ) + 3 • 2 ( j ) + 4 • 3 ( j ) + ... + n • (n - 1) ( j ) = n(n - 1) • 2"“2.

3 1 2 . Demonstre a relação de Euler

Sugestão: (1 + x)m+n = (1 + x)m • (1 + x)n; desenvolva cada membro e iden­


tifique os coeficientes dos termos semelhantes.

3 1 3 . Usando a relação de Euler, prove que:

314. Demonstre a relação de Stifel, isto é:

( “ ) = ( p I j ) + ( " P * ) v n G 'N . n > 2 e p < n.

Solução

Consideremos um conjunto A com n elementos, e consideremos um deter­


minado elemento a G A. Vamos calcular o número de combinações dos
elementos de A , tomados p a p , de dois modos:

76
BINÔMIO DE NEWTON

1? m od o : Diretamente pela fórmula, isto é


•0 ®
2? m od o : Calculamos o número de combinações que não possuam o ele­
mento a.

Tal número é
%')■
Em seguida, calculamos o número de combinações que possuem o elemen­
to a. Tal número é ^ j j.

Ao todo, o número de combinações será:

( V M r !) ®
De (7) e (ü ) concluímos que:

3 1 5 . Demonstre que a soma dos quadrados dos n primeiros números inteiros positivos é:
o n • (2 n + 1 ) (n + 1 )
6 *
Sugestão: Use a identidade
(x + l )3 x 3 + 3x2 + 3x + 1
e faça x assumir os valores 1, 2, 3, ..., n.

3 1 6 . Escreva n parcelas contendo o desenvolvimento de (A: + l ) 3 para k = 1, 2, 3,


..., n - 1, n. Some todas as parcelas'* elimine os termos semelhantes e obtenha
l 2 + 2 2 + 3 2 + ... + n 2.

317. Mostre que, se n(n ^ 2) é par, os valores de j para p = 0, 1, 2, ..., n vão

crescendo, atingem um valor máximo para p = -y- e depois vão decrescendo.

Solução

Consideremos dois coeficientes binomiais consecutivos í n_ / )e ( « ) e ca^"


culemos seu quociente: '

77
BINÔMIO DE NEWTON

n!
p! (n - p)! n- p + 1
n n!
P“ 1 (p - 1 )! (n - p + 1 )!

a) Os valores de ^ j irão crescendo, até atingir o máximo se, e somente


n- p + I
s e , ---------------- > /.
P
Portanto:
n + 1
n - p + 1 > p <=> n + l > 2 p p<

Isto é, j irá crescendo, quando p variar de 0 até o menor inteiro que


n + 1 , n í
nao supera — - — , que e — .

b) Os valores de J irão decrescendo se, e spmente se, —— ■■ < 1.


Portanto:
n + 1
n - p + l < p <=> n + l < 2p <=> p >

Isto é j irá decrescendo, quando p variar de - j - + / até n.

c) De (a) concluímos que o maior valor de ( n ) é atingido para p = Ar

Exemplo
Os coeficientes binomiais para n = 4 são:
4
2 5
( )
6 4
t —>
aumenta valor máximo diminui

3 1 8 . Mostre que, se n é ímpar, os valores de ^ j para p = 0 , 1 , 2 , n vão crescendo,

atingirão valor máximo para dois valores de p \p = n ^ ^ t p = — ~ - ) e em


seguida vão decrescendo.

78
BINÔMIO DE NEWTON

3 1 9 . Determine a condição para que ^ j seja o dobro de

3 2 0 . Seja P( x) = a0 + a ,x + a2x 2 + ... + a l00x 100, em que a l00 = /, um polinô-


mio divisível por (x + 9 )100. Calcule o valor de a2.

3 2 1 . Resolva a equação trigonométrica:


sen4x - 4 •sen3x + 6 • sen2x - 4 • sen x + 1 = 0
utilizando o binômio de Newton.

3 2 2 . Calcule p na equação ^

Solução

Já vimos que a equação em x , , tem solução para x = p ou


x = n - p . Em virtude das propriedades dadas nos exercícios 318 e 317,
isto é, os binomiais crescem inicialmente, atingem um, ou dois

valores máximos, e depois decrescem, concluímos que para


no máximo dois valores de x que, conforme já vimos, são x = p e
x = n - p.
Portanto, a solução da equação dada é:
3p = p + 6 ( 1)
ou
,3p = 14 - (p + 6) (2)
(1) => p = 3 ou (2) => p = 2. \

3 2 3 . Sendo ( 3 j = calcule p.

3 2 4 . Resolva ( " ) = ( J í ,)■

12 12
325. Resolva a equação:
P + 3 P“ /

326. Determine m para que

79
BINÔMIO DE NEWTON

3 2 7 . Uma pessoa possui um certo número m de objetos distintos. Agrupando-os 3 a


3 de modo que cada grupo difira do outro por possuir pelo menos um objeto
diferente, obtém-se o mesmo número de grupos se os juntar 5 a 5, do mesmo modo.
fm y
Determine

m
3 2 8 . Sendo m , p e q números inteiros e positivos, com q < p e ^ ™ ^
p -q
determine a relação entre eles.

m - 1
3 2 9 . Sabendo que _ j j = 10 e | = 55, calcule

3 3 0 . Seja IN o conjunto dos números inteiros positivos. Determine o conjunto de todos

os n E IN, n > 2, para os quais

3 3 1 . Qual(is) o(s) maior(es) coeficiente(s) binomial ^ j para:

a) n = 12? b) n = 15?

3 3 2 . Qual o termo de maior coeficiente no desenvolvimento de ( J x + y 2),0l

V. Expansão multinomial

6 5 . J á vim os co m o é possível o b ter o desenvolvim ento de um binôm io


( jc + a )n, v n G IN.
V am os agora, com raciocínio sem elhante, obter o desenvolvim ento de ex­
pressões do tipo ( jc + y + z ) n, ( x + y + z + t)n, etc. ( n G IN), em que a
base da potência de expoente n é um p olinôm io.

66. E x e m p lo 1

(x + y + z)5 (x + y + z) • (x + y + z) • (x + y + z) • (x + y + z) • (x + y + z)|

5 fatores <

P ela propriedade distributiva da m u ltiplicação, devem os tom ar um ter­


m o de cad a fa to r (escolhidos entre x, y , z ) e em seguida m u ltiplicá-los. Feitas
tod as as escolhas possíveis e m ultiplicados os term os, a som a desses produtos

80
BINÔMIO DE NEWTON

será o desenvolvim ento de (x + y + z ) 5. Os tipos de produtos que podem os


o b te r são da form a
X' . yi . zk
em que /, j , k E IN e i + j + k = 5.
P a ra cada i, j , k fixad os, o co eficien te do term o x ‘ • y J • z k será o nú­
m ero de seqüências de cin co letras, com / letras x, j letras y e k letras z, isto é:

pi» j- k __ 5!
5 i! j! k! *

P o rta n to , o co eficien te de x' • y j • z k é


51
ü j ! k! '
T o m an d o tod os os term os do tip o x ‘ • y j • z k para /, j , k E. IN e
/ + j + k = 5 e calcu land o os seus co eficien tes, a som a deles, precedidos pe­
los respectivos co eficien tes, d ará a expansão de (x + y + z } 5.
E m p articular, o co eficien te do term o x 2 - y 2 • z será:
p2.2.I 5!
= 30.
2 ! 2 ! 1!
P o rta n to , o term o em x 2y 2z será 3 0 x 2 • y 2 • z.
D e um m odo geral, a expansão do p olinôm io, (jcy + + . .. + xr)n9
com x It x 2, x r E IR e n E IN será

i nr!

em que a som a é estendida p ara:

(n „ n 2, nr E IN

n, + n 2 + \... + nr = n.

67. Exemplo 2
Q ual o co eficien te de x y z no desenvolvim ento de (x + y + z )3!
O co eficien te de xyz é:

68. Exemplo 3

Qual o co eficien te de x 5 n o desenvolvim ento de (7 4- x + x 2)/0?

81
BINÔMIO DE NEWTON

O term o genérico é:
10 ! 10! xj +2k
a y •w • (x2)k
i! j ! k! i! j ! k!

Devemos im por que j + 2 k = 5 . Vam os resolver essa equação, atribuindo


valores para j e notand o que / está au tom aticam en te determ inado pela condi­
ção i + j + k = 10.

j k i

1 2 7
3 1 6
5 0 5

N otem os que para j = 0 ou j = 2 ou j = 4 ou j = 6 ou j = 7 ou


j = 8 o u j = 9 o u j = 10 não existe k E IN satisfazend o j + 2 k = 5.
T e n lo s, então:

1) i = 7; j = 1; k = 2
O co eficien te de jc5 será: ^ *2! =

2) / = 6; j = 3 ; k = /
10!
O co eficien te de x 5 será: 840.
6 ! 3! 1!

3) i = 5 ; j = 5 ; k = 0

O co eficien te de será: ^ 5^/qt = 252.

L o g o , o co eficien te de x 5 (desenvolvendo to d o o polin ôm io) será:


(3 6 0 + 84 0 4- 252) = 1 452.

EXERCÍCIOS
3 3 3 . Desenvolvendo o polinômio ( jc + y + z)4, qual o coeficiente do termo em x 2yz1
E do termo xyz 2?

3 3 4 . Qual o coeficiente do termo em x 2y 3z 2 no desenvolvimento de (x + y + z )7?

82
BINÔMIO DE NEWTON

3 3 5 . Mostre que o coeficiente de x 3 no desenvolvimento de (/ + 3x + 2x2)w é 3 780.

3 3 6 . Qual a soma dos coeficientes dos termos do desenvolvimento de (x + y +

3 3 7 . Determine o termo independente de x em |/ + x + - y j .

3 3 8 . Qual é o coeficiente de x 8 no desenvolvimento de (/ + x 2 - x 3)9!

LEITURA

Pascal e a Teoria das Probabilidades


H ygino H. D om ingues

Som ente cerca de cem anos depois de G iro lam o C ard ano escre­
ver seu L ib e r d e lu d o a le a e (em to rn o de 1550; ver pág. 5 7 ), o b ra co n ­
siderada o m arco inicial da teoria das probabilidades, seria dado o passo
seguinte (e decisivo) para a criação dessa área da m atem ática.
O cenário ag ora era a F ra n ça , onde o requintado n obre francês
A n toine G am baud , o Chevalier de M éré, co m o C ard ano um invetera­
do jo g a d o r, estava às voltas com problem as co m o : “ D ois jo g a d o res
de igual habilidade resolvem interrom per o jo g o antes do térm ino. Sen ­
do conhecido o núm ero de porups de cad a um até essa altu ra, em que
p rop orção devem ser divididas as a p o sta s?” . A pesar de possuir várias
idéias aritm éticas sobre o assu nto, fru to de sua experiência e perspicá­
cia , G am baud decidiu recorrer ao grande m atem ático francês Blaise
P ascal (1623 -1 6 6 2 ). E ste se entusiásm ou tanto com as questões que até
iniciou correspond ência a respeito com seu con terrân eo P ierre de Fer-
m at, resultando desse episódio as bases da m oderna teoria das p ro b a ­
bilidades.
Ó rfã o de m ãe aos 3 anos de idade, P ascal foi educado por seu
pai, um intelectual respeitad o, com idéias pedagógicas n ão m uito co n ­
vencionais. Assim é que, segundo seus preceitos, considerando a débil
saúde do filh o , achava que este só deveria com eçar a aprender geom e­
tria aos 15 anos de idade. M as o jo v em , aos 12, co n trarian d o a p ro ib i­
çã o , pôs-se a querer reinventar o assunto por co n ta p rópria, com oveu
o pai e acabou ganhando um exem plar dos E lem en tos de Euclides. A os
14 passou a freqü entar as reuniões cien tíficas prom ovidas pelo m ate­
m ático M . M ersenne (1588-1648) das quais participavam D escartes, R o-
berval, Desagues e seu pai, entre o u tro s. D ois anos depois apresen-

83
tava uma contribuição notável: a m em ória “ Essay pour les coniques” ,
uma obra-prim a da geometria projetiva em uma folha impressa apenas.
Em 1642, portanto com 17 anos de idade, para aliviar seu pai dos
exaustivos cálculos que era obrigado a fazer, com o fiscal na N orm an-
dia, planejou uma m áquina de calcu lar. A Pascaline, com o veio a se
cham ar, cu jo modelo definitivo é de 1652, chegou até a ser com erciali­
zada (em bora sem o sucesso previsto por Pascal) e representa um dos
mais antigos protótipos de calculadoras m ecânicas.
Em 1654, depois de se dedicar
por algum tempo à física experi­
m ental, P ascal voltou à m atem áti­
ca em duas frentes. De um lado pa­
ra escrever a “ O bra com pleta sobre
cônicas” (certam ente um a conti­
nuação do pequeno “ E ssay ” ), que
nunca foi impressa e acabou se per­
dendo. A outra frente foi a teoria
das probabilidades.

Pascal (1623-1662).

E m bo ra sem transform ar em livro sua correspondência sobre o


assunto com Ferm at (a qual seria aproveitada por Huygens), em 1654
redige seu T ratad o d o triângulo a ritm ético, uma exposição das pro­
priedades dos coeficientes binom iais e relações entre eles (a primeira
sistem ática a ser feita — daí o triângulo estar associado ao nom e de
P ascal), com alguns princípios de probabilidade. P o r exem plo, a soma
dos term os da terceira diagonal representa o
1 1 1 1 1 núm ero de possibilidades no lançam ento de
três m oedas; e esses term os, as ocorrências
1 2 3 4 possíveis: uma possibilidade de três caras; três
possibilidades de duas caras e uma co ro a; etc.
1 3 6
O triângulo da figura é apresentado segundo
1 4 o m odelo de Pascal.
D epois disso P ascal se recolheu à medi­
1
tação religiosa, voltando à m atem ática ape­
nas uma vez m ais, em 1658, para trabalhar
febrilm ente, m ovido por razões m ísticas, na geom etria da ciclóide. O
mesmo misticismo que fez com que Pascal fosse, dentre os grandes ma­
tem áticos, aquele que provavelm ente menos empenhou tod a a geniali­
dade de que era dotado a serviço de sua ciência.

84
BiNOMIO DE NEWTON

L E IT U R A

Os Irmãos Jacques e Jean Bernoulli


Hygino H. Domingues

Im portantes cam pos novos da m atem ática, com o o C álcu lo, a


G eom etria A nalítica e a T eo ria das Probabilidades, despontaram em
sua form a m oderna no século X V II . M as, obviam ente, considerando
inclusive o estágio da m atem ática na ép oca, de m aneira incipiente e
até meio tosca. E xp lo rar as potencialidades desses cam pos e
fundam entá-los seria um a tarefa longa. E já no século X V II esse tra­
balho se inicia revelando nom es de grande talento m atem ático, com o
os irm ãos Jacqu es Bernoulli (1654-1705) e Je a n Bernoulli (1667-1748),
da Basiléia, na Suíça.
A fam ília Bernoulli pertencia à burguesia com ercial da Basiléia,
onde se fixara, vinda em fuga da A ntuérpia no final do século X V I,
após esta cidade ter sido conquistada pela Espanha cató lica (os B er­
noulli eram huguenotes). C erca de meio século depois, por alguma mu­
tação difícil de explicar, a fam ília com eçou a produzir cientistas (não
sem decepcionar alguns patriarcas) de m aneira talvez inédita na histó­
ria da hum anidade. S ó m atem áticos, até a prim eira metade do século
X I X , contam -se nada m enos que treze. M as possivelmente nenhum te­
nha superado em brilho os irm ãos Jacq u es e Je a n já citados.

Nicolaus
(1623-1708)

Jacqu es I Nicolaus I Je a n I
(1654-1705) (1662-1716) (1667-1748)

N icolaus II Nicolaus III D aniel I Je a n II


(1687-1759) (1695-1726) (1700-1782) (1710-1790)
I I
Je a n 111 D aniel II Jacqu es II
(1746-1807) (1751-1834) (1759-1789)
I
Christoph
(1782-1863)

Jea n Gustave
(1811-1863)
Os Bernoullis matemáticos: árvore genealógica.

85
Jacq u es graduou-se em teologia em 1676 na Universidade da B a ­
siléia, atendendo aos d esejos do pai. Os seus próprios d esejos ap are­
cem no lem a que p osteriorm ente ad otou : “ Invito patre sidera verso”
(E stu d o as estrelas co n tra a vontade de meu pai).
A ssim , entende-se por que desde os tem pos de estudante dedica­
va o m elhor de seu tem po à m atem ática e à astronom ia. D e 1676 a 1682
percorre F ran ça, In g laterra e H oland a para se atualizar cien tificam en ­
te e na volta à Basiléia funda um a escola de m atem ática e ciên cia. C in ­
co anos depois assum iu a cad eira de M atem ática da Universidade lo ­
ca l, onde ficou até à m orte.
N o que se refere ao C álcu lo , Ja cq u es o estudou na form a ideali­
zada por L eibniz, sendo aliás um dos prim eiros m atem áticos a dom i­
nar os artigos em que este lançou as bases de suas idéias sobre o assun­
to . A o co n trário de N ew ton, L eibniz era aberto à tro ca de in fo rm a ­
ções cien tíficas, co m o que conseguiu m uitos seguidores e co rresp o n ­
d entes, entre os quais Ja cq u e s. <

D entre as m últiplas co n tribu içõ es de Ja cq u es à m atem ática, ta l­
vez a que o tenha tornad o m ais co n h ecid o seja seu livro A rs con jectan -
d i ( A a rte d e co n jec tu ra r) no qual trabalh o u cerca de 20 anos (sem
co m p letá-lo totalm ente) e que só fo i publicado após sua m orte (em
171*3). T ra ta -se da prim eira o b ra substancial sobre a teoria das p ro b a­
bilidades.
O A rs c o n jec ta n d i está dividido em qu atro partes. Na prim eira
reproduz a breve in trod u ção de Huygens ao assunto. A segunda é
um apanhad o geral dos resultados básicos sobre perm utações e co m b i­
nações. N ela figura inclusive a prim eira d em onstração co rreta (por in­
dução) do teorem a binom ial para expoentes positivos. A terceira apre­
senta 24 problem as sobre jo g o s de azar m uito populares na ép oca. A
última term ina com o célebre “ teorem a de B em ou lli” ou “ Lei dos gran­
des núm eros” (Ja cq u e s não viveu para incluir nela as aplicações à eco ­
nom ia e à política que tinha em vista).
Je a n B ernou ili, segundo os planos de seu pai, deveria suce­
dê-lo à testa dos negócios da fam ília. C on tu d o , sem v o cação c o ­
m ercial, conseguiu dissuadir o velho de suas intenções co n cord an d o
em fazer m edicina. M as, sim ultaneam ente, era orientad o pelo irm ão
para o cam inho que aspirava trilhar — o da m atem ática e das ciências
físicas. T a n to qu anto Ja cq u e s, logo dom inou os m étodos do cálcu lo
de L eibniz, tornand o-se um dos m aiores expoentes e divulgadores
do assu nto em sua ép oca. A pós 10 anos co m o p rofessor de M a­
tem ática da U niversidade de G roningen (H o lan d a), em 1705 sucedeu
o falecid o irm ão na U niversidade da B asiléia, onde tam bém ficou até
a m orte.

86
Um episódio que m arcou a vida de
Je a n foi seu relacionam ento com o M a r­
quês de L ’H ospital (1 6 6 1 -1 7 0 4 ). E ste n o ­
bre fran cês, desejand o dom inar o C álcu ­
lo , então uma novidade cien tífica , co n tra ­
tou para tan to os serviços de Je a n , o qu al,
sabe-se lá por que, concordou até com que
o M arqu ês usasse com o lhe aprouvesse as
descobertas que fazia e que a ele com u ni­
cava. E em 1696 L ’H ospital lançou o livro
A n a ly se d e s in fin im en t p e tits , o prim eiro
texto de C álcu lo a ser p u blicad o, não sem
agradecim entos especiais, em bora genéri­
Jean Bernoulli (1667-1748). co s, ao “ jo v em professor de G ron in g en ” .
O livro teve m uito sucesso, o que chegou
a envaidecer Je a n . M as este, após a m orte do au tor, passou a reivindi­
car a paternidade de b o a parte do conteü do do livro — tudo indica
que com razão. P o r exem plo, o teorem a sobre limites de qu ocientes,
con hecid o h o je com o reg ra d e L 'H ospital, m uito provavelm ente é de
Je a n Bernoulli.
A m atem ática, que foi um elo de
ligação a mais entre os irm ãos Jacq u es
e Je a n , acabou por estrem ecer as rela­
ções entre am bos, dado o zelo com que
se enpenhavam em suas pesquisas. O pi­
vô da desavença entre am bos pode ter
sido o problem a da b ra q u istó cro n a (no­
me derivado das palavras gregas m en o r
e tem p o ) com que a certa altura Jean' de­
safiou a com unidade m atem ática do
m undo. D ever-se-ia en co n trar a curva
que uma partícula descreve para descer,
sob a ação de gravidade, no m enor es­
paço de tem po possível, de um p onto A
a um p o n to B (n ã o d ir e ta m e n ­
te abaixo d e A ) . A solução do p roble­
m a é o arco (único) da cic ló id e in verti­
d a unindo A com B . A ciclóid e é a cu r­
va descrita por um ponto de uma circun­
ferência que roda sem deslizar sobre
uma reta — ver figura. Jacques Bernoulli (1654-1705).

87
A ciclóide (invertida).

Som ente cinco m atem áticos da época chegaram a essa resposta


acertadam ente: Newton, Leibniz, L ’ Hospital e os irm ãos BernoulU. Se­
gundo algum as versões, a resolução inicial de Je a n n ão era sa tisfa tó ­
ria, o que o teria levado a ten tar u sar, de algum a m aneira, a do irm ão.
D aí talvez o atrito . M as sua solu ção fin al, além de orig in al, tinha um
alcan ce m aior que a de Ja cq u e s, sendo consid erad a, inclusive, o ponto
de partid a de um novo ram o da m atem ática: o cálcu lo de variações.

88
CAPITULO III

Probabilidade
I. Experimentos aleatórios

69. C ham am os de experim entos aleató rio s aqueles que, repetidos em idên­
ticas cond ições, produzem resultados que não podem ser previstos com certe­
za. E m b o ra não saibam os qual o resultado que irá oco rrer num experim ento,
em geral conseguim os descrever o co n ju n to de to d o s o s resu ltad os p o ssív eis que
podem o co rrer. As variações de resultados, de experim ento para experim ento,
são devidas a uma multiplicidade de causas que não podemos co n tro lar, as quais
d enom inam os a c a s o .

70. Exemplos de experimentos aleatórios


s1
a) L an çar uma m oeda e observar a face de cim a.
b) L an çar um dado e observar o núm ero da face de cim a.
c) L an çar duas m oedas e observar as seqüências de caras e co ro as
ob tid as.
d) L an çar duas m oedas e observar o núm ero de caras obtid as.
e) D e um lote de 80 peças boas e 20 d efeitu osas, selecion ar 10 peças
e observar o núm ero de peças d efeitu osas.
f ) D e um a urna contend o 3 bolas verm elhas e 2 bolas b ra n ca s, selecio­
nar um a bola e observar sua co r.
g) De um baralho de 52 cartas, selecionar uma carta e observar seu naipe.
h) Num a cidade onde 10°7o dos habitantes possuem d eterm inada m o­
léstia, selecionar 20 pessoas e observar o núm ero de portadores da m oléstia.

89
PROBABILIDADE

i) Observar o tempo que um certo aluno gasta para ir de ônibus de sua


casa até a escola.
j) Injetar uma dose de insulina em uma pessoa e observar a quantidade
de açúcar que diminuiu.
k) Sujeitar uma barra metálica a tração e observar sua resistência.

II. Espaço amostrai


71. C ham am os de espaço am o strai, e indicam os por Q, um co n ju n to fo r­
m ado por t o d o s o s resu lta d o s p o s s ív e is de um experim ento a leató rio .

72. Exemplos

a) L an ça r um a m oeda e observar a face de cim a.


Q = [K , C ), em que K representa ca ra e C , c o r o a .

b) Lançar um dado e observar o número da face dê cima.


0 = [ 1 , 2 , 3, 4, 5, 6)

c) D e um a urna co ntend o 3 bo las verm elhas ( V ) , 2 b olas b ran cas (B )


e 5 bolas azuis ( A ) , extrair um a b o la e observar sua co r.
0 = [V , B , A )

d) L an ça r um a m oeda duas vezes e observar a seqüência de caras e


co ro a s.
ft = f (K , K ) , ( K , C ) , ( C , K ) , ( C , C ) j

e) L an çar um a m oeda duas vezes e observar o núm ero de caras.


n = [0, í , 2}

f ) U m lote tem 2 0 peças. U m a a um a, elas são ensaiadas e observa-se


o núm ero de d efeitu osas.
0 = [0, 1, 2, 3, ... , 19, 20}
g) U m a m oeda é lançad a até que o resultado cara ( K ) o co rra pela pri­
m eira vez. O bserva-se em qual lançam ento esse fa to ocorre.
ft = ( 1 , 2 , 3, 4, ...}

73. Observação

D irem os que o espaço am o strai í) é fin ito , se # Í I = n E IN* (é o caso


dos exem plos a, b , c, d, e, / ) ; caso co n trário direm os que íí é in fin ito (é o caso
do exem plo g ).

90
PROBABILIDADE

N este liv ro , n o s restrin g irem os a o s ex p erim en to s a le a tó r io s c u jo s es p a ­


ç o s am o stra is s ã o fin ito s .

EXERCÍCIOS
Dê um espaço amostrai para cada experimento abaixo.

3 3 9 . Uma letra é escolhida entre as letras da palavra PRO BA BILID A D E.

3 4 0 . Uma uma contém bolas vermelhas ( K), bolas brancas ( B ) e bolas azuis ( A ). Uma
bola é extraída e observada sua cor.

3 4 1 . Uma urna tem 50 bolinhas numeradas de 1 a 50. Uma bolinha é extraída e obser­
vado seu número.

342. De um baralho de 52 cartas, uma é extraída e observada.

343. Uma urna contém 5 bolas vermelhas ( V) e 2 brancas ( B ). Duas bolas são extraí­
das, sem reposição, e observadas suas cores, na seqüência em que foram extraídas.

3 4 4 . Três pessoas A, B, C são colocadas numa fila e observa-se a disposição das mesmas.
1 **
3 4 5 . Um casal planeja ter 3 filhos. Observa-se a seqüência de sexos dos 3 filhos.

3 4 6 . Dois dados, um verde e um vermelho, são lançados; observam-se os números das


faces de cima. I

Solução

Podemos considerar cada resultado como um par de números (a, b ) em


que a representa o resultado no dado verde e b o resultado no dado verme­
lho. Isto é, f i é o conjunto.
0 = 10, D (2, 1 ) (3, 1 ) (4, 1 ) (5, 1 ) (6, 1 )
( 1 .2 ) ( 2 ,2 ) ( 3 ,2 ) ( 4 ,2 ) ( 5 ,2 ) ( 6 ,2 )
( 1 .3 ) ( 2 ,3 ) ( 3 ,3 ) ( 4 ,3 ) ( 5 ,3 ) ( 6 ,3 )
( 1 .4 ) ( 2 ,4 ) ( 3 ,4 ) ( 4 ,4 ) ( 5 ,4 ) ( 6 ,4 )
( 1 .5 ) ( 2 ,5 ) ( 3 ,5 ) ( 4 ,5 ) ( 5 ,5 ) ( 6 ,5 )
( 1 .6 ) ( 2 ,6 ) ( 3 ,6 ) ( 4 ,6 ) ( 5 ,6 ) (6, 6))

91
PROBABILIDADE

347 . Entre 5 pessoas A, B, C, D , E> duas são escolhidas para formarem uma comis­
são. Observam-se os elementos dessa comissão.

348 . Pergunta-se a uma pessoa (não nascida em ano bissexto) a data de seu aniversá­
rio (mas não o ano do nascimento). Observa-se essa data.

III. Evento

74. Considerem os um experim ento aleatório, cu jo espaço am ostrai é Í2. C ha­


m arem os de ev en to to d o su b co n ju n to de Í2. E m geral indicam os um evento por
um a letra m aiuscula do a lfa b e to : A , B , C, . . . , X , Y, Z.
D irem os que um ev en to A o c o r r e se, realizado o experim ento, o resulta­
do o btid o fo r pertencente a A . O s eventos que possuem um ú n ico elem en to
( # A = 1 ) serão cham ad os eventos elem entares.

75. Exemplos

1?) Um dado é lançad o e observa-se o núm ero da fa ce de cim a.


Q = (1, 2 , 3 , 4 , 5, 6)
E is alguns eventos:

A: o co rrên cia de núm ero ím par. A = [1, 3, 5).


B: o co rrên cia de núm ero prim o. B = (2, 3, 5 j.
C: o co rrên cia de núm ero m enor que 4. C = [1, 2 , 3).
D: o co rrên cia de núm ero m enor que 7. D = [1, 2 , 3, 4 , 5, 6 ] = Í2.
E: o co rrên cia de núm ero m aior ou igual a 7. E = 0.

2 ? ) U m a m oeda é lançad a 3 vezes, e observa-se a seqüência de caras e


co ro as.

fl = {(K , K , K ); (K , K , C ); (K , C , K ); (K , C , C ); (C , K , K ); (C , K , C );
(C , C , K ); (C , C , C )j.

E is alguns eventos:

A : o co rrên cia de cara ( K ) no 1? lan çam en to.


A = ((K , K , K ); (K , K , C ); (K , C , K ); (K , C , C)}

B : o co rrên cia de exatam ente um a co ro a .


B = ((K , K , C ); (K , C , K ); (C , K , K)J

92
PROBABILIDADE

C : o co rrên cia de, no m áxim o, duas co ro as.


C = [(K , K , K ); (K , K , C ); (K , C , K ); (K , C , C ); (C , K , K );
(C , K , C ); (C , C , K))

D : o co rrên cia de pelo m enos duas caras.


D = ((K , K , K ); (K , K , C ); (K , C , K ); (C , K , K)}

76. Observação

N otem os que, se # Í2 = n> en tão fi te rá 2 n su b co n ju n to s e, p o rtan to ,


2 n eventos. E n tre os eventos, salientam os o 0 (cham ad o ev en to im p ossív el)
e o próprio Q (cham ad o ev en to c e r to ).

IV. Combinações de eventos

Se usarm os certas operações entre co n ju n to s (eventos), poderem os co m ­


binar co n ju n to s (eventos) para fo rm ar novos co n ju n to s (eventos).

77. a) União de dois eventos

S ejam A e B dois eventos; então A U B será tam bém um evento que


o co rrerá se, e som ente se, A ou B ( o u a m b o s ) o co r r erem . D izem os que A U B
é a u n ião entre o evento A e o evento B .

78. b) Interseção de dois eventos


S e ja m A e B dois eventos; en tão A f l B será tam bém um evento que
o co rrerá se, e som ente se, A e B o c o r r e r e m sim u ltan eam en te. D izem os que
A f l B é a interseção entre o evento A e o evento B.
Em p articular, se A f l B = 0 , A e B são cham ados m u tu am en te ex ­
clusivos.

79. c) Complementar de um evento

S e ja A um evento; então A c será tam bém um evento que o co rre rá se,


e som ente se, A n ã o o co rrer.
D izem os que A c é o ev en to co m p le m e n ta r d e A .

93
PROBABILIDADE

80. Exemplo

Um dado é lançad o e observado o núm ero da face de cim a.


Ü = í l , 2, 3, 4 , 5, 6)

S ejam os eventos:

A: oco rrên cia de núm ero par. A = (2, 4 , 6 ]


B: oco rrên cia de núm ero m aior ou igual a 4. B = (4, 5, 6 j
C: o co rrên cia de núm ero ím par. C = [1, 3, 5]
en tão , terem os:

A U B : -ocorrência de núm ero par ou núm ero m aior ou igual a 4.


A U B = (2, 4 , 5, 6)
A H B : o co rrên cia de um núm ero par e um núm ero m aior ou igual a 4.
A n B = [4, 6)
A H C : o co rrên cia de um núm ero par e um núm ero ím par.
A f l C = 0 ( 4 e C m utuam ente exclusivos).
A c : o co rrên cia de um núm ero não par.
A c = (1, 3, 5)
B c : o co rrên cia de um núm ero m enor que 4.
B c = [1, 2 , 3}

81. d) União de n eventos

S e ja A „ A 2, . . . , A n um a seqüência de eventos. E n tão

U A ( = A , U A 2 U ... U A n
i=1

será tam bém um evento que o co rrerá se, e som ente se, a o m en o s um d o s even ­
to s A j o c o r r e r . D izem os que A , U A 2 U ... U A„ é a união dos eventos A h
A 2t • A n.

82. e) Interseção de n eventos

S e ja A „ A 2, . . . , A n um a seqüência de eventos. E n tão


n
P ) Aj = a , n a 2 n * ... n a „
i=1

será tam bém um evento que o co rrerá se, e som ente se, to d o s o s ev en to s Aj
o co r r e r e m sim u ltan eam en te.

94
PROBABILIDADE

83. Exemplo

Um núm ero é sorteado entre os 100 inteiros de 1 a 100. S e ja m os even­


tos A ji o co rrên cia de um núm ero m aior que /, V/ E (/, 2, 2, 4}.

E n tão :

A, = (2, 3, 100)
a2 = (3, 4........ 100)
A, = [4. 5........ 100)
a4 = [5. 6, .... 100)
4

u A, = (2, 3, 100) n a . [5, 6 , . . . . 100).

EXERCÍCIOS
3 4 9 . Uma uma contém 30 bolinhas numeradas de 1 a 30. Uma bolinha é escolhida
e observado seu número. Seja Q = (/, 2, 3, ..., 29, 30}. Descreva os eventos:
a) o número obtido é par
b) o número obtido é ímpar
c) o número obtido é primo )
d) o número obtido é maior que 16
e) o número é múltiplo de 2 e de 5
f) o número é múltiplo de 3 ou de 8
g) o número não é múltiplo de 6

3 5 0 . Dois dados, um verde e um vermelho, são lançados. Seja íi o conjunto dos pares
(tf, b ) em que a representa o número do dado verde e b do dado vermelho.
Descreva os eventos:
a) A: ocorre 3 no dado verde
b) B: ocorrem números iguais nos dois dados
c) C: ocorre número 2 em ao menos um dado
d) D: ocorrem números cuja soma é 7
e) E: ocorrem números cuja soma é menor que 7

95
PROBABILIDADE

351. Uma moeda e um dado sào lançados. Seja:


Q = j(K, 1); (K, 2); (K, 3); (K, 4); )K, 5); (K, 6);
(C, 1); (C, 2); (C, 3); (C, 4); (C, 5); (C, 6)!
Descreva os eventos:
a) A : ocorre cara e) B O C
b) B : ocorre número par f) a n c
c) C: ocorre o número 3 g) A c
d) A U B h) C c

352. Um par ordenado (a, b ) é escolhido entre os 20 pares ordenados do produto car-
tesiano A x B , em que A = [/, 2, 3, 4\ e B = [/, 2, 3, 4, 5 j.
Considere Í2 = |(cr, b) \ a G A a b G B J. Descreva os eventos:
a) A = J(x, y)lx = y! d) D = j(x, y) Iy = x2!
b) B = j(x, y)Ix > yl e) E = j(x, y)lx = lj
c) C = |(x, y) I x + y = 2| f) F = j(x, y) I y = 3j

353. Uma urna 1tem duas bolas vermelhas ( V) e três brancas e a urna II tem cinco
bolas vermelhas e seis brancas. Uma urna é escolhida e dela extraída uma bola
e observada sua cor. Seja:
Q = [(I, V); (I, B); (II, V); (II, B)|
Descreva os eventos:
a) A: a urna escolhida é a I e) A UB
b) B: a urna escolhida é a II f) A fl C
c) C: a bola escolhida é vermelhag) Dc
d) D: a bola escolhida é branca

354. Um experimento consiste em perguntar a 3 mulheres se elas usam ou não o sabo­


nete marca A.
a) Dê um espaço amostrai para o experimento.
b) Descreva o evento A : no máximo duas mulheres usam o sabonete marca X.

V. Freqüência relativa

84. Num experim ento aleató rio , em bora n ão saibam os qu al o evento que
irá o co rrer, sabem os que alguns eventos ocorrem freqüentem ente e outros, ra­
ram ente. D esejam o s, en tão , asso ciar aos eventos n ú m eros que nos dêem uma
in d ica ç ã o qu an titativ a da o co rrên cia dos m esm os, quando o experim ento é re­
petido m uitas vezes, nas m esm as cond ições. P a ra isso, vam os definir fr e q ü ê n ­
cia relativ a d e um ev en to.

%
PROBABILIDADE

85. C onsiderem os um experim ento aleató rio com espaço am ostrai Í2, fin i­
to , isto é, ü = j a„ a 2, a k \. Su ponham os que o experim ento seja repetido
N vezes, nas m esmas cond ições. S e ja ni o núm ero de vezes que o co rre o evefi-
to elem entar a r D efinim os fr e q u ê n c ia relativa d o even to [a,\ com o sendo o nú­
m ero f , tal que

fi = Vi e |1 , 2 , . . . . k).
N

P or exem plo, se lançarm os um dado 100 vezes ( N = 100) e observarm os o nú­


m ero 2 (evento 2 ) 18 vezes, então a freqü ência relativa desse evento elem entar
será:
18
f2 0 ,1 8 .
100

A freqüência relativa possui as seguintes propriedades:

a) 0 ^ fj ^ 1 v i, pois 0 ^ ^ 1.
N

b) f, + f2 + ... + fk = 1, pois

+ n‘ - n, + n2 + .. . + nk _ N
n‘ + "2 +
N N " N N N
c) Se A é um evento de Í2 (A 5* 0 ) , a freqü ên cia relativa do evento A
( / a ) é o núm ero de vezes que o co rre A , dividido por N . É claro que:

- Ee A T -
a, a, G A

P o r exem plo, se A = j a„ a 3, a 5J, então:


\
n, + n3 + il5
fA = f. + f, + fs.
N

d) V erifica-se ex p erim en ta lm en te que a freqü ência relativa tende a se


“ estab ilizar” em to rn o de algum valor bem d efinid o, quando o núm ero N de
repetições do experim ento é su ficientem ente grande.

VI. Definição de probabilidade

86. J á vimos que a freqüência relativa nos dá uma in fo rm ação qu an titativ a


da o co rrên cia de um evento, quando o experim ento é realizado um grande nú­
mero de vezes. O que irem os fazer é d efinir um n ú m ero a s s o c ia d o a c a d a ev en ­

97
PROBABILIDADE

to, de m odo que ele tenha as m esm as características da freqü ên cia relativa. É
claro que d esejam os que a freqü ência relativa do evento esteja “ p ró xim a” des­
se número , quando o experim ento é repetido m uitas vezes. A esse núm ero da­
rem os o nom e de probabilidade do evento considerado.

87. C onsiderem os então um espaço am ostrai fin ito ti = [a„ a2, . .. , ak\. A
cad a evento elementar [a,\ vam os associar um número real, indicado por
/?((#,]) ou p h cham ad o probabilidade do evento ja ,}, satisfazend o as seguintes
cond ições:
® o < Pi < 1 V i G (1, 2 , kj
k

® E Pi = Pi + Pz + — + Pk = i-
i=1
D izem os que os núm eros p ,t p 2, ..., p k definem uma distribuição de
probabilidades sôbre ti.
E m seguida, seja A um evento qualquer de ti. D efinim os probabilidade
do evento A (e indicam os por P (A )) da seguinte form a:
a) Se A = 0 , P (A ) = 0

b) Se A * 0 , P (A ) = £ Pi-

Isto é, a probabilidade de um evento constituíd o por um certo núm ero


de elem entos é a som a das probabilidades dos resultados individuais que con s­
tituem o evento A .

88. Exemplo

= (a,, a2, a 3, a 4].


C onsiderando a d istribu ição de probabilidades:
p, = 0,1 p 2 = 0 ,3 p 3 = 0 ,2 p 4 = 0 ,4
S e ja o evento A = [a,, a2, a4J; en tão , por d efin ição:
P (A ) = Pi + p 2 + p 4 = 0,1 -H 0 ,3 -E 0 ,4 = 0 ,8 .

89. Observação

M ostram os, acim a, co m o se pode calcu lar a probabilidade de um even­


to A ( P { A )) quando é dada um a d istribu ição de probabilidades sobre ti. Surge
então a pergunta: Q ue critérios usam os para ob ter os núm eros p„ p 2f . . . , /?*?

98
\

PROBABILIDADE

Pod em os responder dizendo inicialm ente que, do ponto de vista form al,
quaisquer valores p „ p 2, . . . , p k que satisfazem :

I) o ^ Pi < 1 Vi G [1, 2 , . . . . k)
k

H) E pi = i
i = I

constituem uma d istribu ição de probabilidades sobre Í2. P o r o u tro lad o, para
serm os realistas , devemos fazer com que cada núm ero p, esteja “ p ró xim o ” da
freqü ência relativa f , quando o experim ento é repetido m uitas vezes.
Isso pode ser feito levantando-se hipóteses a respeito do experim ento,
co m o por exem plo consid erações de sim etria; é claro que nessas hipóteses são
fundamentais a experiên cia e o b o m sen so de quem vai atribuir as probabilidades
aos eventos elem entares. N enhum a pessoa de bom senso diria que a p ro b ab ili­
dade de observarm os uma bola verm elha é igual a de observ arm os uma bola
b ran ca, quando extraím os uma bola de uma urna contend o 9 bolas verm elhas
e um a b ran ca. P o r outro lad o, se faltam hipóteses para uma conveniente esco ­
lha de uma distribuição, recorre-se então à experim entação para avaliar os p- s
através da freqüência relativa.

90. Exemplos

1?) U m a m oeda é lançad a e é observada a face de cim a.


T em os:

Í2 = {K, CJ
t t
Pi P2

U m a d istribu ição razoável para 0 seria:

Isso significa que adm itim os que a freqüência relativa de caras e de co-
roas é próxim a de — quando a m oeda é lançad a muitas vezes.

Experiências históricas foram feitas por B u ffo n , que lançou uma m oe­
da 4 048 vezes e observou o resultado cara 2 0 4 8 vezes (freqü ên cia relativa de
caras: 2 048 = 0,5059).
4 048
2 ?) Um dado é lançad o e é observado o núm ero da face de cim a.
T em os:

n = |i, 2, 3, 4, 5, 6]
t t t t t t
Pi P; Pj Pj Ps P<,

99
PROBABILIDADE

U m a atribu ição razoável para p„ p» p3, p4, p5 e p6 (p o r razões de si­


m etria) é:
1
Pi — P2 — P3 ~ P4 — Ps — Pó — g •

Nesse caso , a probabilidad e de o co rrên cia de um núm ero ím par (A =


= [1, 3, 5 ]) será:

P (A ) = p, + p 3 + ps = -|- * y .

3?) S e ja ü = \a„ a 2, a„ a 4}.


Se p 4 = 4p „ p 3 = 3 p , e p 2 = 2 p ,, qual a probabilidade do evento
A [a„ cr,)?
T em os:
p, + p 2 + p 3 + p 4 = 1
p, + 2p, + 3p, + 4p, = 1

10 p, = 1 p, = - y .

4
L o go : e p4 =
10
1 4
P o rta n to , P (A ) = Pi + p 4 = +
10 10 2 *

VII. Teoremas sobre probabilidades em


espaço amostrai finito
91. Teorema 1

“ A probabilidad e do evento certo é /.”

D em o n stra çã o
D e fa to , o evento certo é Í2 = [a„ a 2í . .. , a k] e por d efin ição :
P (fl) = p, + p 2 + ... + pk = 1.

92. Teorema 2

“ Se A C B , en tão P ( A ) < P ( B ) . ”

100

.1
PROBABILIDADE

D em o n stra çã o
1) Se A = B , por d efinição P ( A ) = P ( B ) e p ortan to P ( A ) ^ P (B J.

2) Se A £ fl.

S e ja A = [a,, a 2, arJ
e B — [a,, a2,ar, a r+), a r+qJ
en tão :
P (A ) = p, + p 2 + ... + pr
P (B ) = P, + P 2 + ... + Pr+ Pr+, + — + Pr+q.
C om o :
p ,, p2, pr, pr+q são tod os não negativos, d ecorre que:
P (A ) ^ P (B ).
No caso particular de A = 0 , tem os P M ) = 0 e P ( B ) ^ 0, e p ortan to
P ( ,4 ) ^ P ( P ) .

93. Teorema 3

“ Se ^4 é um evento, en tão 0 < P M ) < 7 .”

D em o n stra çã o
0 C A C Ü

L o g o , pelo teorem a 2:
P ( 0 ) < P (A ) ^ P(Q) e p ortanto 0 ^ P (A ) < 1.

94. Teorema 4
“ Se /I e são eventos, então P M U 2?) = P M ) + P ( B ) - P ( A f l £ ) . ”

D em o n stra çã o

P (A U B ) = Yj Pi-
a( G A U B

P o r o u tro lad o , P (A ) = pj e P (B ) = X Pj*


ai e A a. E B

101
PROBABILIDADE

O ra, quando som am os P ( A ) + P ( B ) as probabilidades dos eventos ele­


m entares contid os em A f l B são com putadas duas vezes (u m a, por estarem
em A e o u tra, por estarem em B ).
P o rtan to P ( A ) + P ( B ) — P (A f l B ) é a som a das probabilidades dos
eventos elem entares contid os em A U B , logo:
P (A U B ) = P (A ) + P (B ) - P (A f l B ).

# 95. Observações
a) Em particu lar, se A e B são m utuam ente exclusivos (A f l B = 0 ) ,
en tão
P (A U B ) = P (A ) + P (B ) - P (0 )

= P (A ) + P (B ).

b) O resultado an terio r pode ser generalizado para n eventos A „ A 2


. . . , A n m utuam ente exclusivos dois a dois, da seguinte form a:
P (A , U A 2 U . .. U A n) = P (A ,) + P (A 2) + ... + P (A n).

96. Teorema 5

“ Se A é um evento, então P (A C) = 1 — P ( A ) . ”

D em o n stra çã o
C om o A í l A c = 0 e A U A c = ü
d ecorre pelo teorem a 4 que
P (A U A °) = P (A ) + P (A C)
logo:
1 = P (A ) + P (A C) P (A C) = 1 - P (A ).

97. Exemplo de aplicação dos teoremas


U m a urna contém 100 bolinhas num eradas, de / a 100. U m a bolinha
é escolhida e observado seu núm ero. A dm itindo probabilidades iguais a — —
para tod os os eventos elem entares, qual a probabilidade de:

102
PROBABILIDADE

a) observarm os um m últiplo de 6 e de 8 sim ultaneam ente?


b) observarm os um m últiplo de 6 ou de 81
c) observarm os um núm ero não m últiplo de 5?

T em os:
0 = (1, 2 , 3 ......... 9 9 , 100]

a) Um m últiplo de 6 e 8 sim ultaneam ente terá que ser m últiplo de 24;


p o rta n to , o evento que nos interessa é: A = [24, 48, 72, 96}.

1 4 _ J _
P (A ) = +
100 100 100 25 ‘

b) S e ja m os eventos:

B : o núm ero é m últiplo de 6 . C : o núm ero é m últiplo de 8 .


O evento que nos interessa é B U C , en tão :
B = (6 , 12, 18, 2 4 , 30, 3 6 , 4 2 , 4 8 , 54, 6 0 , 66, 7 2 , 7 8, 8 4 , 9 0 , 96]
16
e P (B ) =
100 25 '
C = [8 , 16, 2 4 , 32, 4 0 , 4 8 , 56, 6 4 , 72, 8 0 , 88, 96)
e
p'c > - i k - w y , '
P o rta n to : P (B U C ) = P (B ) + P (C ) - P (B f l C ).
O ra , f l í l C nada m ais é do que o evento A (do item a).

L o g o , P (B n c ) = ^ .

Segue-se então que: P (B U C ) = — + ~ --------- — =


25 25 25 25

c) S e ja D o evento, o núm ero é m últiplo de 5.

T em os:
D = [5, 10, 15, 20, 25, 30, 35, 4 0 , 4 5 , 50, 5 5, 6 0 , 6 5 , 7 0 , 7 5 , 8 0, 85
90, 95, 100)
20 = J _
P (D ) =
100 5 '
O evento que nos interessa é D c . L o go , PCD0) = 1 - P ( D ) = 1 — j - = - j - .

103
PROBABILIDADE

EXERCÍCIOS
355. Numa urna existem duas bolas vermelhas e seis brancas. Sorteando-se uma bola,
qual a probabilidade de ela ser vermelha?

356. Numa cidade com 1 000 eleitores vai haver uma eleição com dois candidatos, A
e B. É feita uma prévia em que os 1 000 eleitores são consultados, sendo que 510
já se decidiram, definitivamente, por A. Qual é a probabilidade de que A ganhe
a eleição?

3 5 7 . Considere o espaço amostrai 0 = [a,, a 2, a3, a4] e a distribuição de probabili­


dades, tal que: p y = P 2 = Pi e p 4 = 0,1. Calcule:
a) P „ P 2 e p 3.
b) Seja A o evento A = [a,, a 3). Calcule P (A ).
c) Calcule P(A C).
d) Seja B o evento B = { a h a 4}. Calcule P (B ).
e) Calcule P(A U B ) e P(A O B).
f) Calcule P[(A U B )c ] e P[(A fl B )c ).

3 5 8 . Seja fí = [K, C) o espaço amostrai do lançamento de um moeda. É correta a


distribuição de probabilidades P (K ) = 0,1, P (C ) = 0,91
(Lance uma moeda 100 vezes, calcule a freqüência relativa do evento cara e veri­
fique se essa distribuição é compatível com a realidade.)

3 5 9 . Uma moeda é viciada de tal modo que sair cara é duas vezes mais provável do
que sair coroa. Calcule a probabilidade de:
a) ocorrer cara no lançamento dessa moeda;
b) ocorrer coroa no lançamento dessa moeda.

360. Temos duas moedas, das quais uma é perfeita e a outra tem duas caras. Uma
das moedas, tomada ao acaso, é lançada. Qual é a probabilidade de se obter cara?

361. Cm dado é viciado, de modo que a probabilidade de observarmos um número


na face de cima é proporcional a esse número. Calcule a probabilidade de:
a) ocorrer número par;
b) ocorrer número maior ou igual a 5.

104
PROBABILIDADE

Solução

G = [1, 2, 3, 4, 5, 6}
Temos:
P2 = 2p,
Pj = 3p,
P 4 = 4p,

Ps = 5p,
Ps = 6p,
Porém, p, + p2 + p3 + p4 + Ps + P* = !• ,
Logo, p, + 2p, + 3p, + 4p, + 5p, + 6p, = 1 => 21p, = 1 => p, = — .

a) O evento que nos interessa é A = [2, 4, 6 ],

P ( A ) . p I + p< + pl - 2 . J r + 4 . i + 6.Jr . J f . - t

b) O evento que nos inter^sa é B = (5, 6 ].

3 6 2 . Um dado é viciado de modo que a probabilidade de observarmos qualquer nú­


mero par é a mesma, e a de observarmos qualquer número ímpar é também a
mesma. Porém um número par é três vezes mais provável de ocorrer do que um
número ímpar. Lançando-se esse dado, qual a probabilidade de:
a) ocorrer um número primo?
b) ocorrer um múltiplo de 5 ?
c) ocorrer um número menor ou igual a i ?

3 6 3 . Seja o espaço amostrai G = [a,, a2, ..., a l0] e considere a distribuição de proba­
bilidades:
Pi = P(fai)) = K • i vi G (1, 2, 3, ..., 10)
a) Calcule K.
b) Calcule p 3 e p 7.
c) Seja o evento A = j a /t a2, a4, a6\. Calcule P(A ).
d) Calcule P(A C).

3 6 4 . Seja o espaço amostrai:


G = [0, 1, 2, ..., 10)
e considere a distribuição de probabilidades:

Pi = P([i)) = f 1;0) (0.6)* • (0,4)1(M Vi e (0, 1, 2, .... 10)

105
PROBABILIDADE

10
a) Mostre que = 1.
i- 0
b) Calcule p 3.
c) Seja o evento A = 10, 7, 2 j. Calcule P ( A ) e P ( A C).

365 . Se A e B são eventos quaisquer íí, prove que P( A U B) ^ P ( A ) + P( B ) .

366 . Se A e B são eventos de Í2, prove que:


P(A O B) ^ P(A) ^ P(A U B) ^ P(A) + P(B).

367 . Se A e B são eventos tais que: P ( A ) = 0,2, P ( B ) = 0,3 e P ( A fl B ) = 0,7,


calcule:
a) P(A U B) b) P(AC) c) P(BC)

368 . Se A , B e C são eventos de fí, prove que:


P ( A U B U C ) = P(A) + P(B) + P(C) - P(A O B) - P(A O C) - P(B H C ) +
+ p(A n b n c>.

369 . Se A , B e C são eventos tais que:


P(A) = 0,4, P(B) = 0,3, P(C) = 0,6, P(A H B) = P(A fl C) = P(B O C) = 0,2
e P(A fl B H C) = 0,1
calcule:
a) P(A U B) b) P(A U C) c) P(A U B U C)

VIII. Espaços amostrais equiprováveis

9 8 . S e ja Í2 = j a„ a2, . . . , ak\. D irem os que um a d istribu ição de p robabili­


dades sobre Q é eq u ip r o v á v el , se p , = p 2 = ... = pk9 isto é, se todos os even­
tos elementares de Í2 tiverem a mesma probabilidade. Em geral, as característi­
cas do experim ento é que nos levam a supor uma d istribu ição equiprovável.

99. Exemplo
De um baralh o de 52 ca rta s, um a delas é escolhida.
S e ja : Í2 = (2c, 2 o , 2e, 2p , 3 c, 3 o , 3e, 3p , A c, A o , A e , ApJ.

O s índices c, o, e9 p ind icam , respectivam ente, naipe de co p as, ouros,


espadas e paus.

106
PROBABILIDADE

É razoável supor que cad a evento elem entar tenha a m esm a p ro b ab ili­
dade. C om o temos 52 elementos em 0 , então a probabilidade de qualquer ev en ta
elem entar é:

S e ja o evento A : a carta é de copas.


E n tã o : A = j2 c, 3c, 4c, . . . , K c, A c j.
i l _L
C om o # A = 13 P (A ) = 4 r + A r +
52 52 52 4 "

S e ja o evento B : a ca rta é um rei.


E n tã o : B = {K c, K o , K e, Kpj
4_
P(B) =j2 +à +à +i 52 13

S e ja o evento C : a carta é um rei de copas.

E n tã o : c = ÍKJ
P(C) =
52

IX. Probabilidade de um evento num espaço


equiprovável

100. S e ja íí = \a„ a2, akj e uma d istribu ição equiprovável /?, =


Vi e (/. 2 . .... K\. K
S e ja A um evento, tal que:
A — {&), a 2, ...» a rJ

P (A ) = p, + p 2 + ... + pr

r vezes

P (A ) = - j —, isto é, num espaço 8 , com distribuição equiprovável.

#A
p (A ) = f
#8

107
PROBABILIDADE

101. Observação

Dado um conjunto com N elem entos, escolh er a o acaso n elementos desse


co n ju n to significa que cada su bco n ju n to (ordenado ou não) de n elem entos tem
a m esm a probabilidad e de ser escolhido.

102. Exemplo

D e um baralh o de 52 cartas, duas são extraídas ao a ca so , sem reposi­


çã o . Q ual a probabilidade de am bas serem de copas?
T em os:
C ad a par de cartas possíveis de serem extraídas pode ser considerado
co m o um a co m b in ação das 52 cartas tom adas duas a duas. Isto é,

o = f(2c, 2 J , Í2C, 2 PJ, . . . . (5C, 7e], . . . . {Ac, A J )

e nesse caso # 0 = ^ ^ = 1 326.

A é o evento (su b co n ju n to ) form ad o pelas com bin ações de cartas de co ­


pas, isto é:
A = ((2C, 3cj, (2C, 4cj, (Kc, A J)

e nesse caso # A = ^ ^ ^ = 78.

39
L o g o , P (A ) -
663 17 ’

P od eriam os ter resolvido o pro blem a, considerando Í2 co m o sendo fo r­


m ado por a rra n jo s, ao invés de co m b in açõ es, isto é:
íí = {(2 ,, 2 p); (2 p, 2 C); ( 6p) 3C); (3C, 6P); (A c> A p)j
e #Í2 = A 52i2 = 52 • 51 = 2 652
e o evento A seria form ado pelos a rra n jo s de duas cartas de co p as, isto é:
A = ((2C, 3C), (3C, 2C) .........(K c> A c), (A c, K c)j

e # A = A 13-2 = 13 • 12 = 156.

P o rta n to :
156
P (A ) =
2 652 17 '

Isto é, fi pode ser descrito co m o co n ju n to de a rra n jo s ou de com bin a­


çõ es, que a probabilidade do evento será a m esm a. N o en tan to , é im portante

108
PROBABILIDADE

observar que, se Í2 for form ado por co m b in açõ es, A tam bém terá que ser (pois
A C 12), bem co m o , se fo r fi form ad o por a rra n jo s, A tam bém o será.

Em m uitos problem as de probabilidades o co rre esse fa to , isto é, a esco ­


lha do espaço am ostrai é facu ltativa. E n tre tan to , em outros p roblem as, com o
verem os, isso não será possível.

EXERCÍCIOS
3 7 0 . De um baralho de 52 cartas, uma é extraída ao acaso. Qual a probabilidade de
cada um dos eventos abaixo?
a) Ocorre dama de copas.
b) Ocorre dama.
c) Ocorre carta de naipe paus.
d) Ocorre dama ou rei ou valete.
e) Ocorre uma carta que não é um rei.

3 7 1 . Um número é escolhido ao acaso entre os 20 inteiros, de 1 a 20. Qual a probabili­


dade de o número escolhido:
a) ser par? c) ser primo?
b) ser ímpar? d) quadrado perfeito?

3 7 2 . Um número é escolhido ao acaso entre os 100 inteiros de / a 100. Qual a probabi­


lidade de o número:
a) ser múltiplo de 91 c) ser múltiplo de 3 ou de 41
b) ser múltiplo de 3 e de 41

3 7 3 . Uma urna contém 20 bolas numeradas de 1 a 20. Seja o experimento retirada de


uma bola, e considere os eventos:
A = (a bola retirada possui um número múltiplo de 2]
B = ja bola retirada possui um número múltiplo de 5)
Determine a probabilidade do evento A U B.

3 7 4 . Os coeficientes a e b da equação ax = b são escolhidos ao acaso entre os pares


ordenados do produto cartesiano A x A , sendo A = \1, 2, 3, 4\, sendo a o 1?
elemento do par e b o 2?. Qual a probabilidade de a equação ter raízes inteiras?

109
PROBABILIDADE

375. Uma urna contém 3 bolas brancas, 2 vermelhas e 5 azuis. Uma bola é escolhida
ao acaso na urna. Qual a probabilidade de a bola escolhida ser:
a) branca? b) vermelha? c) azul?

Solução

Sejam:
B h B2, B 3 as bolas brancas
V,, V2 as bolas vermelhas
A ,, A 2» Ait A 4, A 5 as bolas azuis.
Um espaço amostrai para o experimento é:
fi = (B„ B2, B 3, V „ V2> A „ a 2, a 3, A4, A5), #Q = 10.
a) Seja o evento A: a bola extraída é branca. Então:
A = (B,, B 2, B 3J, # A = 3, logo

P(A) - - j t

b) Seja o evento B: a bola extraída é vermelha. Então:


B = (V „ V2), # B = 2, logo

P<B, - X . X.
c) Seja o evento C: a bola extraída é azul. Então:
C = [Aj, A2, A3, A4, A5), # C = 5, logo

P(C> ~ - k - T -

376. Uma urna contém 6 bolas pretas, 2 bolas brancas e 10 amarelas. Uma bola é es­
colhida ao acaso. Qual a probabilidade de:
a) a bola não ser amarela?
b) a bola ser branca ou preta?
c) a bola não ser branca, nem amarela?

377. Dois dados, um verde e um vermelho, são lançados e observados os números das
faces de cima.
a) Qual a probabilidade de ocorrerem números iguais?
b) Qual a probabilidade de ocorrerem números diferentes?
c) Qual a probabilidade de a soma dos números ser 7?
d) Qual a probabilidade de a soma dos números ser 121
e) Qual a probabilidade de a soma dos números ser menorou igual a 121
f) Qual a probabilidade de aparecer número 3 em ao menosum dado?

110
PROBABILIDADE

3 7 8 . Jogando 3 dados (ou um dado 3 vezes), qual a probabilidade de se obter soma


menor ou igual a 41
3 7 9 . Um dado especial, em forma de icosaedro, tem suas faces numeradas da seguintfe
forma: duas das faces têm o número zero; as 18 restantes têm os números
- 8, - 7 , /, 2, 9. Qual é a probabilidade de que, lançando dois
destes dados, tenhamos uma soma do número de pontos igual a 2 ?
3 8 0 . Dois indivíduos, A e B, vão jogar cara ou coroa com uma moeda “ honesta” .
Eles combinam lançar a moeda 5 vezes, e ganha o jogo aquele que ganhar em
3 ou mais lançamentos. Cada um aposta R S2 800,00. Feitos os dois primeiros
lançamentos, em ambos os quais A vence, eles resolvem encerrar o jogo. Do
ponto de vista probabilístico, de que forma devem ser repartidos os R t5 600,001
3 8 1 . Um indivíduo retrógrado guarda seu dinheiro em um açucareiro. Este contém 2
notas de R$50,00, 4 de R $ 1 0 ,0 0 , 5 de R $ 5 ,0 0 , 8 de R$ 1,00 e i d e R $0,50.
Se o indivíduo retira do açucareiro duas notas simultaneamente e ao acaso,
qual é a probabilidade de que ambas sejam de R $5,001
3 8 2 . Numa cidade, 30% dos homens são casados, 40% são solteiros, 20% são desqui-
tados e 10% são viúvos. Um homem é escolhido ao acaso.
a) Qual a probabilidade de ele ser solteiro?
b) Qual a probabilidade de ele não ser casado?
c) Qual a probabilidade de ele ser solteiro ou desquitado?
3 8 3 . Em uma sala existem 5 crianças: uma brasileira, uma italiana, uma japonesa, uma
inglesa e uma francesa. Em uma urna existem 5 bandeiras correspondentes aos
países de origem dessas crianças: Brasil, Itália, Japão, Inglaterra e França. Uma
criança e uma bandeira são selecionadas ao acaso, respectivamente, da sala e da
uma. Determine a probabilidade de a criança sorteada não receber a sua bandeira.

3 8 4 . Em um grupo de 500 estudantes, 80 estudam Engenharia, 150 estudam Econo­


mia e 10 estudam Engenharia e Economia. Se um aluno é escolhido ao acaso,
qual a probabilidade de que:
a) ele estude Economia e Engenharifc?
b) ele estude somente Engenharia?
c) ele estude somente Economia?
d) ele não estude Engenharia nem Economia?
e) ele estude Engenharia ou Economia?

Solução

Sejam os eventos:
A: o aluno estuda Engenharia.
B: o aluno estuda Economia.
O diagrama ao lado permite responder
facilmente às perguntas.

111
PROBABILIDADE

É fácil perceber que 280 alunos não estudam Engenharia nem Economia:
(500 - 70 - 10 - 140 = 280).

10 1 280 14
d)
500 50 500 25

70 7 220 11
e)
500 50 500 25

140 = _7_
500 25

3 8 5 . De um grupo de 200 pessoas, 160 têm fator Rh positivo, 100 têm sangue tipo O
e 80 têm fator Rh positivo e sangue tipo O. Se uma dessas pessoas for selecionada
ao acaso, qual a probabilidade de:
a) seu sangue ter fator Rh positivo?
b) seu sangue não ser tipo O?
c) seu sangue ter fator Rh positivo ou ser tipo O?

3 8 6 . Uma cidade tem 50000 habitantes e 3 jornais, A, B, C. Sabe-se que:


15 000lêem o jarnal A
10 000lêem o jornal B
8 000lêem o jornal C
6 000lêem os jornais A e B
4 000lêem os jornais A e C
3 000lêem os jornais B e C
1000 lêem os três jornais.
Uma pessoa é selecionada ao acaso. Qual a probabilidade de que:
a) ela leia pelo menos um jornal?
b) leia só um jornal?

3 8 7 . Um colégio tem 1 000 alunos. Destes:


200 estudam Matemática
180 estudam Física
200 estudam Química
20 estudam Matemática, Física e Química
50 estudam Física'e Química
70 estudam somente Química
50 estudam Matemática e Física.
Um aluno do colégio é escolhido ao acaso. Qual a probabilidade de:
a) ele estudar só Matemática?
b) ele estudar só Física?
c) ele estudar Matemática e Química?

112
PROBABILIDADE

388. Uma moeda é lançada 3 vezes. Qual a probabilidade de:


a) observarmos três coroas?
b) observarmos exatamenté uma coroa?
c) observarmos pelo menos uma cara?
d) não observarmos nenhuma coroa?
e) observarmos no máximo duas caras?

389. Lançando 4 vezes uma moeda “ honesta” , qual é a probabilidade de que ocorra
cara exatamente 3 vezes?

3 9 0 . Tirando, ao acaso, 5 cartas de um baralho de 52 cartas, qual é a probabilidade


de saírem exatamente 3 valetes?

3 9 1 . Com os dígitos 1, 2, 3, 4, 5 são formados números de 4 algarismos distintos. Um


deles é escolhido ao acaso. Qual a probabilidade de ele ser:
a) par? b) ímpar?

Solução

Seja 0 o conjunto dos números de 4 algarismos distintos formados com


os dígitos /, 2, 3, 4, 5. Então:

# í í = A 5 4 = - p - = 120.

a) Seja B o evento, o número escolhido é par. Então:

III4

b) Seja C o evento, o número é ímpar. Como C = B c , decorre que:

P(C) = i - P(B) = i - j - = - L

3 9 2 . Em uma uma existem 6 bolinhas numeradas, de I a 6. Uma a uma elas são extraí­
das, sem reposição. Qual a probabilidade de que a sequência de números obser­
vados seja crescente?

3 9 3 . Uma urna contém bolas numeradas de 1 a 9. Sorteiam-se, com reposição, duas


bolas. Qual é a probabilidade de que o número da segunda bola seja estritamente
maior do que o da primeira?

3 9 4 . Numa urna são depositadas n etiquetas numeradas de 1 a n. Três etiquetas são


sorteadas (sem reposição). Qual a probabilidade de que os números sorteados se­
jam consecutivos.

113
PROBABILIDADE

395. Oito pessoas (entre elas Pedro e Sílvia) são dispostas ao acaso em uma fila. Qual
a probabilidade de:
a) Pedro e Sílvia ficarem juntos?
b) Pedro e Sílvia ficarem separados?

396. Nove livros são colocados ao caso numa estante. Qual a probabilidade de que
3 livros determinados fiquem juntos?

397. Uma loteria consta de 1 000 números, de 1 a 1 000. Dez números são sorteados
ao acaso, sem reposição, e ao 1 ? número sorteado corresponde o 1 ? prêmio, ao
2 ? número sorteado, o 2 ? prêmio, e assim por diante, até o 10? número sorteado.
Se uma pessoa é portadora do bilhete n? 341 , qual a probabilidade de ela ganhar:
a) o 1? prêmio? b) o 4? prêmio? c) o 10? prêmio?

398. Uma moeda é lançada 10 vezes. Qual a probabilidade de observarmos 5 caras


e 5 coroas?

399. Um adivinho diz ser capaz de ler o pensamento de outra pessoa. É feita a seguin­
te experiência: seis cartas (numeradas de / a 6 ) são dadas à pessoa, que concentra
sua atenção em duas delas. O adivinho terá que descobrir essas duas cartas. Se
o adivinho estiver apenas ^chutando” , qual a probabilidade de ele acertar as duas
cartas nas quais a outra pessoa concentra a atenção?

400. (Problema clássico do aniversário.)


Em um grupo de n pessoas, qual a probabilidade de que pelo menos duas façam
aniversário no mesmo dia? (Supondo que nenhuma tenha nascido em ano bissexto.)

Solução

Sejam os eventos:
A: pelo menos duas entre as n pessoas fazem aniversário no mesmo dia.
A c : todas as n pessoas fazem aniversário em dias distintos.
Cada data de aniversário pode ser considerada como um número entre 1
e 365 (inclusive). Logo, o espaço amostrai é constituído de todas as w-uplas
ordenadas em que cada elemento pode ser um inteiro de 1 a 365 (inclusive).
Logo, pelo princípio fundam ental d a contagem :
#Q = 365”.
O evento A c consiste em todas as enuplas ordenadas, de elementos distin­
tos , em que cada elemento pode ser um inteiro de 1 a 365. Logo,
#A C = ^365. n 365 • 364 • 363 • ... • (365 - n + 1).
#AC 365 - 364 - 363 - ... • (365 - n -l- 1)
Logo, PÍA0) =
# fí 365”

114
PROBABILIDADE

Portanto, P(A) = 1 - P(AC) = 1 - 365 ■364 ♦ 363 - . . . •(365— n_f 1)


7 365n

Eis os valores de P (A) para alguns valores de n.

n = 20, P(A) = 0,41


n = 40, P(A) = 0,89
n = 50, P(A) = 0,97 (quase certeza).

4 0 1 . Uma urna contém seis bolinhas numeradas de / a 6. Quatro bolinhas sào extraí­
das ao acaso sucessivamente, com reposição. Qual a probabilidade de que todas
assinalem números diferentes?

4 0 2 . Cinco algarismos são escolhidos ao acaso, com reposição, entre os algarismos 0,


l y 2, 3, 4, 5y 6y 7, 8y 9. Qual a probabilidade de os cinco algarismos serem dife­
rentes?

4 0 3 . Uma urna contém 5 bolas vermelhas e 3 brancas. Duas bolas são extraídas ao
acaso, com reposição, qual a probabilidade de:
a) ambas serem vermelhas? b) ambas serem brancas?

4 0 4 . Uma urna contém 5 bolas vermelhas, 3 brancas e 2 pretas. Duas bolas são extraí­
das ao acaso, e com reposição. Qual a probabilidade de:
a) ambas serem vermelhas? c) nenhuma ser preta?
b) nenhuma ser branca?
u
x•
4 0 5 . De um baralho de 52 cartas, três são extraídas sucessivamente ao acaso, sem re­
posição. Qual a probabilidade de que as cartas sejam de “ paus” ?

4 0 6 . De um baralho de 52 cartas, duas s^o extraídas ao acaso e sem reposição. Qual


a probabilidade de observarmos: \
a) dois ases? b) um ás e um rei (sem levar em conta a ordem)?

4 0 7 . Uma urna contém 5 bolas vermelhas e 7 brancas. Duas bolas são extraídas suces­
sivamente ao acaso e sem reposição. Qual a probabilidade de:
a) ambas serem brancas?
b) ambas serem vermelhas?
c) uma vermelha, outra branca (sem levar em conta a ordem)?

4 0 8 . De um lote de 200 peças, sendo 180 boas e 20 defeituosas, 10 peças são seleciona­
das ao acaso, sem reposição. Qual a probabilidade de:
a) as 10 peças serem boas?
b) as 10 peças serem defeituosas?
c) 5 peças serem boas e 5 serem defeituosas?

115
PROBABILIDADE

4 0 9 . u m lote contém 60 lâmpadas, sendo 50 boas e 10 defeituosas. 5 lâmpadas são


escolhidas ao acaso, sem reposição. Qual a probabilidade de:
a) todas serem boas?
b) todas serem defeituosas?
c) 2 serem boas e 3 defeituosas?
d) pelo menos uma ser defeituosa?

410 . Numa gaveta há 10 pares distintos de meias, mas ambos os pés de um dos pares
estão rasgados. Tirando da gaveta um pé de meia por vez, ao acaso, qual a pro­
babilidade de saírem dois pés de meia do mesmo par, não rasgados, fazendo duas
retiradas?

411 . Em uma loja existem 100 camisas, sendo 80 da marca A. Se 5 camisas forem es­
colhidas ao acaso, sem reposição, qual a probabilidade de 4 serem da marca A l

412 . De um baralho de 52 cartas, 5 são extraídas ao acaso, sem reposição. Qual a pro­
babilidade de:
a) saírem os 4 reis?
b) não sair nenhum rei?
c) sair ao menos um rei?

413 . De um baralho de 52 cartas, duas são extraídas ao acaso e sem reposição. Qual
a probabilidade de que pelo menos uma seja de copas?

414 . De um grupo de 10 pessoas, entre elas Regina, cinco são escolhidas ao acaso e
sem reposição. Qual a probabilidade de que Regina compareça entre as cinco?

415 . De 100 000 declarações de imposto de renda (entre as quais a do sr. K) que che­
gam a um órgão fiscal, 10000 são escolhidas ao acaso e analisadas detalhada­
mente. Qual a probabilidade de a declaração do sr. K ser analisada detalha­
damente?

416 . Entre 100 pessoas, uma única é portadora de uma moléstia. 10 pessoas entre as
100 são escolhidas ao acaso. Qual a probabilidade de a pessoa portadora da mo­
léstia estar entre as 101

417 . EJm grupo é constituído de 6 homens e 4 mulheres. Três pessoas são selecionadas
ao acaso, sem reposição. Qual a probabilidade de que ao menos duas sejam
homens?

Solução

Consideremos o espaço amostrai Q constituído de todas as combinações


das 10 pessoas, tomadas 3 a 3. Logo,

116
PROBABILIDADE

O evento A que nos interessa é formado por todas as combinações de Q,


tais que em cada uma existem dois ou três homens. Isto é:

#A = + = 80.

80 2_
Logo P(A) =
120 3 *

418 . Entre 10 meninas, 4 têm olhos azuis. Três meninas são escolhidas ao acaso, sem
reposição. Qual a probabilidade de pelo menos duas terem olhos azuis?

419 . Uma urna contém 4 bolas brancas, 2 vermelhas e 3 azuis. Cinco bolas são selecio­
nadas ao acaso, sem reposição. Qual a probabilidade de que 2 sejam brancas,
uma vermelha e 2 azuis?

420 . De um baralho de 52 cartas, 3 são extraídas ao acaso, sem reposição. Qual a pro­
babilidade de que as 3 sejam do mesmo naipe?

Solução

Seja o espaço amostrai fí constituído das combinações das 52 cartas toma­


das i a i . Então:

# fi = j 532j = 22 100.

O evento A que nos interessa é formado por todas as combinações de Q,


nas quais as 3 cartas são do mesmo naipe. Logo,

# A = 4 •í = 1 144.

1 144 22
Portanto, P(A) =
22 100 425 *
1

421 . De um baralho de 52 cartas, duas são selecionadas ao acaso e sem reposição. Qual
a probabilidade de que seus naipes sejam diferentes?

422 . De um baralho de 52 cartas, duas são escolhidas ao acaso e sem reposição. Qual
a probabilidade de observarmos dois reis ou duas cartas de copas?

423 . Um grupo é constituído de 10 pessoas, entre elas Jonas e César. O grupo é dis­
posto ao acaso em uma fila. Qual a probabilidade de que haja exatamente 4 pes­
soas entre Jonas e César.

424 . Um homem encontra-se na origem de um sistema cartesiano ortogonal. Ele só


pode andar uma unidade de cada vez, para cima ou para a direita. Se ele andar
10 unidades, qual a probabilidade de chegar no ponto P(7, J ) ?

117
PROBABILIDADE

X. Probabilidade condicional

1 0 3 . S e ja íl um espaço am ostrai e considerem os dois eventos, A e B . C om


o sím bolo P (A \ B ) indicam os a probabilidad e do evento A , d ado que o evento
B o co rreu , isto é, P (A \ B ) é a p r o b a b ilid a d e c o n d ic io n a l d o ev en to A , u m a vez
q u e B ten h a o c o r r id o . Q uando calcu lam os P (A \ B ), tudo se passa com o se B
fosse o novo espaço am ostrai “ reduzido” dentro do qual querem os calcu lar
a probabilidad e de A .

104. Exemplos
1?) C onsiderem os o lançam ento de um dado e observ ação da face de
cim a.

íi = (1, 2, 3, 4 , 5, 6}

S e ja m os eventos:
A : o co rre um núm ero ím par
B : o co rre um núm ero m aior ou igual a 2
B = [2, 3, 4 , 5, 6)

P (A \ B ) será en tão a p robabilidad e de oco rrer núm ero ím par no novo espaço
am o strai reduzido.
B = (2, 3, 4 , 5, 6)

A tribu ind o - j - para a p robabilidad e de cad a evento elem entar de B , o

evento o co rrer núm ero ím par no espaço am ostrai “ reduzido” será [3, 5 ] e
p o rtan to :

P (A IB )

2 ? ) Numa cidade, 400 pessoas foram classificad as, segundo sexo e esta­
do civil, de aco rd o com a tabela:

solteiro casado desquitado viúvo


(S) (C ) (D ) (V)

M asculino (M ) 50 60 40 30

Fem inino (F) 150 40 10 20

200 100 50 50

118
PROBABILIDADE

U m a pessoa é escolhida ao acaso. S ejam os eventos:


S : a pessoa é solteira,
M : a pessoa é do sexo m asculino.
P(S\M ) significa a probabilidad e de a pessoa ser solteira, no novo esp a­
ço am ostrai reduzido das 180 pessoas do sexo m asculino. O ra , co m o existem
50 solteiros nesse novo espaço am ostrai:
50 _5_
P (S IM )
180 18 '

Sejam ainda os eventos:

F : a pessoa escolhida é do sexo fem inino


D: a pessoa escolhida é desquitada
en tã o , P(F\D ) significa a probabilidade de a pessoa escolhida ser do sexo fe­
m inino, no novo espaço am ostrai reduzido das 50 pessoas desquitadas. O ra,
co m o existem 10 pessoas do sexo fem inino nesse novo espaço am ostrai,

P (F ID ) = —
50 5 *

Notemos que F\F\D) ^ P(D \F), pois um cálculo simples nos m ostra que:
1
P (D IF ) =
' 220 22 *

105. Observação

P ara d efinirm os form alm ente P(A I B ) y vam os recorrer novam ente ao
co n ceito de freqüência relativa.
Se um experim ento aleató rio fo r repetido N vezes, sejam nAf nB e
nA n B o núm ero de vezes que ocorrem A , B e A P) B , respectivam ente. N o­
tem os que a freqü ência relativa de A y naqueles resultados em que B o co rre , é

nA n— , isto é, a freqü ência relativa de A cond icionad a a o co rrên cia de B


nH

nAn b

nAn b _ N _ Ía n b

nB nB f*B
N

em que f A r b z / b representam as freqüências relativas da ocorrência de A f l B


e de B y respectivamente. Q uando N é grande, f A n # é “ próxim a” de P(A D B )

119
PROBABILIDADE

e / , é próxim a de P ( B ) . Isto sugere então a d efin ição :

p (a n B)
P(AIB) P(B) > 0.
P(B)

Em resum o, tem os dois m odos de calcu lar P(A IB ):

1 ?) C onsiderando que a p robabilidad e do evento A será calcu lad a em relação


ao espaço am ostrai “ reduzido” B .

2°) E m pregando a fórm u la:


P (A n b >
P (A IB ) =
P (B )
em que tan to P(A D B ) co m o P ( B ) são calculadas em relação ao espaço
am ostrai original ü.

106. Exemplo

D ois dados d , e d 2 são lançad os. C onsiderem os o espaço am ostrai:

f d, D (2 , 1) (3, 1) (4 , 1) (5 , 1) (6, D
( 1, 2) (2, 2) (3 , 2) (4 , 2) (5 , 2) (6, 2)
0 , 3) (2 , 3) (3, 3) (4 , 3) (5, 3) (6, 3)
d , 4) (2 , 4) (3, 4) (4 , 4) (5, 4) (6, 4)
d , 5) (2 , 5) (3, 5) (4 , 5) (5 , 5) (6, 5)
[ i U 6) (2 , 6) (3, 6) (4 , 6) (5 , 6) (6, 6)

S ejam os eventos:
A : o dado d , apresenta resultado 2 ,
B : a som a dos pontos nos dois dados é 6 .
C alculem os P(A IB ) .

1 ? m o d o : O novo espaço am o strai reduzido é:


B = f ( l , 5), (2, 4 ), (3 , 3), (4 , 2 ), (5 , 1)}.
Nesse novo espaço am o strai, a probabilidad e de A (d, apresentar o re-
sau ltad o 2 ) é - y . L o g o :

P(AIB) = -t-.

120
PROBABILIDADE

2 ? m odo:
P (A n B)
P (A IB ) =
P (B )

T em os:

A = ((2, 1), (2, 2), (2, 3), (2, 4 ), (2 , 5), (2, 6)) P (A ) =
6 '
B = [(1, 5), (2 , 4 ), (3 , 3 ), (4, 2 ), (5, 1)) P (B ) =

A f l B = ((2, 4 )j P (A n B ) = l .

L o g o : P (A IB ) = = y .

^6

EXERCÍCIOS
4 2 5 . Um dado é lançado e o número da face de cima é observado.
a) Se o resultado obtido for par, qual a probabilidade de ele ser maior ou igual a 5 ?
b) Se o resultado obtido for maior ou igual a 5, qual a probabilidade de ele ser par?
c) Se o resultado obtido for ímpar, qual a probabilidade de ele ser menor que 3 1
d) Se o resultado obtido for menor que 3 , qual a probabilidade de ele ser ímpar?

4 2 6 . Um número é sorteado ao acaso entre os 100 inteiros de 1 a 100.


a) Qual a probabilidade de o número ser par?
b) Qual a probabilidade de o número ser par, dado que ele é menor que 501
c) Qual a probabilidade de o número ser divisível por 5, dado que é par?

4 2 7 . Dois dados d , e d 2 são lançados.


a) Qual a probabilidade de a soma dos pontos ser 6, se a face observada em d,
foi 21
b) Qual a probabilidade de o dado d , apresentar face 2, se a soma dos pontos
foi 61
c) Qual a probabilidade de a soma dos pontos ser menor que 7, sabendo que em
ao menos um dado apareceu o resultado 21
d) Qual a probabilidade de a soma dos pontos ser menor ou igual a 5, se a soma
dos pontos nos dois dados foi menor ou igual a 41
e) Qual a probabilidade de o máximo dos números observados ser 5, se a soma
dos pontos foi menor ou igual a 91

121
PROBABILIDADE

4 2 8 . Considere um tetraedro, como um dado, com 4 faces numeradas de 1 a 4. Dois


tetraedros tj e t2 são lançados sobre um plano e observam-se os números das fa­
ces nas quais se apóiam os tetraedros. Se a soma dos pontos obtidos for maior
que 5, qual a probabilidade de que o número observado em t, seja:
a) 4? b) 3?

4 2 9 . Um grupo de 50 moças é classificado de acordo com a cor dos cabelos, e dos


olhos de cada moça, segundo a tabela

cabelos olhos

azuis > castanhos

loira 17 9

morena 4 14

ruiva 3 3

a) Se você marca um encontro com uma dessas garotas, escolhida ao acaso, qual
a probabilidade de ela ser:
1) loira? 2) morena de olhos azuis? 3) morena ou ter olhos azuis?
b) Está chovendo quando você encontra a garota. Seus cabelos estão còmpleta-
mente cobertos, mas você percebe que ela tem olhos castanhos. Qual a proba­
bilidade de que ela seja morena?

4 3 0 . De um total de 100 alunos que se destinam aos cursos de Matemática, Física e


Química sabe-se que:
1. 30 destinam-se à Matemática e, destes, 20 são do sexo masculino.
II. O total de alunos do sexo masculino é 50 , dos quais 10 destinam-se à Química.
III. Existem 10 moças que se destinam ao curso de Química.
Nessas condições, sorteando um aluno ao acaso do grupo total e sabendo que
é do sexo feminino, qual é a probabilidade de que ele se destine ao curso de Mate­
mática?

4 3 1 . De um baralho de 52 cartas, uma é extraída e observa-se que seu número está


entre 4 e 10 (4 e 10 inclusive). Qual a probabilidade de que o número da carta
seja 61

4 3 2 . Uma comissão de 3 pessoas é formada escolhendo-se ao acaso entre Antônio, Be­


nedito, César, Denise e Elisabete. Se Denise não pertence à comissão, qual a pro­
babilidade de César pertencer?

122
PROBABILIDADE

433. Um prédio de três andares, com dois apartamentos por andar, tem apenas três
apartamentos ocupados. Qual é a probabilidade de que cada um dos três andares
k tenha exatamente um apartamento ocupado?

434. Se A e B são eventos e P {A ) > 0 , prove que:


a) P(A IA) = 'l
b) P(ACI A) = 0
c) Se A e B são mutuamente exclusivos, P (B IA ) = 0.
d) P(A U BI A) = 1
P(A)
e) Se A e B são mutuamente exclusivos, P(A IA U B) =
P(A) + P(B)

XI. Teorema da multiplicação

107. U m a conseqüência im portante da d efinição form al de probabilidade con­


dicional é a seguinte:

P (A n B)
P (A IB ) P (A n B) = P (B ) • P (A IB )
P (B )

p (a n B)
P (B IA ) = P (A n B) = P (A ) • P (B IA )
P (A )

Isto é, a p r o b a b ilid a d e d a o co r r ê n c ia sim u ltân ea d e d o is ev en to s


(P (A O B )) é o p r o d u to d a p r o b a b ilid a d e d e um d eles p e la p r o b a b ilid a d e d o
o u tr o , d a d o o p rim eiro .

1 0 8 . Exemplo 1

U m a u m a I contém 2 b o las verm elhas e 3 bolas b ran cas, a urna II co n ­


tém 4 bolas verm elhas e 5 bolas bran cas. U m a urna é escolhida ao acaso e dela
uma b o la é extraída ao acaso. Q ual a p robabilidad e de observ arm os urna I e
bola verm elha?

123
PROBABILIDADE

® ~ ® ® ® ®
® ® ^ ®
(x)
® ® ® ® ®

O s dados do p roblem a podem ser co lo cad o s num diagram a de árvore.


C om o cada u m a é selecionada ao acaso, a probabilidade é - y para cada um a I e II
/ , . .
(escrevem os — em cad a ram o que parte do p onto inicial para a urna obtid a).

D ada a u m a escolhida, escrevem os as probabilidades con d icion ais de


extrairm os da m esm a um a b o la de determ inada co r. T a is probabilidades são
co lo cad as nos ram os que partem de cad a urna para cad a resultado do 2? expe­
rim ento (extração da bola).

[ Uh o evento escolher u m a I
eJ am ^ [/ o evento escolher b o la verm elha.

E stam os interessados no evento U, 0 V. L o g o , pelo teorem a da mul­


tip licação :
P (U , H V ) = P (U ,) • P (V IU ,)

O ra , P (U ,) = - i - , P (V IU ,) =

1
L o g o , P (U , n V ) = y - | =
5

124
PROBABILIDADE

Isto é, a probabilidad e da o co rrên cia sim ultânea de U, e V é o produto


das probabilidades que aparecem nos ram os da árvore onde estão situ ad o s/ e V.

A nalogam ente, indicando por U„ o evento urna II e por B o evento b o ­


la b ra n ca , terem os:

1 3 3
P (U , f l B ) =
2 5 10
1 4 2
P (u „ n V) =
2 9 9
1 5 5
P (u „ n B) =
2 9 18

109. Exemplo 2

Um lote contém 50 peças boas ( B ) e 10 defeituosas (D ). U m a peça é


escolhida ao acaso e, sem reposição desta, o u tra peça é escolhida ao acaso.
O diagram a de árvore correspond ente é:
l ^

P elo d iagram a, conclu ím os que a probabilidad e de am bas serem d efei­


tuosas é:
10 9 _ 90 = 3
6 0 ’ 59 3 540 118 '

125
PROBABILIDADE

XII. Teorema da probabilidade total

110. Inicialm ente, considerem os n eventos B h B 2, . . . , B„. D irem os que eles


form am uma p artição do espaço am ostrai íí, quando:

I) P (B k) > 0 Vk
II) Bj H Bj = 0 para i ^ j
n
III) U B; = Í2
i= 1

Isto é, os eventos B „ B 2, . .. , B„ são dois a dois m utuam ente exclusivos


e exaustivos (sua união é 0 ).

111. Ilu stração para n = 11:

S e ja íi um espaço am ostrai, A um evento qualquer de Q e B ,, B 2, Bn


um a p artição de Q.
É válida a seguinte relação:
a = (B , n A) u (b 2 n a ) u (b 3 n a ) u . .. u (B n n a ).

112. A figura ab aixo ilustra o fato para n = 5.

126
PROBABILIDADE

Nesse caso:

A = ^ ( B , n A ) j U j (B2n A ) j U ( B 3n A ) ü ( B 4n A ) U ( B 5n A ) .
'------V------ J v------ v------■
0 0

N otem os que (B , D A ) ; (B 2 O A ) ...; (B n f l A ) são dois a dois m utua­


m ente exclusivos, p o rtan to :

P(A ) = P(B, n A) + p (b 2 n A ) + ... + P(Bn n A).

E ste resultado é conhecid o co m o te o r e m a d a p r o b a b ilid a d e total. Ele


é utilizado quando P ( A ) é d ifícil de ser calcu lad a diretam ente, porém simples
se for usada a relação acim a.

1 1 3 . Exemplo 1

U m a urna I tem 2 bolas verm elhas ( V ) e 3 brancas ( £ ) ; o u tra urna II


tem 3 bolas verm elhas e um a b ran ca e a urna III tem 4 b o las verm elhas e 2
bran cas. U m a urna é selecionad a ao acaso e dela é extraída um a b o la. Qual
a probabilidad e de a b o la ser verm elha?

® ® ® ® ® ® ©
® ® ®
©
©

®
® @ ®

127
PROBABILIDADE

N otem os que os eventos, U, (sair urna I), (J,, (sair urna II) e U„, (sair
urna III) determ inam uma partição de 0 . S e ja V o evento sair bola verm elha.
E n tã o , pelo teo rem a d a p r o b a b ilid a d e to ta l , P( V) = P(U , f ! V) + P (U MD V ) +
+ P(U ,n n V ).

P orém , pelo teorem a da m u ltiplicação:

_2_
P(u, n v) = - — —
3 5 15
J_
p(u„ n v) = -i- •4-
3 4 4
2_
P (u „ , n v ) = y •y
9 ‘

D ecorre então que:


109
P (V ) = — + — + —
15 4 9 180 *

1 1 4 . Exemplo 2 (P ro b lem a da m oeda de Bertrand)

Existem três caixas idênticas. A 1? contém duas m oedas de ouro, a 2?


contém uma m oeda de ouro e o u tra de prata, e a 3 ! , duas m oedas de prata.
U m a caixa é selecionada ao acaso e da mesm a é escolhida uma m oeda ao aca­
so. Se a moeda escolhida for de ouro, qual a probabilidade de que a outra moeda
da caixa escolhida tam bém seja de ouro?

É claro que o problem a pode ser form ulado da seguinte form a: “ Se a


m oeda escolhida é de o u ro , qual a probabilidade de que ela tenha vindo da
caixa I (pois a caixa I é a única que contém duas m oedas de ou ro).

128
PROBABILIDADE

S ejam os eventos:

C , : a caixa sorteada é a 1 ?
C „ : a caixa sortead a é a 2?
C //7: a caixa sortead a é a 3?
O : a m oeda sorteada é de ouro.

T em os:

P(C, n o ) = | . | = j

p(O ) = P (c , n o ) + P ( c 2 n o ) + P ( c 3 n o )
1
P(O) = 4 - • i + 4 - • 4 - + °
3 3 2 2

2_
- p (c
,io ) = P (C ' n Q) = i
V ' ' P(O) ± 3 '
2

Isto é, a probabilidade buscada

EXERCÍCIOS
______________ i______________

4 3 5 . Um juiz de futebol possui três cartões no bolso. Um é todo amarelo, outro é todo
vermelho e o terceiro é vermelho de um lado e amarelo do outro. Num determi­
nado lance, o juiz retira, ao acaso, um cartão do bolso e o mostra a um jogador.
Determine a probabilidade de a face que o juiz vê ser vermelha e de a outra face,
mostrada ao jogador, ser amarela.

4 3 6 . Uma urna I tem 3 bolas vermelhas e 4 pretas. Outra uma II tem 6 bolas verme­
lhas e 2 pretas. Uma urna é escolhida ao acaso e dela é escolhida uma bola tam­
bém ao acaso. Qual a probabilidade de observarmos:
a) uma I e bola vermelha?
b) uma I e bola preta?
c) urna II e bola vermelha?
d) urna II e bola preta?

129
PROBABILIDADE

4 3 7 . Uma urna tem 8 bolas vermelhas, 3 brancas e 4 pretas. Uma bola é escolhida ao
acaso e, sem reposição desta, outra é escolhida, também ao acaso. Qual a proba­
bilidade de:
a) a 1? bola ser vermelha e a 2? branca?
b) a 1? bola ser branca e a 2? vermelha?
c) a 1? e a 2! serem vermelhas?

4 3 8 . O mês de outubro tem 31 dias. Numa certa localidade, chove 5 dias no mês de
outubro. Qual a probabilidade de não chover nos dias 1? e 2 de outubro?

4 3 9 . Seja Px a probabilidade de que uma pessoa com X anos sobreviva mais um ano
e nPx a probabilidade de que uma pessoa com x anos sobreviva mais n anos (n
inteiro positivo).
a) O que significa P40^-
b) O que significa 2P 401
c) Mostre que 2P 40 — P 40 ' 1*41 *

4 4 0 . A urna I tem 3 bolas vermelhas e 4 brancas, a urna II tem 2 bolas vermelhas e


6 brancas e a urna III tem 5 bolas vermelhas, 2 brancas e 3 amarelas. Uma uma
é selecionada ao acaso e dela é extraída uma bola, também ao acaso. Qual a pro­
babilidade de a bola ser:
a) vermelha? b) branca? c) amarela?

4 4 1 . Uma uma contém 1 bola preta e 9 brancas. Uma segunda urna contém x bolas
pretas e as restantes brancas num total de 10 bolas. Um primeiro experimento
consiste em retirar, ao acaso, uma bola de cada uma. Num segundo experimen­
to, as bolas das duas umas são reunidas e destas, duas bolas são retiradas ao aca­
so. Qual é o valor mínimo de a: a fim de que a probabilidade de saírem duas bolas
pretas seja maior no segundo do que no primeiro experimento?

4 4 2 . Em um lote da fábrica A existem 18 peças boas e 2 defeituosas. Em outro lote


da fábrica B y existem 24 peças boas e 6 defeituosas, e em outro lote da fábrica
C, existem 38 peças boas e 2 defeituosas. Um dos 3 lotes é sorteado ao acaso e
dele é extraída uma peça ao acaso. Qual a probabilidade de a peça ser:
a) boa? b) defeituosa?

4 4 3 . Em um jogo de cara ou coroa, em cada tentativa a moeda é lançada 3 vezes con­


secutivas. Uma tentativa é considerada um sucesso se o número de vezes que se
obtém cara supera estritamente o número de vezes que se obtém coroa. Qual é
a probabilidade de serem obtidos 2 sucessos nas 2 primeiras tentativas?

4 4 4 . A urna I tem 2 bolas vermelhas e 3 amarelas e a urna II tem 4 bolas vermelhas,


5 amarelas e 2 brancas. Uma bola é escolhida ao acaso na urna I e colocada na
urna II, em seguida uma bola é escolhida na urna II ao acaso. Qual a probabili­
dade de essa segunda bola ser:
a) vermelha? b) amarela? c) branca?

130
PROBABILIDADE

4 4 5 . Sejam A e B dois eventos tais que: P(A O B ) = 0,8 e P(A fl B c ) = 0,1. Cal­
cule P(A ).

4 4 6 . Uma urna I tem 3 bolas vermelhas e 4 brancas, a urna II tem 6 bolas vermelhas
e 2 brancas. Uma urna é escolhida ao acaso e nela é escolhida uma bola, também
ao acaso.
a) Qual a probabilidade de observarmos urna I e bola vermelha?
b) Qual a probabilidade de observarmos bola vermelha?
c) Se a bola observada foi vermelha, qual a probabilidade que tenha vindo da
urna I?

Solução (diagrama da árvore)

3
V

a) P(U, n V) - i ■j - . TT
B

b> P < v > - T - T + T - T " i


V

c) Estamos interessados em P(Uj\ V). Por definição

P(U ,IV ) =
P(U, n V)^
P(V)

_3_
14 _4_
Usando os resultados dos itens a q b 9 P(U|IV) = —
11 *

“56

447. Uma caixa contém 3 moedas M/t M„ e Mm. A M, é “ honesta” , a Mu tem duas
caras e a Min é viciada de tal modo que caras são duas vezes mais prováveis que
coroas. Uma moeda é escolhida ao acaso e lançada.
a) Qual a probabilidade de observarmos moeda A/; e cara?
b) Qual a probabilidade de observarmos cara?
c) Se o resultado final foi cara, qual a probabilidade de que a moeda lançada
tenha sido A/,.

131
PROBABILIDADE

448. Duas máquinas A e B produzem peças idênticas, sendo que a produção da má­
quina A é o triplo da produção da máquina B. A máquina A produz 80% de
peças boas e a máquina B produz 90%. Uma peça é selecionada ao acaso no esto­
que e verifica-se que é boa. Qual a probabilidade de que tenha sido fabricada
pela máquina A ?

4 4 9 . Uma clínica especializada trata de 3 tipos de moléstias: X Y e Z. 50% dos que


procuram a clínica são portadores de X, 40% são portadores de Y e 10% de Z.
As probabilidades de cura, nessa clínica, são:
moléstia X: 0,8
moléstia Y: 0,9
moléstia Z: 0,95.
Um enfermo saiu curado da clínica. Qual a probabilidade de que ele sofresse da
moléstia Y1

Solução

Façamos um diagrama de árvore:

C : indica o evento “ o enfermo fica curado”


C : indica o evento “ o enfermo não Fica curado”
1? etapa: P(Y e C ) = 0,4 • 0,9 = 0,36
2? etapa: P(C) = 0,5 •0,8 + 0,4 • 0,9 + 0,1 •0,95 = 0,855
3 ? eta p a : P(Y IC) = •

Logo, P (Y IC ) = = 0,421 = 42,1% ,

que é a probabilidade procurada.

450. No exercício anterior, se o enfermo saiu curado, qual a probabilidade de que ele
sofresse:
a) da moléstia X I b) da moléstia Z?
PROBABILIDADE

4 5 1 . Uma certa moléstia A é detectada através de um exame de sangue. Entre as pes­


soas que efetivamente possuem a moléstia A, 80% delas têm a moléstia detectada
pelo exame de sangue. Entre as pessoas que não possuem a moléstia A> 5% delas
têm a moléstia detectada (erroneamente) pelo exame de sangue. Numa cidade,
2% das pessoas têm a moléstia A. Uma pessoa da cidade foi submetida ao citado
exame de sangue que a acusou como portadora da moléstia A . Qual a probabili­
dade de essa pessoa estar efetivamente atacada pela moléstia?

4 5 2 . Em uma população, o número de homens é igual ao de mulheres. 5% dos ho­


mens são daltônicos e 0,25% das mulheres são daltônicas. Uma pessoa é selecio­
nada ao acaso e verifica-se que é daltônica. Qual a probabilidade de que ela seja
mulher?

453 Dispõe-se de um mapa. Dispõe-se também de um dado com 3 faces vermelhas


e 3 faces azuis. Considerando as regras:
I. partindo do quadro 7, pode-se cami­
nhar, no sentido indicado pelas setas,
para os demais quadros, a cada lan­
çamento do dado;
II. lançando-se o dado, se sair face azul,
segue-se pela seta da direita até o qua­
dro seguinte;
III. lançando-se o dado, se sair face ver­
melha, segue-se pela seta da esquerda
até o quadro seguinte.
Determine a probabilidade de chegar ao
quadro 13 partindo do /.

XIII. Independência de dois eventos

115. D ados dois eventos A e B de um espaço am ostrai Q, direm os que A in­


d e p e n d e d e B se:

P ( A I B ) = P (A )

isto é, A independe de B se a o co rrên cia de B não afeta a p robabilidad e de A .

O bservem os que, se A in d ep en d e d e B ( P ( A ) > 0 ), en tão B in d ep en d e


d e A , pois:

P (A f l B ) = P (B ) • P ( A I B ) _ P (B ) • P (A )
P(BIA) = = P (B ).
P (A ) " P (A ) P (A )
PROBABILIDADE

116. Em resum o, se A independe de B , então B independe de A e além disso:

P (A f l B ) = P (A ) -l P( BI A)j = P (A ) • P (B )
V
P(B)

Isso sugere a d efin ição :


D o is ev en to s A e B s ã o c h a m a d o s in d ep en d en tes se
P (A f l B) = P (A ) • P (B ).

117. Exemplo 1
U m a m oeda é lançad a 3 vezes. S ejam os eventos:
A : ocorrem pelo m enos duas caras.
B : ocorrem resultados iguais nos três lançam entos.
T em os:
fl = [(K , K , K ), (K , K , C ), (K , C , K ), (K , C , C ), (C , K , K ),
(C , K , C ), (C , C , K ), (C , C , Q ) .

A = ((K , K , K ), (K , K , C ), (K , C , K ), (C , K , K )J. P (A ) = y

B = {(K . K , K ), (C , C , O ) , P (B ) = = -J-.

A n B = |(K, K , K )(, P (A D B ) = 4 -
O

L o g o , P (A f l B ) = P (A ) • P (B )
t t t
_L _L _L
8 2 4

P o rta n to A e B são independentes.

118. Observação

a) S e A e B n ã o s ã o in d ep en d en tes , eles são cham ados d ep en d en tes


b) P ro va-se que (ver ex ercício s), se A e B são independentes, então:

A e B c são independentes.
A c e B são independentes.
A c e B c são independentes.

134
PROBABILIDADE

119. Exemplo 2

Duas pessoas praticam tiro ao alvo. A probabilidad e de a 1? atingir o


1 2
alvo é P (A ) = — e a probabilidad e de a 2? atingir o alvo é P ( B ) = — .

A dm itindo A e B independentes, se os dois atiram , qual a p ro b ab ilid a­


de de:

a) am bos atingirem o alvo?


b) ao menos um atingir o alvo?

T em os:

a) P (A f l B ) = P (A ) • P (B ) = Y ■\ = \ -

b) P (A U B) = P (A ) + P (B ) - P ( A D B ) = | + | - | = | .

XIV. Independência de três ou mais eventos

1 2 0 . C onsiderem os 3 eventos A , B e C do m esm o espaço am ostrai íl. D ire­


m os que A , B e C s ã o in d ep en d en tes , se
P (A O B ) = P (A ) • P (B ) (A
P (A f l C ) = P (A ) • P (C ) ’ i v
P (B H C ) = P (B ) • P (C )
P (A f l B n C ) = P (A ) • P (B ) • P (C ).
G eneralizando, direm os que n eventos A „ A 2, . .. , A n são independen­
tes se: *

P (A j n A j) = P(A i) • P (A j) Vi, j i * j
P(A j f l Aj n Ak) = P(A j) • P (A j) • P (A k) V i, j , k, i 5* j , i ^ k, j 5* k

P(A| n a 2 n ... n A n) = p (a ,) • p (a 2)•. . . •P (A n).

121. Observação

Em geral, para m ais do que 2 eventos não precisam os v erificar todas


essas con d ições, pois do ponto de vista p rático nós a d m itim o s a in d ep en d ên cia
(basead os nas particularidades do experim ento) e usam os esse fa to para calcu ­
larm os, por exem plo, P (A , f l A 2 f l ... (T A n) co m o P ( A ,) - P ( A 2) - ... -P (A „ ).

135
PROBABILIDADE

122. Exemplo 1

U m a m oeda é lançada 10 vezes. Q ual a probabilidade de observarm os


cara nos 10 lançam entos.
S ejam os eventos:

A ,: o co rre cara no 1? lançam ento


A 2: o co rre cara no 2? lançam ento

A l0: o co rre cara no 10? lançam ento

C om o o resultado de cad a lançam ento nào é afetad o pelos o u tro s, po­


dem os adm itir A „ A 2, . .. , A ,0 co m o eventos independentes. P o rta n to ,
P (A , n a 2 n ... n a 10) = p (a ,) • p ( a 2) . . . . • p ( a io).

C om o:

P ( A ,) = P (A 2) = ... = P ( A /0) = — (a probabilidade de ocorrer

cara em qualquer lançam ento é - y

d ecorre que:

P(A| n a 2 n ... n a i0) = . JL - = ( - ) 10 = -


I Z Z 2, \2 / 1 0 24

10

123. Exemplo 2

Um dado é lançado 5 vezes. Q ual a probabilidade de que a face “ 2 ”


ap areça pelo m enos uma vez nos 5 lançam entos?
S e ja m os eventos:
A ,: o co rre um núm ero d iferen te de 2 no 1?lançam ento
A 2: o co rre um núm ero diferen te de 2 no 2? lançam ento

A 5: o co rre um núm ero diferen te de 2 no 5? lançam ento.

A dm itindo A „ A 2, . . . , A 5 independentes e tendo em co n ta que


P iA t) = V/ E [1, 2 , 3 , 4, 5 j , resulta que:
6

p (a , n a 2 n ... n a 5) =

136
PROBABILIDADE

E n tão é a p robabilidad e de n ã o o b s e rv a rm o s o “ 2 ” em nenhum

lan çam en to . O ra, aparecer o “ 2 ” pelo m enos uma vez é o evento com plem en­
tar de o evento não com parecer nenhuma vez. L o go , a probabilidade desejada é:

1-

EXERCÍCIOS
4 5 4 . Se A e B são eventos independentes, prove que A c e B também o são. Isto é,
prove que a implicação abaixo é verdadeira:
P(A O B) = P(A) • P(B) => P(AC fl B) = P(AC) • P(B).

Demonstração
P(B) = P(A fl B) + P(AC fl B) (teorema da probabilidade total)
Logo:
P(B) = P(A) • P(B) + P(AC 0 B)
P(AC O B) = P(B) - P(A) - P(B) , _
P(AC n B) = P(B) [1 - P(A)] P(AC n B) = P(B) • P(AC).

4 5 5 . Prove (usando o exercício 454) que se A e B são independentes:


a) A e B c são independentes
b) Ac e B c são independentes.

4 5 6 . Prove que, se A e B são mutuamente exclusivos, f\ A ) > 0 e P (B ) > 0, então


A e B são dependentes.

457. Numa sala existem 4 homens e 6 mulheres. Uma mosca entra na sala e pousa nu­
ma pessoa, ao acaso.
a) Qual a probabilidade de que ela pouse num homem (P(//))?
b) Qual a probabilidade de que ela pouse numa mulher
c) Os eventos H e M são independentes?

137
PROBABILIDADE

4 5 8 . De um baralho de 52 cartas, uma é extraída ao acaso. Sejam os eventos:


A : a carta é de copas
B : a carta é um rei.
C : a carta é um rei ou uma dama.
Quais dos pares de eventos são independentes?
a) A e B b) A e C c) B e C

4 5 9 . As probabilidades de que duas pessoas A e B resolvam um problema são: F \ A ) =


= — e P (B ) = — . Qual a probabilidade de que:

a) ambos resolvam o problema?


b) ao menos um resolva o problema?
c) nenhum resolva o problema?
d) A resolva o problema mas B não?
e) B resolva o problema mas A não?

4 6 0 . A probabilidade de um certo homem sobreviver mais 10 anos, a partir de uma


certa data, é 0,4, e de que sua esposa sobreviva mais 10 anos a partir da mesma
data é 0,5. Qual a probabilidade de:
a) ambos sobreviverem mais 10 anos a partir daquela data?
b) ao menos um deles sobreviver mais 10 anos a partir daquela data?

4 6 1 . A probabilidade de que um aluno A resolva certo problema é F\A ) = - y , a de que


1 . 2
outro aluno B o resolva é P (B ) = — e a de que um terceiro aluno C o resolva
1 .
é P (C ) = — . Qual a probabilidade de que:
4
a) os três resolvam o problema?
b) ao menos um resolva o problema?

Solução

Assumindo que A , B e C são eventos independentes, temos:

a) P(A n B n C) = P(A) • P(B) • P(C) = y •y •y =

b) Queremos calcular P(A U B U C ).

Temos:
P(A U B U C) = P(A) + P(B) + P (C )-P (A fl B )- P (A O C ) - P ( B H C ) +
+ P(A O B O C)

138
P R O B A B IL ID A D E

Logo:
P(A U B U C) = P(A) + P(B) + P (C )- P (A )P (B ) - P ( A ) P ( C ) - P ( B )P ( C ) +
+ p (a n b n o

P(A U B U C) = y + y + j -------- 1--------L + J _


6 . 8 12 24
18 3
P(A UBUC) = ^ = j .
24 4

4 6 2 . Luís tem probabilidade — de convidar Alice para um passeio num domingo.


4 2 1
A probabilidade de que César a convide é — e a de Olavo é — . Qual a pro­
babilidade de que:
a) os três a convidem para o passeio?
b) ao menos um a cortvide para o passeio?
c) nenhum a convide para o passeio?

463. Em um circuito elétrico, 3 componentes são ligados em série e trabalham inde­


pendentemente um do outro. As probabilidades de falharem o 1?, 2? e 3? com­
ponentes valem respectivamente p , = 0,1, p 2 = 0,1 e p 3 = 0,2. Qual a proba­
bilidade de que não passe corrente pelo circuito?
Ci C2 C3

464. (Problem a proposto p o r Chevalier De Meré a Pascal)


O que é mais provável:
a) obter pelo menos um “ 6 ’* jogando um dado 4 vezes ou
b) obter um par de 6 pelo menos um^ vez jogando dois dados simultaneamente
24 vezes?

465. Uma moeda é lançada 10 vezes. Qual a probabilidade de:


a) observarmos 10 caras?
b) observarmos 10 coroas?
c) observarmos 4 caras e 6 coroas?

XV. Lei binomial da probabilidade


124. Ensaios de Bernoulli
C onsiderem os um experim ento que consiste em uma seqüência de en ­
saios ou tentativas independentes, isto é, ensaios nos quais a p r o b a b ilid a d e d e
um resu ltad o em c a d a en sa io n ã o d e p e n d e d o s resu ltados o c o r r id o s n o s en saios

139
P R O B A B IL ID A D E

a n terio res , nem d o s resu ltad os n o s en sa io s p o s terio res. Em cad a en saio , po­
dem oco rrer apenas dois resultados, um deles que cham arem os de su cesso ( S )
e o u tro que cham arem os de fr a c a s s o ( F ). A probabilidad e de o co rrer su cesso
em cad a ensaio é sem pre /?, e conseqüentem ente, a d e fr a c a s s o é q = 1 - p .
T a l tipo de experim ento recebe o nom e de en sa io d e B ern ou lli (pois os prim ei­
ros estudos a esse respeito devem -se a Jacq u es B ernou lli, m atem ático do século
X V II ).

1 2 5 . Exemplos de ensaio de Bernoulli

1) U m a m oeda é lançad a 5 vezes. C ad a lançam ento é um en saio, em que


dois resultados podem o co rrer: cara ou co ro a . C ham em os de su cesso o resulta­
do ca ra e de fr a c a s s o o resultado c o r o a . Em cada en saio, p = - y e q = - j - .

2) U m a urna contém 4 bolas verm elhas e 6 bran cas. U m a bo la é extraí­


d a , observada sua co r e reposta na u rna; este procedim ento é repetido 8 vezes.
C ad a ex tração é um ensaio, em que dois resultados podem o co rre r: bola ver­
m elha ou bola bran ca. C ham em os de su ce sso o resultado b o la v erm elha e
6
fr a c a s s o o resultado b o la bra n ca . Em cad a caso p = e q
10
3) Um dado é lançado 100 vezes. C onsiderem os os dois resultados: sair
o núm ero “ 5 ” ou sair um núm ero d iferente de “ 5 ” . C ada lan çam en to é um
ensaio de B ernou lli. C ham em os de su cesso o resultado sa ir o “ 5 ” e de fr a c a s s o
o resultado n ã o sair o “ 5 ” . Em cad a ensaio p = — e q =
6 6

126. Observação

O s nom es su cesso e fr a c a s s o não têm aqui o significado que lhes dam os


na linguagem co tid ian a. São nom es que servem apenas para designar os dois
resultados de cad a ensaio. A ssim , no ex e m p lo /, poderiam os ch am ar de suces
s o o resultado c o r o a e de fr a c a s s o o resultado cara.
No exem plo 1, sejam os eventos:

A ,: o co rre cara no 1? lan çam en to , P ( A ,)

A 2: o co rre cara no 2? lan çam en to , P (A 2) =

A 5: o co rre cara no 5? lan çam en to , P (A 5) = — .

140
PROBABILIDADE

E n tã o , o evento A , H A 2 f l ... f l A 5 corresponde ao evento sa ir cara


n o s 5 la n ça m en to s , que é:
Í(K, K , K , K , K)(.

C om o os 5 eventos são independentes,


1
P(A, n a2 n ... n a 5) = 4 - • 4 - • 4 - 2 2 32

Se quiserm os a probabilidad e de o b ter duas caras e em seguida três c o ­


ro as, então o evento que nos interessa é:
A, n A2 n Af n Ac
4 n A£ que é |(K, K , C , C , C)|

L o go ,
_L 1
P (A , n a 2 n a? n Af n ao =
2 2 2 2 32

É fácil perceber neste exem plo que a probabilidade de qualquer quíntupla


ord enad a de caras e co ro as é — -, pois em qualquer quíntupla ordenada

_ , __ ) a probabilidade P ((__ , _)| será:

_L _i_ 1
2 2 2 2 2 32’
\\
Su p onham os, agora, o evento sair exatam ente uma ca ra . Isto é:
|(K, C , C , C , C ), (C , K , C , C , C ), (C , C , K , C , C ), (C , C , C , K , C ), (C , C , C , C , K)j

P o rta n to , a p robabilidad e deste qvento é:


5
1 + 4 - + ^ + ^ + 1
32 32 32 32 32 32

Se quiserm os a probabilidad e de o evento revelar exatam en te duas c a ­


ras, terem os que calcu lar o n ú m ero de quíntuplas ordenadas, em que existem
duas caras ( K ) e três co ro as ( C ) . O ra, a A nálise C om b in ató ria nos ensina que
este n ú m ero é o núm ero de perm utações de 5 elem entos, com dois repetidos
(iguais a K ) e três repetidos (iguais a C ), isto é,

5!
PP = = 10.
2! 3!

L o go , a probabilidade d esejada é

141
PROBABILIDADE

127. D is t r ib u i ç ã o b i n o m i a l

O s exem plos anteriores podem ser generalizados, segundo o que se co ­


nhece por d istribu ição bin o m ia l.
C onsiderem os então um a seqüência de n ensaios de B ern ou lli. S e ja p
a probabilidade de su cesso em cad a ensaio e q a probabilidade de fr a c a s s o .
Q uerem os calcu lar a p r o b a b ilid a d e P k, d a oco rrên c ia d e ex atam en te K
su cessos , n o s n en sa io s. É evidente que K G [0, 1, 2, . .. , n J.
S e ja m os eventos:
A , : o co rre sucesso no i-é sim o ensaio, P(/4,) = p .
A f : o co rre fracasso no i-é sim o en saio , P M f ) = q .
O evento “ ocorrem exatam ente K sucessos nos n en saio s” é form ado
por to d a s a s en u p las o r d e n a d a s em q u e existem K su cessos (S) e n - K fr a c a s ­
s o s (F ). O núm ero de enuplas ordenadas nessas condições é:

p K , n—K _ ________ D.!________ = í^\


" K ! (n - K )! \K/

A probabilidad e de cad a enupla ordenada de K sucessos ( S ) e (n - K )


fracassos ( F ) é dada por:

I p • p • ... • pj *^q • q • ... • qj = pK • q n" K


v V J v V *

K vezes (n—K) vezes

pois qualquer enupla ordenada deste tipo é a in terseção de K eventos do tipo


A, e (n - K ) eventos do tipo A f , e, com o esses eventos são independentes, a
probabilidade da in terseção dos mesmos é o p r o d u to das probabilidades de cada
um , isto é, p K • q n~K. P o r exem plo, a enupla (j S, S , S , S ,j |F , F , F )j
é igual à interseção 1 v v —
K n -K

a , n a 2 n ... n a k n a £ +1 n ... n A£

cu ja probabilidade é /7a • q n K.
L o g o , se cad a enupla ord enad a com exatam ente K sucessos tem proba­
bilidade p K • q n K e existem ( ^ ) enuplas desse tipo, a probabilidade P K d e exa­

tam en te K su cessos n o s n en sa io s será:

Pk
(k ) •

142
PROBABILIDADE

1 2 8 . Exemplo 1

U m a urna tem 4 bolas verm elhas ( V) e 6 brancas ( B ). U m a b o la é ex­


traíd a, observada sua co r e reposta na urna. O experim ento é repetido 5 vezes.
Q ual a probabilidad e de observarm os exatam ente 3 vezes b o la verm elha?
Em cada en saio , considerem os co m o su cesso o resultado “ bola verm e­
lh a ” , e fr a c a s s o “ b o la b ra n ca ” . E n tã o :

E stam o s interessados na probabilidad e P 3. T em os:


5!_______8______9 _ = 7 20
3! 2 ! ’ 125 ’ 25 3 125

129. Exemplo 2

Num a cidade, 10% das pessoas possuem ca rro de m arca A . Se 30 pes­


soas são selecionadas ao acaso , com reposição, qual a probabilidad e de ex a ta ­
m ente 5 pessoas possuírem carro da m arca A ?
E m cad a escolha de um a pessoa, considerem os os resultados:

S u c esso : a pessoa tem carro m arca A .


F r a c a s s o : a pessoa não tem carro m arca A .
E n tã o : p = 0,1, q = 0,9, " 30.
E stam os interessados em P 5. T em os:

1 3 0 . Observação

O problem a de o b ter K sucessos em n ensaios de Bernoulli pode ser en ­


carad o co m o um problem a cu jo espaço am o strai é Q = [ 0 , 1, 2 , . . . , /?}, isto
é, cad a elem ento de Í2 é o núm ero de sucessos em n ensaios de Bernoulli e a
d istribu ição de probabilidade é dada por:

T a l d istribu ição é cham ad a b in o m ia l , pois cad a probabilidad e P K é d a­


da pelo term o g e r a l d o b in ô m io d e N evtton (p + q ) n.

143
PROBABILIDADE

EXERCÍCIOS
4 6 6 . Considere uma distribuição binomial com n = 10 e p = 0,4. Calcule:
a) P0 b) P4 c) P6 d) P8

4 6 7 . Uma moeda é lançada 6 vezes. Qual a probabilidade de observarmos exatamente


duas caras?

4 6 8 . Um dado é lançado 5 vezes. Qual a probabilidade de que o “ 4 ” apareça exata­


mente 3 vezes?

4 6 9 . Um estudante tem probabilidade p = 0,8 de acertar cada problema que tenta re­
solver. Numa prova de 8 problemas, qual a probabilidade de que ele acerte exata­
mente 6 .

4 7 0 . Uma pessoa tem probabilidade 0,2 de acertar num alvo toda vez que atira. Su­
pondo que as vezes que ela atira são ensaios independentes, qual a probabilidade
de ela acertar no alvo exatamente 4 vezes, se ela dá 8 tiros?

4 7 1 . A probabilidade de que um homem de 45 anos sobreviva mais 20 anos é 0,6. De


um grupo de 5 homens com 45 anos, qual a probabilidade de que exatamente
4 cheguem aos 65 anos?

4 7 2 . Um exame consta de 20 questões tipo certo ou errado. Se o aluno “ chutar” todas


as respostas, qual a probabilidade de ele acertar exatamente 10 questões? (Indi­
que somente os cálculos.)

4 7 3 . Uma moeda é lançada 2n vezes. Qual a probabilidade de observarmos n caras


e n coroas?

4 7 4 . Uma moeda é lançada 6 vezes. Qual a probabilidade de observarmos ao menos


uma cara?

Solução

Temos: n = 6, p = —, q = —.

Estamos interessados em calcular: P, + P 2 + P 3 + P 4 + P 5 + P6-


Ora, como:
P0 + P, + P2 + P3 + P4 + P 5 + P6 = 1
P, + P2 + P 3 + P4 + P5 + P6 = 1 - P0

144
PROBABILIDADE

logo, basta calcularmos P0.


Temos:

1 63
Logo, a probabilidade desejada é: 1 - — .= .
64 64

475. Uma moeda é lançada 10 vezes. Qual a probabilidade de observarmos pelo me­
nos 8 caras?

476. Um time de futebol tem probabilidade p = -y- de vencer todas as vezes que joga.
Se disputar 5 partidas, qual a probabilidade de que vença ao menos uma?

477. Uma moeda é lançada 9 vezes. Qual a probabilidade de observarmos no máximo


3 caras?

Solução

Temos: n = 9, p = y , q = y .

Estamos interessados em calcular: P 0 + P , + P 2 + Ps. Então:

Logo, P0 + P, + P2 + P3 = = 0,254.

478. Em 4 ensaios de Bemoulli, a probabilidade de sucesso em cada um é p = 0,4.


Qual a probabilidade de observarmos no mínimo 3 sucessos?

479. Um teste tipo certo ou errado consta de 6 questões. Se um aluno “ chutar” as


respostas ao acaso, qual a probabilidade de que ele acerte mais do que 2 testes?

480. Numa cidade, 30% da população é favorável ao candidato A. Se 10 eleitores fo­


rem selecionados ao acaso, com reposição, qual a probabilidade de que mais da
metade deles seja favorável ao candidato A l (Indique os cálculos.)

145
P R O B A B IL ID A D E

481. Um casal planeja ter 5 filhos. Admitindo que sejam igualmente prováveis os re­
sultados: filho do sexo masculino e filho do sexo feminino, qual a probabilidade
de o casal ter:
a) 5 filhos do sexo masculino?
b) exatamente 3 filhos do sexo masculino?
c) no máximo um filho do sexo masculino?
d) o 5? filho do sexo masculino, dado que os outros 4 são do sexo feminino?

LEITURA

Laplace: a Probabilidade
Chega aos Céus
Hygino H . D om ingues

“ É notável que um a ciência que com eçou com consid erações so ­


bre jo g o s de azar pudesse ter se elevado ao nível dos m ais im portantes
assuntos do co n h ecim en to .” C om efeito , foi isso o que efetivam ente
ocorreu com a teoria das probabilidades, a que se refere a citação . M as,
deve-se acrescentar a bem da verdade, co m o conseqüência do gênio e
do esfo rço de grandes m atem áticos que se dedicaram ao assu n to, entre
os quais o próprio autor da frase: Pierre-Sim on de Laplace (1749-1827).
L ap lace nasceu de um a fam ília de hum ildes cam poneses da N or-
m and ia, na F ran ça. A inteligência brilhante de que era d otad o foi o
instrum ento que lhe propiciou ir rom pendo, desde cedo e a passos lar­
gos, com os vínculos de sua origem . Assim é que aos 16 anos de idade
j á estava na U niversidade de C aen , onde deveria cursar teo lo g ia. M as
logo se inclinou para a m atem ática, sua verdadeira v o cação . F o rm a ­
d o , dirige-se a P aris com cartas de
apresentação a Je a n D ’A lem bert
(1717-1783), o m ais em inente m ate­
m ático francês da ép oca. C om o es­
tas não funcionassem , resolveu
apresentar-se à sua m aneira: enviou
a D ’A lem bert um su bstancioso a r­
tigo de sua au toria so bre os princí­
pios gerais da m ecânica. L ap lace
n ão a p e n a s fo i re c e b id o p o r
D ’A lem bert mas tam bém , graças à
influência deste, em 1769 era indi-

146
cad o professor da E sco la M ilitar de P aris e em 1773, com 24 anos de
idade apenas, para a seleta A cad em ia R eal de C iências.
A ssim , foi já co m o m atem ático de valor reconhecido e com a car- r
reira cien tífica em plena ascensão que L ap lace viveu o período m ais
tum ultuado da h istória de seu país: a revolução fran cesa, o período
n apoleônico e a restau ração m onárqu ica. P od ería alguém em sua p o ­
sição , ainda mais com am bições pessoais co m o ti nha, deixar de se en ­
volver com as paixões que agitaram aqueles dias? A crítica que se faz
a L aplace qu anto a seu com p ortam en to nesse período diz respeito à
sua volubilidade po lítica: em nenhum a das fases (em bora an tagôn icas)
deixou de colher hon rarias ou p osições. M enos m al para a m atem ática
e a m ecân ica, nas quais pôde trab alh ar sem m aiores con tratem p o s.
A obra-p rim a de L ap lace é o T raité d e m eca n iq u e c e le s te , publi­
cad a ao longo de 26 anos (1799-1 8 2 5 ), em cinco volum es que totalizam
2 0 00 páginas. R eunindo as grandes descobertas até en tão realizadas
no cam po da m ecânica celeste com sua enorm e co n trib u ição ao assun­
to , L ap lace com pletou o trab alh o de Newton no sentido de m ostrar
que tod os os m ovim entos dos corp os do sistem a so lar são dedutíveis
da lei da gravitação.
Newton não conseguira provar a estabilidade do sistem a solar,
o que o levou a co g itar da intervenção divina de tem pos em tem pos
para m antê-lo em ordem . L ap lace provou essa estabilidade. D aí por
que, talvez, certa feita, a uma observ ação de N apoleão sobre a ausên­
cia de Deus em sua o b ra , teria respondido não precisar dessa hipótese.
A m ecânica celeste contribu iu fortem ente para que a teo ria das
probabilidades viesse a ser uma das preocupações cien tíficas de L ap la­
ce. A fin a l, era preciso, entre outras co isas, determ inar a p ro b ab ilid a­
de de erros em dados de observações experimentais. M as outros tópicos,
co m o por exem plo a d em o g rafia, tam bém o levaram para esse cam po.
A ssim , de um co n ju n to de m em órias ligadas ao tem a, a prim eira de
1774, resulta em 1 8 1 2 o clássico T h éo rie a n a ly tiqu e d es p r o b a b ilité s .
E sta o b ra , além de reunir e sistem atizar boa parte do que era pre­
viam ente conhecido sobre o assunto, traz contribu ições próprias de L a ­
place, m uitas das quais serviram de fonte até para avanços em outros
cam pos da m atem ática, co m o a idéia de função geradora e a de tran s­
form ad a de L ap lace. Um dos pontos altos do livro é a ap licação da
probabilidade ao m étodo dos quadrados m ínim os, ju stifica n d o a co n ­
veniência de seu uso.
C om o astrô n o m o , a teo ria das probabilidades n ão era um fim
para L ap lace, m as apenas um m eio. M esm o assim ele é, sem dúvida,
um dos grandes nom es desse cam po que com tanto talen to aju d o u a
cria r.

147
Respostas
dos
Exercícios
Capítulo I 21. 0 = 46 656

1. 40 22. 810 000

2. 30 23. a) 8 b) 510

3. 30 25. 160

4. 7 200 27. 2® = 256

5. 56 28. 64

6. 132 30. (a + 1) (b + 1) (c + 1) (d + 1)

7. 600 31. 28

8. 42 (n + 1) •(n + 2)
32.
2
10. 220 = 1 048 576 formas
33. i°
11. Aproximadamente 1 000 000. 35. AA, ABB, ABAA, ABAB, BB, BAA, BABB,
232
BABA
12.
36. 40
13. 210 - 1 = 1 023
37. No máximo duas voltas.
14. 2* - 1 = 63
38. R I 1 000,00; R| 3 000,00 ou R$ 5 000,00
15. 25 — 1 = 3 1
39. 11
16. 266 = 308 915 776. Sim. 40. m = r + 4
17. 243 41. (a, b), (a, c), (a, d), (b, a), (b, c) (b, d), (c, a),
(c, b). (c. d), (d. a), (d, b), (d. c)
18. 125
42. a) 120 c) 20
19. 200 b) 5 040 d) 132

20. 10 43. 6 840

148
RESPOSTAS DOS EXERCÍCIOS

44. 30 79. 58?

45. 30 80. 13 •313

47. 192 81. 7! = 5 040

48. 240 82. (n - r + 1) •Ak

49. 24 84. 480

85. 64
50. 60

51. 720 86. 2 •(N!)2

52. 3 • A,9 l0 87. 10! c 7!

53. A^ j = 681 384 88. 288

89. 120
55. a) mr b> m!
(m - r)!
90. 8!
56. 360
91. 7!
57. 72
93. a) 17 280 b) 5 760
59. 60
94. 28 800
60. mn
95. 3 •4! = 72
61. n!
96. 144
62. 504
97. 480
63. 252
98. 720
65. 480
99. 25 •3 •7 = 672
66. 280 101. 11!
67. 125
102. 3
68. 3 537
103. (m - 1)! m!
69. 18
106. 6
70. 72
107. 12
71. 60
108. (7|
72. 180 109. J7j
73. 16
110. (12)
74. 60
111. n = 5
75. 96
115. a) (2n)! b) (n!)2
77. 198 2n •n!

78. 48 116. K! K2

149
RESPOSTAS DOS EXERCÍCIOS

117. (n - r) (n - r + 1) 153. 140

11*. í(m + l)!]2


■1 = ■ ( ?
119. (m + 1)! - 1
» ( ? ) • S ) -
120. n!

124. a) 15 b) 15 155. 112

125. (7. 8). [7. 9). {7. Oj. |8. 9), (8, Oi. |9, OJ 156. 3 136

157. 55
126 24 •(n - 4)!
= 98
•5«- ( 4 ) + ( 2 )
127. 10
159. 112
128. 1
161. a) 165 b) 60
129. p = 0 ou p = 1

130. 504
)
131. 8
= 350
132. (5)
164.4 512
133. 4
165. a) 3 b) 10 c) 15
134.7 r
166. 2 080
135. f(a) = 109
167. 140
134. 9
16*. 700
137. 15)
169. 267 960
13*. n = 13 e p = 2
170. 280
141. C524
171. 630
142. 10
/300\ /300\ /300 \
143. 10 ,7M 3 ) + ( 2 ) +{ . )
144. 848 173. C8 4 • P4 = 1 680
145. 1 023 174. 840
146. 91
175. 10
14*. C5 2 ~ C3 2 = 7
176. 4

149. ( 4 ) = 126 n!
,77* 3! (n - 3)!
150. 56
17*. 55
151. 7

152. 15

150
RESPOSTAS DOS EXERCÍCIOS

180. a) 28 b) 56 c) 28 (a + b)!
210.
a! b!
182. a) 4 b) 3
211. 252
183. 100
212. 182
184. n (n - 3)
213. 120
185. 35
214. 70
.«6. ( 5 ) - 9
215. 255

187. 969
216.
( 4 ) •( 5 ) = 27 720

217. 4 620

c p.2 C q2 52!
218.
(13!)4
191. 28 800

219. 20!
192. ( “ ) • ( “ ) = 3 8 .2 256 (5!)4

220. 2io
(\P Q
q ’/
)
221. 280

195. m 222. 126


p- m
15!
196. 21 223.
(5Í)3 •6
c o.
1 1
&
r>'

224. a) 28 b) 286 c) 1 001

198. 495 225. 126

226. 21
199. 60
227. 1 287
200. 15
| 228. 35
20,. 2!8 2,
229. 6

202. 831 600 minutos ou 577 dias e meio

203. 20!
10! 10!

204. 210 Capítulo II


205. 10 230. a) x3 + 9x2b + 27xb2 + 27b3
b) 1 - 5x2 + 10x4 - 10x6 + 5x8 - x10
207. 10
c) x2 - 4x J xy + 6xy - 4y Nxy + y2
208. 40 d) sen4 0 + 4 sen3 0 cos 0 + 6 sen2 0 cos2 0 +
+ 4 sen 0 cos3 0 + cos4 0
209. 210 e) 243 - 405y + 270y2 - 90y3 + 15/ - ys

151
RESPOSTAS DOS EXERCÍCIOS

257. -352
231. 10m3 +
m m5
232. x7 + 7x6a + 21x5a2 + 35x4a3 + ro
+ 35x3a4 + 21x2a5 + 7xa6 + a7
259. 252
233. a = 1 e b = 3
260. 6
234. a) 8 b) 11 c) n + 1
261.
235. a) 51 (? )
b) x50; x49 ■a; x48 •a2; x47 •a3
262. 280

236. y99 263. 153


( T ) * 90'
264. 1120
237. ( ‘ ? ° ) x«y; 625
( T ) * 100: 265. a = ±2
238. 16
266. a = 3
239. IO10
267. Não existe.

240. 16
268. 1! posição
5
269. n deve ser divisível por 3.
241. 32°

242. -1 6 0 Js 270. não

271. 720 x V
243. 2n
272. n = 8
244. X 12
273. 2n - 1 = 13
245. 5n
274. n = 4
246. 60
275.
247. 30 618 x V %
[(!)]’
248. 80 276. 2n (n - 1)

249. 28
277.
250. 5 005 a,2x3
c ; 1)
278. -20
251. -455 x3a6
279. 17
252. -280
280. 6

253. Í 2 1 282. a) 1,02 b) 0,94


32
283. 1,06
254. Não há termo de grau 1.
285. a) 510 b) 68
64
255.
81 286. 74

256. 15 287. a) 0 b) 16

152
RESPOSTAS DOS EXERCÍCIOS

2** . 16 807
331.
*> ( e )
2*9. p = 9 / I Ç\ /I
290. 1 b) ( 7 ) e (« )
5
291.
II
vO
3

332. 252 xT y10


292. m = 5
333. 12 e 12
293. 24
334. 210
294. a) F c) V e) V
b) V d) V 0 V 336. 243

296. 16 337. 13

297. zero 33*. i = 6, j = 1 , k =


3 3 + C9 4 = .

299. 10 Capítulo III


300. 2n - 1 339. Í2 = {P, R, O, B, A, I, L, D, E)

301. 211 - 1 = 2 047 340. n = (V, B. A)


302. zero 341. íl = (1, 2, 3....... 49, 50)

^=
>

303. 342.
li

|2C» 2C, 2p, 20, ..., 8C, 8C, 8p, 80, ...,
o

>

>
>
>
O
o

305. 2" - 1
em que os índices e, c, p , o indicam res­
306. 2 pectivamente espadas, copas, paus e ouros
343. n = l(V, V), (V, B), (B, V), (B, B»
n (n + 1) (2n + 1)
316.
6 344. n = ((A, B, C), (A, C, B), (B, A, Q , (B, C, A),
(C, A, B) (C, B, A))
319. n = 3k - 1
345. í) = {(MMM), (M M F), (M FM ), (M FF),
320. a2 = 50 •99 •9" (FMM), (FMF), (FFM), (FFF))
em que M indica o sexo masculino e F, fe­
321. x = ^ - + 2kir; k 6 2 minino.

347. Q= Í(A, B), (A, C), (A, D), (A, E), [B, C),
323. p = 5 (B, D), (B, E), (C. D), (C, E), JD, E ] )
324. x = 1 ou x = 5 34*. Q= [1, 2, 3, .... 364, 365)
325. p = 5 349. a) (2,4, 6, 8, 10, 12, 14, 16, 18, 20, 22, 24, 26,
28, 30)
326. m = 2 ou m = 5
b) ( 1 ,3 ,5 ,7 , 9, 11, 13, 15, 17, 19,21, 23,25,
27, 29)
327. m = 8; = 56 c) {2, 3, 5, 7, 11, 13, 17, 19, 23, 29)
d) [17, 18, 19, 20, 21, 22, 23, 24, 25, 26, 27,
32*. m = 2p 28, 29, 30)
e) (10, 20, 30)
329. 45
0 {3, 6, 8, 9, 12, 15, 16, 18, 21, 24, 27, 30)
330. ln € Z | n > 3) g) ÍJ - (6, 12, 18, 24, 30)

153
RESPOSTAS DOS EXERCÍCIOS

350. a) ((3, 1), (3, 2), (3, 3), (3, 4), (3, 5), (3, 6))
359. a) y b) y
b) |(1, 1), (2, 2), (3, 3), (4, 4), (5, 5), (6, 6»
c) ((2. 1), (2, 2), (2, 3), (2, 4), (2, 5), (2, 6),
(1, 2), (3, 2) (4, 2), (5, 2), (6. 2))
d) [(1, 6),(2, 5), (3, 4), (4, 3), (5, 2), (6, 1))
e) |(1, 1),(1, 2), (2, 1), (1, 3) (2, 2), (3, 1),
(1, 4), (2, 3), (3, 2), (4, 1), (I, 5), (2, 4), 362.
(3, 3), (4, 2), (5, 1)}
3)l2 » T a l2
351. a) {(K, 1), (K, 2), (K, 3), (K, 4), (K, 5), (K, 6)) 363.
b) ((K, 2), (K, 4), (K, 6), (C, 2), (C, 4), (C, 6)]
a)i r <
c) ((K, 3), (C, 3))
d) |(K, 1), (K, 2), (K, 3), (K, 4), (K, 5), (K, 6),
b)i ; ^ « t
(C, 2), (C, 4), (C, 6)) c) 0,0123; 0,9877
364. b) 0,042
e) 0 , B e C são mutuamente exclusivos.
0 l(K. 3)} 367. a) 0,4 b) 0,8 c) 0,7
g) |(C, 1), (C, 2), (C, 3), (C, 4), (C, 5), (C, 6))
369. a) 0,5 b) 0,8 c) 0,8
h) ((K, 1), (K, 2), (K, 4), (K, 5), (K, 6), (C, 1),
(C, 2), (C, 4), (C, 5), (C, 6))
X 12
370.
352. a) ((1, 1),(2, 2), (3, 3), (4, 4))
a)l2 C)T e)1 7
b) [(2, 1),(3, 1), (4, 1), (3, 2), (4, 2), (4, 3))
c) í(l, D) »TT •-è-
d) 1(1, 1), (2, 4)j
371. 1
— 2
c)X — l
d) —
e) {(1, 1), (1, 2), (1, 3), (1, 4), (1. 5)} « T » 2 5 5
0 ((1, 3), (2, 3), (3, 3), (4, 3))
372. X 11
353. a) ((I, V), (I. B)} a ) lõ õ " » 'ir C)T
b) [(II, V), (II, B)j
c) 1(1, V), (II, V)) 3
373.
5
d) [(I, B), (II, B)j
e) Q 1
0 ld, V)] 374.
2
g) [(I, V), (II, V))
376.
354. a) 0 = ((S, S, S), (S, S, N), (S, N, S), a) T » T C)T
(S, N, N), (N, S, S), (N, S, N),
(N, N, S), (N, N, N)) 377. e) 1
em que S representa resposta sim e N, res­
a) i ° T
posta não.
b) A = ((S, S, N), (S, N, S), (S, N, N), b)T » ír
(N, S. S). (N. S, N), (N, N, S),
(N, N, N))
378.
“54"
1
355. -J-
379.
19 *
400
356. 1
380. R S 4 900,00 para A e R J 700,00 para B
357. a) Pi = P2 = P 3 = °» 3 d) °»4
b) 0,6 e) 0,7; 0,3 10
c) 0,4 0 0,3; 0,7

35S. A distribuição é correta. 382. a) 0,4 b) 0,7 c) 0,6


RESPOSTAS DOS EXERCÍCIOS

20 49 16
3*3. b)
25 a)T 100 25

405. 11
385. b)
a)T c> l õ 850

406. a) 1 8
. 21 b)
386. a) ----- 221 663
50
7
387. a) b) 22 « t
100 10 '>■»

388. 408. a) ( ? ) c)
( ? ) ( ? )
a)T « T •’ í (200\ (200\
{ 10 j \ io j
-I /20 \
\ ío/
b)
389. — (200\
[ 10 j
4 •C«
390. /50\ /10\
409. a) ( ? ) c)
\2 / \ 3 /

392. (?) (?)


720

393. — (?) d)
, (?)
b)

(n ~ 2)! 3!
394.
410. 9
190
395. b)
a)T
1 411.
396.
12
(?)
1
397. a) b)
1 000 1 000 1 000
412. a)
48
C) 1
(? )
10!
398.
5! 5! 63 (fí (? )
210 256

1 b)
m
399.
15
(? )
5
401.
18 413.-
34
30 • 240
402.
105
25
414. SL
403. a)
64 11-á- (?)

155
RESPOSTAS DOS EXERCÍCIOS

3
415. - L c)
10 436 a ) T T 8

4
416. J L c)
10 437‘ a) i r 15

418. ±
3 « • 1

439. a) Probabilidade de uma pessoa de 40 anos sa


breviver mais um ano.
b) Probabilidade de uma pessoa de 40 anos so­
breviver mais 2 anos.
421.
17
440. a)
14 28
422.
221
441. x = 3

423.
9 b)
60
15
424.
128

i i
425. a) c)
T T

í í
b) d) 445. 0,9
2 2

1 24 1 _3_
426. a) b) c) 44? . a) ± C)
2 49 5 13

1 7 4
427. a) c) e)
6 11 15

_i_ 450: a) 46,7% b) 11,1%


b) d) 1
5 Obs.: As respostas foram arredondadas pa­
ra uma casa decimal.
1 1
428. a) b)
2 3 451. 24,6%

13 . 2 19 7 452. 4,8%
429. a) b)
25 ’ 25 ’ 25 13
453.
1
430. 8
5
457. a) 0,4
b) 0,6
c) Não, pois P(H H M) = 0 * P(H) • P(M).

1 458. a) A e B são independentes.


432. b) A e C são independentes.
4
c) B e C são dependentes.

433.
2_
5 459. a) —
0 -í t
435.
6
\_
■-S- •-â-

156
RESPOSTAS DOS EXERCÍCIOS

460. a) 0,2 b) 0,7 471. 0,259

■. 31 , 9
462. a) — 472.
' 20 b ) iõ - c) 40 (io) (t )" s 0,176
463. 0,352

464. o evento enunciado em (a) é mais provável; sua


473.
(?)(*r
probabilidade é de aproximadamente 0,5177 e 7
475.
a do evento enunciado em (b) é de 0,4914. 128

105 3 093
465. a) 1 476.
1 024 b) 1 024 C) 512 3 125

466. a) 0,006 b) 0,251 c) 0,111 d) 0,011 47S. 0,180

467.0,234 21
479.
32
4M. 125 ■ 10
3 888
480. J ] f ' v° ) (0,3)» ■(O,'
469. 0,294 x=,6 \ A /

470. 0,046 , 1 ., 5
481.
a) -M b> 1 6 -

157
Testes de
vestibulares
Análise com binatória

i. ( U F - E S ) N um a p a relh o te le fô n ic o , a s d e z te c la s n u m erad as
e s tã o d is p o s ta s em f ile ir a s h o r iz o n ta is , c o n fo rm e in d ic a a
fig u ra a o lad o . S e ja Na q u an tid ad e de n ú m ero s d e te le fo n e 1? fileira
c o m 8 d íg ito s , q u e c o m e ç a m p e lo d íg ito 3 e te rm in a m p elo
d íg ito z e ro , e , a lé m d is so , o 2• e o 3 - d íg ito s s ã o da p r im e i­
2- fileira
ra file ir a do te c la d o , o 4 ? e o 5 ” d íg ito s sã o da segu n d a f i­
le ira . e o 6? e o 7^ s ã o da te rc e ira file ir a .
O v a lo r de N é: 3‘? file ira

a) 27
b) 216
c> 5 1 2
d) 729
e) I 331

2. ( P U C - R J ) A sen h a de a c e s s o a um jo g o de c o m p u ta d o r c o n s is te em q u a tro c a ra c te re s a lf a b é tic o s ou n um é­


ric o s . sen d o o p rim e iro n e c e s sa ria m e n te a lfa b é tic o . O n ú m ero d e sen h a s p o s sív e is s e rá . en tã o :

a) 3 6 4 c) 2 6 - 3 6 ' e) 10 • 2 6 4
b) 1 0 -3 6 ' d) 2 6 4

3 . ( F a a p - S P ) Q u an tas m o to s podem s e r lic e n c ia d a s se ca d a p la c a tiv e r 2 v o g a is (p o d e n d o h a v e r v o g a is re p e ­


tid a s) e 3 a lg a r ism o s d is tin to s?

a) 25 0 0 0 c ) 120 0 0 0 e) 32 0 0 0
b) 120 d ) 18 000

4. ( F a t e c - S P ) Para p a rtic ip a r de um c a m p e o n a to d e fu te b o l, o té c n ic o da F a te c s e le c io n o u 2 2 jo g a d o r e s . 2
para c a d a p o s iç ã o . O n ú m ero de m a n e ira s d is tin ta s q u e o té c n ic o pode fo rm a r e s s e tim e de m od o q u e n e ­
nhum jo g a d o r atue fo ra de sua p o s iç ã o é :

a) 2 541 c) 462 e) 44
b) 2 0 4 8 d ) 231

158
T E S T E S D E V E S T IB U L A R E S

5 . ( U F F - R J ) O e stu d o da g e n é tic a e s t a b e le c e q u e , c o m a s b a s e s a d e n in a (A ), tim in a ( T ), c it o s in a ( C ) e


g u an in a ( G ) , p o d e m -se fo rm a r, a p e n a s, q u a tro tip o s de p a res: A -T . T -A . C -G e G -C .
C e r t o c i e n t i s t a d e s e ja s i n t e t i z a r um
fra g m e n to d e D N A c o m d ez d e s se s pa­
r e s , de m od o q u e:
• d o is p a r e s c o n s e c u t iv o s n ã o s e ja m
ig u a is;

• um par A -T não s e ja seg u id o p or um


par T -A e v ic e -v e r s a ;

• um par C -G não s e ja seg u id o p or um


par G -C e v ic e -v e r sa .

S a b e -s e que d ois fragm entos de D N A sã o id ê n tico s se co n stitu íd o s por pares igu ais d isp osto s na m esm a ordem .
L o g o , o n ú m ero d e m a n e ira s d is tin ta s q u e o c ie n tis ta pode fo rm a r e s s e fra g m e n to d e D N A é :

a) 2 " d ) 2 10
b ) 2 20 e ) 2 2 • 10
c) 2 -1 0

6 . (U . F. O u ro P r e to -M G ) O s ra m a is t e le f ô n ic o s de um a e m p re sa sã o in d ica d o s p or n ú m ero s de trê s a lg a r is ­


m os d is tin to s, sen d o q u e o p rim e iro a lg a rism o d o n ú m ero in d ica o d e p a rta m e n to da e m p re s a a o q u al p er­
te n c e o ram al. S e o s q u a tro d ep a rta m e n to s da e m p re s a sã o in d ic a d o s p e lo s a lg a r ism o s d e I a 4 . q u a n to s
n ú m e ro s d e ram ais e x is te m , n o m á x im o ?

a) 2 2 4 c ) 324
b) 2 8 8 d) 4 0 0

7 . ( U F - E S ) E m um g ru p o de 6 0 m u lh e res e 4 0 h o m en s e x is te m e x a ta m e n te 2 5 m u lh e res e 12 h o m e n s que


to cam algu m in stru m e n to m u s ic a l. D e q u a n ta s m a n e ira s p o d em o s fo rm a r um a du p la de um h om em e um a
m u lh e r d e m od o q u e p e lo m en os u m a d as p e ss o a s da du pla to q u e algu m in stru m e n to ?

a) 3 0 0 d) I 420
b) 7 2 0 \\ e) I 720
c) I 000 ’ i

8 . ( P U C -P R ) D u ran te um e x e r c íc io da M a rin h a de G u e rra , e m p re g a ra m -se s in a is lu m in o so s para tra n s m itir o


c ó d ig o M o rse . E s te c ó d ig o s ó e m p re g a du as le tra s ( s in a is ): pon to e tra ç o . A s p a la v ra s tra n sm itid a s tin h am
de um a a s e is le tra s . O n ú m ero de p a la v ra s q u e p od iam s e r tra n sm itid a s é:

a) 3 0 \ d) 1 26
b) 15 e) 64
c) 720

9 . ( U F - C E ) A ss in a le a alte rn a tiv a na q u al c o n s ta a q u a n tid a d e de n úm eros in te iro s fo rm a d o s p or trê s a lg a r is ­


m os d is tin to s, e s c o lh id o s d e n tre I . 3 , 5 , 7 e 9 , e que sã o m a io r e s q u e 2 0 0 e m e n o re s q u e 8 0 0 .

a) 3 0 d) 48
b) 36 e) 54
c) 42

10 . ( U F - P E ) S u p o n h a que e x is ta m 2 0 d ife re n te s tip o s d e a m in o á c id o s . Q u al d o s v a lo res a b a ix o m a is se a p r o ­


x im a d o n ú m ero de a g ru p a m e n to s o rd en a d o s, fo rm a d o s de 200 a m in o á c id o s , q u e pod em s e r o b tid o s ?
D ad o : U se a a p r o x im a ç ã o : lo g 2 = 0 ,3 0 .

a) 10" d)
b) 10" e)
c) I 0 2*

159
T E S T E S D E V E S T IB U L A R E S

11 . (U F -R N ) A fig u ra ao lado rep resen ta um m apa das estra d a s


que in te rlig am as com u n id ad es A. B. C, D. E e F.
A s s in a le a o p ç ã o q u e in d ic a q u a n to s p e rc u rso s d ife re n te s
e x is te m para se c h e g a r à co m u n id a d e D (p a rtin d o -s e de >4),
sem q u e se p a sse m ais de um a vez num a m e sm a c o m u n i­
d ad e. em c a d a p ercu rso .

a) 7 2
b) 12
c) 18
d) 3 6

12. (V u n e s p -S P ) N a c o n v e n ç ã o de um p artid o para la n ç a m e n to da ca n d id a tu ra de um a c h a p a a o g o v e rn o de


c e rto e sta d o h av ia 3 p o s sív e is c a n d id a to s a g o v e rn a d o r, sen d o d o is h o m e n s e um a m u lh er, e 6 p o s sív e is
ca n d id a to s a v ic e -g o v e rn a d o r. sen d o q u a tro h o m e n s e du as m u lh eres. F ic o u e s ta b e le c id o q u e a c h a p a g o -
v e rn ad o r/ v ice-g o v e m a d o r s e ria fo rm a d a p or du as p e ss o a s d e s e x o s o p o s to s. S a b e n d o -s e q u e o s n ove c a n ­
d id a to s são d is tin to s, o n ú m e ro d e m a n e ira s p o s s ív e is de se fo rm a r a c h a p a é:

a) 18 d) 6
b) 12 e) 4

c) 8

13. ( M a c k e n z ie - S P ) O s n ú m ero s p ares co m 4 a lg a r ism o s d is tin to s, q u e p od em os o b te r c o m o s e le m e n to s do


c o n ju n to {(); 3 ; 4 ; 5 ; 6 : 7 ; 8 } . sã o em n ú m e ro de:

a ) 63 d) 5 •4 3
b) 4 2 0 e) 380
c ) 5 • 62

14 . (U . F. S a n ta M a r ia -R S ) Para te r a c e s s o a u m a s a la re serv a d a , ca d a u su á rio re c e b e um c a rtã o de id e n tific a ­


ç ã o co m 4 lis tra s c o lo rid a s , de m od o q u e q u a lq u e r c a rtã o deve d ife rir de to d o s o s o u tro s p ela n a tu rez a das
c o re s ou p ela ord em das m e sm a s n as lis tra s . O p era n d o c o m 5 c o re s d istin ta s e o b s e rv a n d o q u e lis tra s v iz i­
nhas n ão têm a m esm a c o r. q u a n to s u su á rio s pod em s e r id e n tific a d o s?

a) 10 d) 3 2 0
b ) 20 e) 625
c) 120 156
*

1 5 . ( U n ir io - R J ) U m a fa m ília fo rm a d a p or 3 a d u lto s e 2 c ria n ç a s vai v ia ja r num a u to m ó v el de 5 lu g a re s, send o


2 na fre n te e 3 atrás. S a b e n d o -s e q u e s ó 2 p e ss o a s podem d ir ig ir e q u e a s c r ia n ç a s d evem ir a trá s e na
ja n e la , o n ú m ero total de m a n e ira s d ife re n te s a tra v é s d as q u a is e s ta s 5 p e sso a s p odem s e r p o sic io n a d a s,
não p e rm itin d o c r ia n ç a s irem n o c o lo d e n in g u ém , é igu al a:

a) 120 d) 24
b) 9 6 e) 8
c) 48

16 . ( U F - M G ) E m um a la n c h o n e te , o s s o rv e te s s ã o d iv id id o s e m trê s g ru p o s: o v e rm e lh o , co m 5 s a b o re s; o
a m a r e lo , c o m 3 s a b o re s; e o ve rd e , c o m 2 s a b o re s. P o d e -s e ped ir u m a c a sq u in h a c o m 1, 2 ou 3 b o la s , m as
ca d a c a sq u in h a não pode c o n te r 2 b o la s de um m e sm o grupo.
O n ú m e ro d e m an eira s d is tin ta s de se p e d ir u m a c a sq u in h a é:
a) 71
b ) 86
c) 131
d ) 61

160
T E S T E S D E V E S T IB U L A R E S

17. ( E n e m -M E C ) O c ó d ig o de b a rra s, co n tid o na m a io r parte d os produtos in d u stria liz a d o s, c o n s iste num c o n ­


ju n to de v árias b arras que podem e sta r p ree n ch id a s co m c o r e sc u ra ou não. Q u an do um le ito r ó p tic o p assa
sob re e ssas barras, a leitura de um a barra cla ra é con vertid a no núm ero 0 e a de um a barra e scu ra , no n ú m ero 1 .
O b serv e a seg u ir um e x e m p lo s im p lifica d o de um c ó d ig o em um siste m a de c ó d ig o co m 2 0 barras.

S e o le ito r ó p tic o fo r p assad o da e sq u e rd a para a d ire ita irá le r: 0 1 0 1 1 0 1 UI 1 1 0 1 0 1 1 0 0 0 1


S e o le ito r ó p tic o fo r p assad o da d ire ita para a e sq u e rd a irá le r: 1 0 0 0 1 1 0 1 0 1 1 1 0 1 0 1 1 0 1 0
N o s iste m a de c ó d ig o de b a rra s, para se o rg a n iz a r o p ro ce s so de le itu ra ó p tic a de c a d a c ó d ig o , d e v e -se
le v ar em c o n s id e r a ç ã o que a lg u n s c ó d ig o s p od em te r le itu ra da e sq u e rd a para a d ire ita ig u al à da d ireita
para a e sq u e rd a , c o m o o c ó d ig o 00000000111100000000 , no s iste m a d e s c r ito a cim a .
Em um sistem a de có d ig o s que utilize apen as c in c o b arras, a quantidade de có d ig o s co m leitura da esquerd a
para a direita igual à da d ireita para a esquerd a, d escon sid eran d o-se todas as barras c la ra s ou todas as e scu ra s, é:

a) 14 c) 8 e) 4
b) 12 d) 6

18. (F u v e st-S P ) C onsidere todas as trinta e duas seq ü ên cias, com c in c o elem en tos cad a uma, que podem ser form adas
com o s algarism os 0 e 1. Quantas dessas seq üên cias possuem pelo m enos três zeros em p osições con secu tiv as?

a) 3 c) 8 e) 16
b) 5 d ) 12

19. ( U E - C E ) Q u a n to s núm eros ím p a re s, c a d a um c o m trê s a lg a r ism o s , podem s e r fo rm a d o s c o m o s a lg a r is m o s


2 , 3 , 4 , 6 e 7 . se a re p e tiç ã o de a lg a r is m o s é p e rm itid a ?

a) 6 0 c) 40
b) 50 d) 30

2 0 . ( U E -P A ) D u ran te o m ês de ju n h o , o c o rr e a tra d icio n a l c o m p e tiç ã o en tre q u a d rilh a s d o s b a irro s. D e q u a n ta s


m a n e ira s pod em o c o rr e r a e s c o lh a da cam peã e yice-campeã e n tre a s cinco q u a d rilh a s fin a lis ta s , sa b e n d o -
se q u e não o c o rre m e m p a te s ?

a) 10 c) 60 e) 150
b ) 20 d ) 120

2 1 . ( U F - A L ) C o m o s e le m e n to s d o c o n ju n to { 1 , 2 , 3 , 4 , 5 , 6 , 7 } fo rm a m -s e n ú m ero s de 4 a lg a r ism o s d is tin to s.


Q u a n to s d os n ú m ero s fo rm a d o s não s ã o d iv is ív e is pdr 5 ?

a) 15 c) 343 e) 8 4 0
b) 120 d) 7 2 0

2 2 . (P u c c a m p -S P ) U san d o o s a lg a r ism o s 2 , 3 , 4 , 5 , 6 . 8 e 9 . sem re p e tiç ã o , q u a n to s n ú m ero s p ares de trê s a l­


g a r ism o s e m a io res q u e 2 3 4 p o d e -se fo rm a r?

a) 110 c ) 125 e ) 1 32
b) 119 d ) 129 23

2 3 . ( U E - R J ) N u m a c id a d e , o s n ú m eros t e le f ô n ic o s n ã o pod em c o m e ç a r p or z e ro e têm o ito a lg a r ism o s , d o s


q u a is o s q u atro p rim eiro s c o n s titu e m o p re fix o .
C o n s id e re q u e o s q u a tro ú ltim o s d íg ito s d e to d a s a s fa r m á c ia s s ã o 0 0 0 0 e q u e o p r e fix o da f a rm á c ia
V iv av id a é fo rm a d o p e lo s d íg ito s 2 , 4 , 5 e 6 , n ão re p e tid o s e n ão n e c e s sa ria m e n te n e sta o rd em .
O n ú m e ro m á x im o de te n tativ a s a serem fe ita s para id e n tific a r o n ú m ero t e le f ô n ic o c o m p le to d e s sa fa rm á ­
c ia e q u iv a le a:

a) 6 c ) 64
b) 2 4 d ) 1 68

161
T E S T E S D E V E S T IB U L A R E S

24. ( IT A -S P ) Q u an to s n ú m eros de s e is a lg a r ism o s d is tin to s p o d em o s fo rm a r usando o s d íg ito s I . 2 , 3 , 4 , 5 e 6 ,


n o s q u a is o 1 e o 2 n u n ca ocu p a m p o s iç õ e s a d ja c e n te s , m as o 3 e o 4 sem p re ocu p a m p o s iç õ e s a d ja c e n te s ?

a) 144 c) 240 e) 360


b) 18 0 d) 288

2 5 . ( IT A -S P ) C o n s id e re o s n ú m ero s d e 2 a 6 a lg a r ism o s d is tin to s fo rm a d o s u tiliz a n d o -s e a p e n a s 1, 2 , 4 , 5 . 7 e


8 . Q u a n to s d e s s e s n ú m ero s sã o ím p a re s e c o m e ç a m co m um d íg ito par?
a) 3 7 5 c) 545 e) 625
b) 465 d) 585

26 . (M a ck en z ie -S P ) C onsidere todos os núm eros de cin co algarism os distintos, escritos com I, 2, 3. 4 e 5. S e esses
núm eros são ordenados em ordem crescen te, o algarism o das unidades do núm ero que ocupa a trigésim a posição é:

a) 5 c) 4 e) 2
b) I d) 3

2 7 . (U F -P 1 ) E s c re v e n d o -se em ord em d e c re s c e n te to d o s o s n úm eros de c in c o a lg a r ism o s d is tin to s fo rm a d o s


p e lo s a lg a r ism o s 3 . 5 . 7 , 8 e 9 , a ord em d o n ú m e ro 7 5 3 8 9 é :

a) 54 c ) 66 e) 56
b) 6 7 d) 55

2 8 . ( U F - C E ) O n ú m ero d e m a n e ira s seg u n d o a s q u a is p o d e m o s d isp o r 3 h o m en s e 3 m u lh e res em trê s b a n c o s


fix o s , de tal fo rm a q u e em ca d a b a n c o fiq u e um c a s a l, sem le v a r em co n ta a p o s iç ã o d o c a s a l no b a n c o , é :

a) 9 c) 24 e) 36
b) 18 d) 32

2 9 . (F u v e s t- S P ) C o m a s 6 le tra s da p a la v ra F U V E S T p od em s e r fo rm a d a s 6 ! = 7 2 0 “ p a la v r a s " (a n a g ra m a s ) de
6 le tra s d is tin ta s ca d a um a. S e e s s a s " p a la v r a s ” fo re m c o lo c a d a s e m ord em a lfa b é tic a , c o m o num d ic io n á ­
rio . a 2 5 0 “ “ p a la v ra " c o m e ç a c o m :

a) E V c ) FV e) SF
b) FU d) S E

3 0 . (P U C -P R ) D o s an a g ra m a s da p a la v ra C A S T E L O , q u a n to s têm a s v o g a is em ord em a lf a b é tic a e ju n ta s ?

a) 180 c) 120 e) 360


b) 144 d) 7 2 0

3 1 . (IT A -S P ) O núm ero d e an agram as da palavra V E S T IB U L A N D O . que não apresentam a s c in c o vogais ju n ta s, é:

a) 12! c) 1 2 ! - ( 8 !) ( 5 ! ) e ) 12! - ( 7 ! ) ( 5 ! )
b ) ( 8 !) ( 5 ! ) d ) 12! - 8!

3 2 . ( M a c k e n z ie - S P ) O n ú m ero d e fila s d ife re n te s que pod em s e r fo rm a d a s co m 2 h o m e n s e 3 m u lh e res, de


m od o que o s h o m e n s n ã o fiq u em ju n t o s , é :

a) 9 6 c) 48 e ) 1 20
b) 72 d) 8 4

3 3 . (V u n esp -S P ) Quatro am igos vão ocupar a s p oltro­


nas a. b , c, d de um ônibus dispostas na m esm a fila
horizontal, m as em lados diferentes em relação ao
corredor, con form e a ilustração.
íoTõl n o i□ »

W 9
D ois deles d esejam sentar-se ju n to s, se ja do m es­
m o lad o d o c o rred o r, s e ja em la d o s d ife re n te s .
N essas co n d içõ es, de quantas m aneiras distintas o s w w
quatro podem ocupar a s poltronas referidas, c o n s i­
derand o-se distintas as p osições em que pelo m e­
nos dois dos am igos ocupem poltronas diferen tes?
in io i m rn i
a) 24 b) 18 c) 16 d) 12 e) 6

162
T E S T E S D E V E S T IB U L A R E S

3 4 . (V u n e s p -S P ) Q u a tro a m ig o s . P e d ro . L u ís a . Jo ã o e R ita . v ão a o c in e m a , s e n ta n d o -se e m lu g a re s c o n s e c u ti­


v os na m esm a fila . O n ú m e ro de m a n e ira s q u e o s q u a tro pod em fic a r d isp o sto s de fo rm a q u e P ed ro e L u ísa
fiq u em sem p re ju n to s e J o ã o e R ita fiq u em sem p re ju n t o s é :

a) 2 c) 8 e) 24
b) 4 d) 16

(U . F. S ã o C a r lo s - S P ) C o n s id e re a tig u ra a o la d o . O n ú m ero de c a m in h o s m ais c urtos. B


a o lo n g o d as a re s ta s d os c u b o s , lig a n d o o s p o n to s Ae /?, ó: 7
i
a) 2 i
b) 4 y
c) 12 / : 1 s 0
i
d) 18 i
O'

^✓
r.

- ———-
V

>
3 6 . (U . F. Ju iz de F o ra -M G ) C in c o a m ig o s vão v ia ja r u tilizan do um ca rro com c in c o lugares. S a b en d o -se que ap e­
nas dois d eles podem d irigir, o núm ero de m an eiras que o s c in c o a m ig o s podem se a co m o d a r para a viagem é:

a) 12 c) 48
b) 24 d) 120

3 7 . (U F -P 1 ) O núm ero natural n q u e s a tis fa z a e q u a ç ã o n ! - (n - 1 )! = l()()(n - 2 )! 6:


a) 9 c) II e ) 13
b) 10 d ) 12

3 8 . ( E S P M - S P ) A o se e fe tu a r o c á lc u lo de 2 5 ! e n c o n tr a -s e um n ú m e ro in te iro te rm in a d o c o m q u a n to s z e ro s ?

a) 6 c) 4 e) 2
b) 5 d) 3

3 9 . ( U F - P I ) S e ja n o n ú m ero de c o m is s õ e s d e trê s ou m a is p e ss o a s q u e podem s e r e s c o lh id a s d e um g ru p o de


5 p e ss o a s ; o v a lo r d e n c ig u al a:

a) 14 c ) 16 e ) 18
b) 15 d ) 17

4». ( U n ifo r -C E ) S e 11 a tle ta s se c la s s ific a r e m para a fa se fin a l de um c a m p e o n a to d e b o x e , e su p o n d o que


ca d a a tle ta lu te um a ú n ic a vez c o m ca d a um d o s o u tro s, e n tã o o n ú m ero to ta l d e lu ta s q u e p o d erã o ser
re a liz a d a s e n tre os c la s s if ic a d o s será :

a) 110 c) 44 e) II!
b) 55 d ) 22

41. ( U E -P A ) N o c o n c u r s o da Q u in a da C a ix a E c o n ô m ic a F e d e ra l p o d e -se fa z e r a p o sta de 5 , 6 , 7 e 8 n ú m eros.


P re e n c h e n d o um c a rtã o co m 8 n ú m ero s o a p o sta d o r c o n c o rr e r á a o p rêm io c o m :

a ) 5 2 q u in as. c ) 5 4 q u in a s. e ) 5 6 q u in a s.
b ) 5 3 q u in as. d ) 5 5 q u in a s.

42. (U . F. S a n ta M a r ia -R S ) E m um a v ia g em de e stu d o s re a liz a d a p elo s a lu n o s d os C u rso s de M a te m á tica e E n g e ­


nharia M e c â n ica da U F S M . o b serv o u -se q u e. d os 4 0 p a ssa g e iro s. 2 5 eram c o n h e cid o s en tre s i. F e ita s a s a p re ­
s e n ta ç õ e s. o s que não se c o n h e cia m a p e rta ra m -se a s m ãos uns a o s ou tros. O n úm ero de a p e rto s de m ão é :

a) 156 c) 210 e) 480


b) 2 0 0 d) 3 0 0

43. ( P U C - R J ) O c a m p e o n a to b ra s ile iro te m , e m sua p rim eira fa s e , 2 8 tim e s q u e jo g a m to d o s e n tre s i. N e sta


p rim eira e ta p a , o n ú m ero de jo g o s é de:

a) 3 7 6 c) 380 e) 3 96
b) 378 d) 388

163
T E S T E S D E V E S T IB U L A R E S

44 . ( P U C -S P ) N o sa g u ã o d e um te a tro , h á um lu stre c o m 10 lâ m p a d a s, tod as de c o re s d is tin ta s en tre s i. C o m o


m e d id a d e e c o n o m ia d e e n e rg ia e lé tr ic a , o g e re n te d e s se te a tro e s ta b e le c e u que só d e v e ría m s e r a c e sa s ,
s im u lta n e a m e n te , de 4 a 7 lâ m p a d a s, de a c o rd o c o m a n e c e s sid a d e . N e ssa s c o n d iç õ e s , d e q u a n to s m odos
d is tin to s p odem s e r a c e s a s a s lâ m p a d a s d e sse lu s tre ?

a) 6 6 4 d) 9 1 2
b) 7 9 2 e) I 044
c) 852

45 . (P U C -S P ) B u sc a n d o m elh o ra r o d esem p en h o de seu tim e . o té c n ic o de um a s e le ç ã o de fu teb o l d ecid iu in o


var: co n v o co u ap en as 15 jo g a d o r e s , 2 dos q u a is só jo g a m no gol e o s d e m a is atuam em q u a isq u er p o siçõ es,
in clu siv e no g o l. D e qu an tos m od os e le pode s e le c io n a r o s 11 jo g a d o re s que irão c o m p o r o tim e titu la r?

a) 4 5 0 d) 5 8 0
b) 4 8 0 e) 6 5 0
c) 550

4 6 . (LI. F. U b e r lâ n d ia -M G ) C o n s id e re n ove b a rra s de m etal q u e m ed em , re s p e ctiv a m e n te : 1, 2 , 3 , 4 , 5 , 6 , 7 , 8


e 9 m etro s. Q u an tas c o m b in a ç õ e s de c in c o b a rra s, o rd en a d a s em ord em c re s c e n te de c o m p rim e n to , podem
s e r fe ita s d e tal fo rm a q u e a b a rra de 5 m e tro s o c u p e sem p re a q u a rta p o s iç ã o ?

a) 3 2 c) 20 e) 1 20
b) 16 d ) 18

4 7 . ( U F - R N ) E m virtu d e d e u m a c r is e fin a n c e ir a , um a fá b r ic a d isp õe de a p e n a s q u a tro v ig ila n te s para o c u p a ­


rem s e te p o sto s de v ig ilâ n c ia .
C o n s id e ra n d o q u e . em c a d a p o sto . Fica, n o m á x im o , um v ig ila n te e que o posto de en tra d a p rin cip a l não
p od e fic a r d e s g u a rn e c id o , in d iq u e a o p ç ã o c o rre s p o n d e n te a o n ú m ero de m a n e ira s d is tin ta s de que o c h e fe
de se g u ra n ç a p od e d isp o r para d is trib u ir o s v ig ila n te s .
O b s .: D u as m a n e ira s s ã o d ita s id ê n tic a s s e , e m a m b a s , o s v ig ila n te s o cu p a m o s m e sm o s p o s to s e ca d a p os­
to é o c u p a d o p e lo m e sm o v ig ila n te ; c a s o c o n trá r io , s ã o d ita s d is tin ta s.

a) 35 c) 4 8 0
b) 80 d) 840

4 8 . ( U n ir io -R J) O b u fê de sa la d a s d e um re sta u ra n te a p re s e n ta a lf a c e , to m a te , a g r iã o , c e b o la , p ep in o , b e te rra ­
b a e c e n o u ra . Q u an to s tip o s d e sa la d a s d ife re n te s pod em s e r p rep a ra d o s c o m c in c o d e s s e s in g re d ie n te s, de
m od o q u e to d as as sa la d a s c o n ten h a m a lf a c e , to m a te e c e b o la ?

a) 4 c) 8 e) 6
b) 12 d) 3

49. ( U n if e s p - S P ) O c o rp o c l ín i c o da p e d ia tria d e um c e rto h o sp ita l é c o m p o s to de 12 p r o fis s io n a is , d os q u a is 3


sã o c a p a c ita d ó s para a tu a ç ã o ju n t o a c r ia n ç a s q u e a p rese n ta m n e c e s sid a d e s e d u c a c io n a is e s p e c ia is . Para
fin s de a s s e s s o r ia , d ev erá s e r c ria d a um a c o m is s ã o d e 3 p r o fis s io n a is , de tal m a n e ira q u e I d e le s, p elo
m e n o s, te n h a a c a p a c ita ç ã o re fe rid a . Q u a n ta s c o m is s õ e s d is tin ta s pod em s e r fo rm a d a s n e sta s c o n d iç õ e s ?

a) 7 9 2 d) 136
b) 4 9 4 e ) 1 08
c) 369

5 0 . ( E S P M - S P ) N o te to de um sa lã o há 6 lâm p ad as que sã o c o m an d a d a s p or 6 in terru p tores in dep en d en tes uns


d os ou tro s. E x is te m N m an eiras d ife re n tes d e sse s a lã o e sta r ilu m in ad o por e ss a s lâm p ad as. O v a lo r de N é:
a) 6 c ) 21 e) 63
b) 12 d) 3 6

5 1 . ( M a c k e n z ie - S P ) S e .r é in te iro tal que |x| < 10, e n tã o o n ú m ero de fo rm a s d e se e s c o lh e re m trê s v a lo re s de


x c o m so m a par é:

a) 5 2 7 d) 4 0 5
b) 4 8 9 e) 600
c) 432

164
T E S T E S D E V E S T IB U L A R E S

52. (U . E . L o n d r in a -P R ) U m a a p o sta n a M e g a S e n a (m o d a lid a d e d e a p o sta s da C a ix a E c o n ô m ic a F e d e ra l) c o n ­


s is te na e s c o lh a de 6 d e n tre o s 6 0 n ú m ero s de 01 a 6 0 . O n ú m ero m á x im o d e a p o sta s d ife re n te s , c a d a um a
d e la s in c lu in d o o s n ú m ero s 1 2 . 2 2 e 2 3 , é igu al a:

6 0 -5 9 -5 8 5 7 •5 6 •5 5
a) d)
1 -2 -3 1 -2 -3

6 0 •5 9 •5 8 •5 7 •5 6 •55 5 7 •5 6 •5 5 •5 4 •5 3 •5 2
b) e)
1 •2 •3 •4 •5 •6 1 • 2 • 3 •4 •5 •6

f 60 -5 9 -5 8 5 7 -5 6 -5 5 1
c)

53. ( IT A -S P ) Q u an to s an a g ra m a s c o m 4 le tra s d is tin ta s p o d em o s fo rm a r co m a s 10 p rim e ira s le tra s do a lfa b e to


e q u e c o n ten h a m 2 das le tra s a, b e c?

a) 1 692 c) I 520 e) 1 392


b) I 572 d) I 512

54. ( F u v e s t - S P ) U m a c la s s e de E d u c a ç ã o F í s i c a d e um c o l é g i o é fo rm a d a p o r d e z e s tu d a n te s , to d o s c o m
a ltu r a s d ife r e n te s . A s a ltu r a s d o s e s tu d a n te s , e m o rd em c r e s c e n t e , s e r ã o d e s ig n a d a s p o r h ,, h 2, . .. . h K>
(h| < hi < ... < h9 < h|0 ). O p r o fe s s o r vai e s c o lh e r c i n c o d e s s e s e stu d a n te s p a ra p a r tic ip a r d e um a
d e m o n s t r a ç ã o n a q u a l e l e s se a p r e s e n t a r ã o a l i n h a d o s , e m o r d e m c r e s c e n t e d e s u a s a l t u r a s . D o s

?j = 2 5 2 g ru p o s q u e p od em s e r e s c o lh id o s , em q u a n to s o e stu d a n te c u ja a ltu r a é h 7 o c u p a r á a p o s i­

ç ã o c e n tr a l d u ran te a d e m o n s tra ç ã o ?

a) 7 c ) 21 e) 60
b) 10 d) 4 5

55. (U . E . L o n d r in a -P R ) S ã o d a d o s n p o n to s, d o is a d o is d is tin to s e n tre s i. 4 d o s q u a is p e rte n c e m a u m a re ta r


e o s d e m a is e n c o n tr a m -se so b re u m a re ta p a r a le la a r. S e podem s e r c o n s tru íd o s 1 2 6 q u a d rilá te ro s co m
v é rtic e s n e ss e s p o n to s, e n tã o né um n ú m ero:

a ) q u ad rad o p e rfe ito . c ) m ú ltip lo d e 7 . e) m a io r q u e 15.


b ) p rim o . d ) m e n o r q u e 10 .
I &
56. ( M a c k e n z ie - S P ) O s p o líg o n o s d e k la d o s (k m ú ltip lo d e 3 ) q u e pod em os 1
o b te r c o m v é rtic e s n o s 9 p o n to s da Figura sã o e m n ú m ero de:

a) 83
b) 84
c ) 85
d) 168
e) 169

57. ( C e f e t -P R ) U m p lan o tem 7 p o n to s (A, B, C, D, £. F e G)


c o n fo r m e a fig u ra a o la d o . S e n d o a s s im , o n ú m e ro de
p o líg o n o s c o n v e x o s q u e c o m e le s p o d em o s fo rm a r é :

a) l
b) 64
c) I 20
d) 85
e) 99

165
T E S T E S D E V E S T IB U L A R E S

58. ( C e f e t -M G ) O n ú m ero d e re ta s d e fin id a s p o r o ito p o n to s c o p la n a re s . co m trcs d e le s n u n ca a lin h a d o s , é :

a) 2 0 b) 2 8 c) 35 d) 4 0 e) 70

59. ( F a t e c - S P ) Em um a O lim p ía d a , a d e le g a ç ã o d e um p a ís A se a p rese n to u c o m 1 0 a tle ta s e a d e um a país


B. c o m 6 a tle ta s . O s a lo ja m e n to s da V ila O lím p ic a e ra m para q u a tro p e s s o a s , e um d e le s fo i o c u p a d o por
2 a tle ta s de Ae 2 a tle ta s d e B.
O n ú m ero d e m a n e ira s d is tin ta s d e fo rm a r e s s e g ru p o d e 4 a tle ta s e ra :

a) 675 b) 4 5 0 c) 270 d) 6 0 e) 16

6 0 . ( U C D B - M S ) U m a p e ss o a se e sq u e c e u de su a sen h a b a n c á ria e se le m b ra a p en a s q u e é fo rm a d a p or q uatro


a lg a r ism o s d is tin to s, sen d o trc s p a res e um ím par.
C o m b a se n e ssa in fo rm a ç ã o , c o n c lu í-s e q u e o n ú m ero de p o s sib ilid a d e s para le m b ra r a sen h a é igu al a:

a) I 200 c) I 800 e) 2 400


b) I 500 d) 2 100

61. ( U n ir io -R J) C om o s a lg a rism o s d e I a 9 , o total de n ú m eros de 4 a lg a rism o s d ife re n te s, fo rm a d o s por 2 a lg a ­


rism o s p ares e 2 ím p ares, é igual a:

a) 12 6 c) 720 e) 5 760
b) 504 d) I 4 4 0

62. ( U n if e s p - S P ) E m um e d if íc io re s id e n c ia l d e S ã o P a u lo , o s m o ra d o res fo ra m c o n v o c a d o s para um a reu n iã o ,


c o m a fin a lid a d e de e s c o lh e r um s ín d ic o e q u a tro m e m b ro s do c o n s e lh o fis c a l, se n d o p ro ib id a a a cu m u la ­
ç ã o d e c a rg o s . A e s c o lh a d ev erá s e r fe ita e n tre d ez m ora d o re s.
D e q u a n ta s m an e ira s d ife re n te s será p o s sív e l fa z e r e s ta s e s c o lh a s ?

a) 64 c) 252 e) I 260
b) 126 d) 6 4 0

63. (U . F. S anta M a ria -R S ) Numa C âm ara de V ereadores, trabalham 6 vereadores do partido A. 5 vereadores do parti­
do Be 4 vereadores do partido C. O núm ero de co m issõ es de 7 vereadores q ue podem ser form adas, devendo cada
co m issão ser constituída de 3 vereadores do partido A, 2 do partido Bc 2 vereadores do partido C . c igual a:

a) 7 c) 152 e) 28 800
b) 36 d) I 200

64. ÍU . F. S ã o C a r lo s - S P ) Num a c a m p a m e n to e s tã o 14 jo v e n s , sen d o 6 p a u lista s. 4 c a r io c a s e 4 m in e iro s . Para


fa z e r a lim p e za d o a c a m p a m e n to , será fo rm a d a u m a e q u ip e co m 2 p a u lista s, I c a r io c a e I m in e iro , e s c o ­
lh id o s ao a c a s o . O n ú m ero de m a n e ira s p o s s ív e is p ara se fo rm a r e s s a e q u ip e de lim p e z a é:

a) 9 6 c) 212 e) 256
b) 182 d) 2 4 0

65. ( M a c k e n z ie - S P ) U m ju iz d isp õ e de 10 p e s s o a s , d as q u a is so m en te 4 s ã o ad v o g a d o s, p ara fo rm a r um ú n ico


jú r i c o m 7 ju r a d o s . O n ú m ero de fo rm a s de c o m p o r o jú r i, c o m p e lo m en os I a d v o g a d o , c :

a) 120 c ) 160 e) 1 28
b) 108 d ) 140

6 6 . ( F u v e s t- S P ) U m a O N G d e c id iu p rep arar s a c o la s , c o n te n d o 4 ite n s d is tin to s ca d a . para d is trib u ir en tre a


p o p u la çã o c a re n te . E s s e s 4 ite n s d evem s e r e s c o lh id o s e n tre 8 tip o s d e p rod u tos d e lim p e z a e 5 tip o s de
a lim e n to s n ão p e re c ív e is . E m c a d a s a c o la d ev e h aver p e lo m en os um item q u e s e ja a lim e n to n ão p e re cív el
e p e lo m e n o s um item q u e s e ja prod u to d e lim p e z a . Q u a n to s tip o s d e s a c o la s d is tin ta s pod em s e r fe ito s ?

a) 3 6 0 c) 540 e) 640
b) 4 2 0 d) 6 0 0

67. (U . F. S a n ta M a r i a -R S ) D e q u a n ta s m a n e ira s d is tin ta s p o d e m -se a lin h a r c in c o e s ta c a s a z u is id ê n tic a s, um a


v e rm e lh a c u m a b ra n c a ?

a) 12 c) 42 e) 5 040
b) 3 0 d) 240

166
T E S T E S D E V E S T IB U L A R E S

68 . ( U F - A M ) O n ú m ero de an a g ra m a s da p a la v ra G R E V E é :

a) 120 c) 20 e) 30
b) 60 d) 4 0

69. (C e s g r a n r io - R J ) Na fig u ra a o la d o . te m o s u m a “ m a lh a "


fo rm ad a p or 16 re tâ n g u lo s ig u a is.
U m a p a rtíc u la deve ir d o p o n to P a o pon to M. p e rc o r­
ren d o a m e n o r d is tâ n c ia p o s sív e l, d e s lo c a n d o -s e so m e n ­
te p or so b re as lin h as da figu ra e co m v e lo c id a d e m éd ia
d e 2 cm/s. C o m o e x e m p lo , te m o s, a o la d o . um a re p re ­ P
s e n ta ç ã o d e um d e s se s c a m in h o s .
Q u a n to s sã o o s p o s sív e is c a m in h o s q u e tal p a rtíc u la p o ­ ------------------- M
____________ 1------------------
d erá p e rc o rre r?

a) 2 5 6 d) 70
b) 12 8 e) 56
c) 120 P

70. ( U F - R S ) N o d e s e n h o a o la d o . a s lin h a s h o r iz o n ta is e
v e rtic a is rep resen tam ru as, e o s q u ad rad os rep resen ta m
q u a rte irõ e s . A q u a n tid a d e d e tr a je to s d e c o m p r im e n to
m ín im o lig a n d o Ac B q u e p assa m p or C c:

a) 12 d) 24
b) 13 e) 30
c) 15

7 1 . ( E S P M - S P ) U m a p rop agan d a diz o s eg u in te:

“ N ão s e ja b o to c u d o nem h o te n to te ,
lav e sua roupa s u ja em c a s a m e sm o .
E sem p re c o m um a lavad ora B r a ste m p o ”

Sabe-se que o número de botocudos e de hotenlotes coincide, respectivamente, com o número de anagra­
mas das palavras BOTOCUDO e HOTENTOTE.
R e u n in d o -s e to d o s o s b o to c u d o s e h o te n lo te s , q b té m -s e um c o n ju n to d e , n o m á x im o , q u a n to s in d iv íd u o s?

a) 4 0 3 2 0 0 c) 134 4 0 0 1 e) 2 1 8 4 0
b ) 201 6 0 0 d) 6 7 2 0 0

7 2 . (M a c k e n z ie - S P ) 9 p e ss o a s d e s e ja m s u b ir à c o b e rtu ra d e um e d if íc io , d isp o n d o , para is s o . de d o is e le v a d o ­


re s . um co m 4 lu g are s e o u tro c o m 5 lu g a re s. O n ú jn e ro de fo rm a s de d is trib u í-la s n o s e le v a d o re s é :

a) 6 3 0 c ) 180 e) 126
b) 252 d) 3 7 8

7 3 . (M a ck e n / .ie -S P ) 6 re frig e r a n te s d ife re n te s devem s e r d is trib u íd o s e n tre 2 p e ss o a s , de m od o q u e c a d a p e s­


so a re c e b a 3 re frig e r a n te s . O n ú m ero de fo rm a s de se fa z e r is so é :

a) 12 c) 24 e) 20
b) 18 d ) 15

7 4 . ( U F - M G ) O jo g o d e d o m in ó p ossu i 2 8 p e ç a s d is tin ta s. Q u a tro jo g a d o r e s rep artem e n tre si e s s a s 2 8 p e ç a s,


fica n d o ca d a um co m 7 p e ç a s.
D e q u an tas m an e iras d is tin ta s se p od e fa z e r tal d is trib u iç ã o ?

28! 28!
a) c)
7 !4 ! ( 7 ! >4

28! 28!
b) d)
4 !2 4 ! 7 !2 I!

167
T E S T E S D E V E S T IB U L A R E S

75. ( F E I - S P ) N um s e to r d e um a fá b r ic a q u e s e rá d e sa tiv a d o , tra b a lh a m 8 o p e rá rio s. D e s s e s 8 fu n c io n á rio s , 3


s e r ã o d e m itid o s , 2 serã o tra n fe rid o s para o u tro s e to r da m e sm a fá b r ic a e o u tros 3 s e r ã o d e s lo c a d o s para
o u tra unid ade d aq u e la in d ú stria . D e q u a n ta s m a n e ira s d ife re n te s podem s e r fe ita s a s e s c o lh a s ?

a) 2 8 0 c) 640 e) 400
b) 5 6 0 d) 360

Binômio de Newton
76. (P U C -P R ) O v a lo r da e x p re ss ã o 1 0 3 4 - 4 • 1 0 3 3 • 3 + 6 • I 0 3 2 • 3 2- 4 •1 0 3 • 3 3+ 3 4 é ig u al a:

a) IO14 c) IO10 e) 106


b) I 0 12 d) IO 8

77. ( F G V - S P ) S a b e n d o que x e v s ã o n ú m e ro s p o s itiv o s , x - y = I e x 4 + 4 x 3y + 6 x 2y 2 + 4 x y 3 + y 4 = 16,


p o d em o s c o n c lu ir q ue:

a) x = — e) x =
6 c> x = 7
4
b) x = -j d) x = T

78. ( M a c k e n z ie - S P ) C o n s id e re a e q u a çã o

(o) - 2)5 + [ |) (x ” 2 )4 + ( 2) ~ + ***+ [ 5 ) = ( 7 x ~ l 3 )5, e n tã o (x _ 2)6 v a ,e :

a) 26 c) 56 e) 4 6
b) 0 d ) 66

7 9 . (M a c k e n z .ie -S P ) E m |0, 2 jü|. s e a é a m a io r ra iz da e q u a çã o

2
8(1. ( U F - C E ) O c o e fic ie n te de x 3 no p o lin ô m io p (x ) = (x - I ) • (x + 3 )5 é :

a) 3 0 c) 100 e) 180
b) 5 0 d) 120

8 1 . ( U F - C E ) S e ja m a e p n ú m ero s re a is . S u p o n h a q u e. a o d e sen v o lv erm o s ( a x + p y )5, o s c o e fic ie n te s d os


m o n ô m io s x 4y e x y s e ja m ig u a is a 2 4 0 e 7 2 0 . re sp e ctiv a m e n te . N e ssa s c o n d iç õ e s , a s s in a le a o p ç ã o que

c o n té m o v alo r de

_1_
a)
2 3
b) 2 d) 3
2

8 2 . (P U C -P R ) S a b e n d o q u e o d e se n v o lv im e n to de p o ssu i 7 te rm o s e q u e um d e le s c 2 4 0 a x h,

a c h a re m o s para a o v alor:

4_
a) o JL
9 9
b)
2_ d)
9 3

168
TESTES DE VESTIBULARES

/ 2 Vo
83. (U n if o r - C E ) N o d e s e n v o lv im e n to d o b in ô m io l x 3 + — y l . o te rm o in d ep en d en te d e x é:

a) 15 7 6 0 d) 5 7 8 0
b) 13 4 4 0 e) 3 3 6 0
c) 8 064

84. ( U C D B - M S ) N o d e s e n v o lv im e n to de + 2 x 2 j , o te rm o in d ep en d en te de .v é:

a ) 20 c) 60 e) 172
b) 32 d) 6 4

' 1 3'
1 3 - f
85. (U . F. S an ta M a ria -R S ) D ad as a s m a triz es M e / V MM= : e N = -2 1 on d e mé o te rm o in d e­
2 -3 m
1 0

p endente d o d e sen v o lv im en to d o b in ô m io í—+ x" j e n tã o o determ in an te da m atriz Q = M • N c igu al a:

a) 15 d) - 1 2 6
b) 126 e) -1 5 6
c) 374

86 . ( F G V - S P ) A so m a d o s c o e fic ie n te s do d e s e n v o lv im e n to d e ( 2 x + y )5 c igu al a:
a ) 81 d) 5 1 2
b) 128 e) 7 29
c) 243

87. ( IT A -S P ) S ab en d o que é de I 0 2 4 a som a d os c o e fic ie n te s d o p o lin ô m io em x e y. o b tid o p e lo d e sen v o lv i­


m ento do b in ô m io (x + y )n. te m o s q u e o n úm ero de a rra n jo s sem re p e tiç ã o de n e le m e n to s , tom ad os 2 a 2 , é :
a) 80 d ) 100
b) 90 e) 60
c) 70

88 . (U . F. O u ro P r e to -M G ) A c o n d iç ã o para q u e o b in ó tiiia l ( n J s e ja o d o b ro do b in o m ia l 6:
k - 1
a) n = 2k c ) n = 3k + 1
b ) n = 3k d) n = 3k - I

2n
8 9 . ( IT A -S P ) A re s p e ito d as c o m b in a ç õ e s a n = e bn te m o s q u e . para ca d a n = I. 2 . 3 , .

d ife re n ç a an - b n é igu al a:
(!:
a) d)
n + 1 n + 1
2n 1
b) e)
n + I n + 1

c)
n + 1

9 0 . ( IT A -S P ) C o n s id e re o c o n ju n to S = { ( a . b ) E N X N : a + b = 1 8 } . A so m a de to d o s o s n ú m ero s da fo rm a

18!
V (a . b ) E S , é:
a !b ! '
a ) 86 d ) 12 ft
b) 9 ! e) 12!
c) 96

169
TESTES DE VESTIBULARES

9 1 . (U n if o r - C E ) S o b re as se n te n ç a s

é c o r r e to a firm a r q u e:

a ) s o m en te I é v erd ad eira. d ) so m e n te I e II sà o v erd ad eiras.


b ) s o m e n te II é v erd a d eira . e) I. II e III s à o v erd ad eiras.
c ) s o m en te III 6 v erd ad eira.

n + I
9 2 . ( U F - S E ) S e o q u in to te rm o da se q ü ê n c ia é ig u al a 12 6 . e n tã o o nú-

m e ro n c:

a) ím par. d ) d iv is ív e l p or 5 .
b ) m en o r q u e 6 . e ) m ú ltip lo de 3.
c) um c u b o p e rfe ito .

fio ^ n o '!
( P U C - R J ) A so m a a lte rn a d a

2 io
lo)-(.)+ c) 10
de c o e fic ie n te s b in o m ia is v a le:

e) 0
a)
b ) 20 d) 10 !

Probabilidade

9 4 . ( F G V - S P ) U m a fa lia d e p ã o c o m m a n te ig a p o d e c a ir no c h ã o d e du as m a n e ira s a p e n a s:

• c o m a m a n te ig a para c im a (e v e n to A );
• c o m a m a n te ig a para b a ix o (e v e n to B ).
U m a p o ssív e l d is trib u iç ã o de p ro b a b ilid a d e para e s s e s e v e n to s é :

a) P (A ) = P ( B ) = — d ) P (A ) = 0 .4 e P ( B ) = 0 , 6

b) P (A ) = 0 c P ( B ) = y e) P (A )= y e P ( B ) = <)

c) P(A) = - 0 ,3 e P(B) = 1.3

9 5 . ( U F - P I ) N o la n ç a m e n to de um d ad o v ic ia d o , a s fa c e s d ife re n te s de 5 o c o rre m c a d a um a c o m p ro b a b ilid a d e


p. e n q u a n to a fa c e 5 o c o rr e c o m a p ro b a b ilid a d e 3 p . A ss im sen d o , o v a lo r de p é:

a) -L c) — e)
s 8

b) 2 . d) ±
8 8
96. (U n ip - S P ) E m um a u m a há 10 b o la s id ê n tic a s n u m erad as de I a 10. S e re tira rm o s u m a b o la da u m a . a
p ro b a b ilid a d e de n ã o o b te r a b o la n ú m ero 7 c igu al a:

2
a) O —
9 II
1 d, JL
b)
10 10

170
T E S T E S D E V E S T IB U L A R E S

9 7 . ( U F - P E ) Um s a c o c o n lé m 12 b o la s v erd es e 8 b o la s a m a re la s . Q u a n ta s b o la s a z u is d ev em s e r c o lo c a d a s no

s a c o . d e m od o q u e a p ro b a b ilid a d e de re tira rm o s d o m e sm o , a le a to ria m e n te , um a b o la a z u l. s e ja — ?

a) 5 c) 20 e) 40
b) 10 d) 3 0

98. (C e s g r a n r io - R J ) N um a c a ix a s à o c o lo c a d o s v á rio s c a rtõ e s , a lg u n s a m a re lo s , a lg u n s v erd es e o s re sta n te s


p reto s. S a b e - s e q u e 5 0 % d o s c a rtõ e s s ã o p r e lo s e q u e. para c a d a trê s c a rtõ e s v erd es, há 5 c a rtõ e s p reto s.
R e tir a n d o -s e a o a c a s o um d e s se s c a rtõ e s , a p ro b a b ilid a d e d e q u e e ste s e ja a m a r e lo é de:

a) 10 % c) 20% e) 40%
b) 15 % d) 25%

99. ( F G V - S P ) U m a c a ix a c o n té m I 0 0 0 b o lin h a s n u m erad as de I a I 0 0 0 . U m a b o lin h a é so rte a d a . A p r o b a b i­


lid ad e de a b o lin h a so rte a d a le r um n ú m ero m ú ltip lo de 7 é :

a) 0 .1 3 9 c ) 0 .1 4 1 e ) 0 ,1 4 3
b ) 0 ,1 4 0 d ) 0 .1 4 2

100. ( U F - R N ) “ B lo c o s L ó g ic o s ” é um a c o le ç ã o d e p e ç a s u tiliz a d a n o e n s in o de M a te m á tic a . S ã o 4 8 p e ç a s


c o n s tru íd a s c o m b in a n d o -se 3 c o re s (a z u l, v e rm e lh a e a m a r e la ). 4 fo rm a s (tria n g u la r, q u a d ra d a , re ta n g u la r
e c ir c u la r ), 2 ta m a n h o s (g ra n d e e p e q u e n o ) e 2 e sp e s su ra s (g ro s s a e fin a ). C a d a p e ça tem a p e n a s u m a c o r,
u m a fo rm a , um ta m a n h o e um a e sp e s su ra .
S e u m a c r ia n ç a p e g a r um a p e ç a . a le a to ria m e n te , a p ro b a b ilid a d e d e s sa p e ça s e r a m a re la e g ra n d e é :

a) O ±
12 3
J_
b)
6
101. ( U E - R J ) O s núm eros naturais de I a 10 foram e sc rito s, um a um , sem rep etição, em dez b o la s de pingue-pongue.
S e duas d elas forem e sco lh id a s a o a c a so , o valor m ais provável da som a dos núm eros sortead os é igual a:

a) 9 c) II
b) 10 d) 12

102. ( P U C -S P ) U m a u m a c o n lé m b o la s n u m e ra d a s d,e I a 5 . S o r te ia -s e um a b o la. v e rific a -s e o seu n ú m ero e ela


é re p o s ta na urna. 4 _ <
N um seg u n d o s o rte io , p r o c e d e -se da m esm a fo rm a q u e n o p rim eiro so rte io . A p ro b a b ilid a d e d e q u e o nú-
m e ro da seg u n d a b o la s e ja e s trita m e n te m a io r q u e o da p rim eira é:

1
a, ± d)
5 25
s 15
b) 2- \ e)
5 25

c) —
5

1 0 3 . ( U C D B - M S ) U m grupo d e 1(K) p e sso a s a p rese n ta a s s e g u in te s c a r a c te r ís tic a s :

Mulheres Homens
Estudantes 15 35
Não estudantes 5 45
S e n d o e s c o lh id a a o a c a s o u m a p e ss o a d e sse g ru p o, a p ro b a b ilid a d e d e s e r m u lh e r estu d a n te ou h om em não
e stu d a n te é :

a) J - d)
10 5
3
b) J- e)
2 20
O ±
5

171
T E S T E S D E V E S T IB U L A R E S

104. (V u n e s p -S P ) Em um c o lé g io fo i re a liz a d a um a p esq u isa so b re as atividades e x tra cu rricu la re s de seu s alu n os.
D o s 5 0 0 alu n os e n tre v ista d o s. 2 4 0 p raticavam um tipo de e sp o rte . 1 8 0 freqü en tavam um c u rso de id io m a s e
120 realizav am e sta s duas ativ id a d es, ou s e ja . praticavam um tipo de esp orte e freqü en tavam um c u rso de
id io m as. S e n e sse grupo de 5 0 0 estu d an tes um é e sc o lh id o a o a c a so , a p robab ilid ad e de q u e e le re a liz e pelo
m en os um a d e ssa s ativ id a d es, isto é , pratiq u e um tip o de e sp o rte ou freqü en te um c u rso de id io m a s, é:

18
O ü e)
2_
25 25 5
3
d) £
5 25

105. ( U F - R J ) A ta b e la a b a ix o fo rn e c e o n ú m ero de e stu d a n tes m a tric u la d o s p or s e x o e c u r s o , n o C o lé g io T é c n i ­


c o d a U F R R J no ano 2 0 0 0 .

Sexo
Curso
Homens Mulheres
Ensino M édio Regular 30 52
T écn ico em Econom ia D om éstica 2 100
T écnico em Agropecuária 132 120

A o e s c o lh e r um a lu n o , a p r o b a b ilid a d e d e o m e sm o s e r d o s e x o fe m in in o o u d o C u r s o T é c n i c o em
A g ro p e c u á ria é :

'1 33 C) 101
J f)
109 109
98 108
b) d)
109 109

106. (E n e m -M E C ) E m um c o n c u r s o d e t e le v is ã o , a p re s e n ta m -s e a o p a rtic ip a n te três fic h a s v o lta d a s para b a ix o ,


e sta n d o re p rese n tad a s em ca d a um a d e la s a s le tra s T, V e E. A s fic h a s e n c o n tra m -se a lin h a d a s em um a or­
dem q u alq u er. O p a rtic ip a n te d ev e o rd en a r a s fic h a s a seu g o s to , m an ten d o a s le tra s v o lta d a s para b a ix o ,
ten tan d o o b te r a s ig la T V E . A o d e s v irá -la s , p a ra c a d a le tra q u e e s te ja na p o s iç ã o c o rr e ta , g a n h a rá um prê­
m io d e R $ 2 0 0 .0 0 .
A p ro b a b ilid a d e d e o p a rtic ip a n te n ã o g a n h a r q u a lq u e r p rêm io é igu al a:

i
a) 0 c)
4

b) — d) 2
3 2
107. (U m e s p -S P ) N um jo g o de d a rd o s , um a lv o é fo rm a d o p or trê s fig u ra s
c o n c ê n tr ic a s , a p rim eira d e ra io ig u al a 2 c m . a seg u n d a de ra io igu al a
4 c m e a te r c e ir a d e r a io ig u a l a 8 c m . S u p o n d o q u e a p e ss o a sem p re
a c e rte n o a lv o , a p ro b a b ilid a d e de a c e rta r na á re a que p e rte n c e s o m en te ü
c irc u n fe rê n c ia m a io r é de:

a)
1 d)
10 2
b)
J_
e) 2
4 4

c)
4_
7

108. ( F G V - S P ) A á re a da s u p e rfíc ie da T e rra é d e a p ro x im a d a m e n te 5 1 0 m ilh õ e s d e k m 2. U m s a té lite a rtific ia l


d ir ig e -s e a le a to ria m e n te para a T e rra . Q u al a p ro b a b ilid a d e de e le c a ir num a c id a d e c u ja s u p e rfíc ie tem
á re a ig u al a 102 k n r ?

a ) 2 • I 0 -9 c) 2 • I0 ' 7 e) 2 • I 0 "5
b ) 2 • I 0 “8 d ) 2 • IO -6

172
T E S T E S D E V E S T IB U L A R E S

109. (E n e m -M E C ) N um d e te r m in a d o b a ir r o h á d u a s e m p re s a s d e
H orário dos ônibus
ô n ib u s , A n d a b e m e B o m p a s s e io , q u e fa z e m o t r a je t o le v a n d o
e tra z e n d o p a s s a g e ir o s d o s u b ú r b io a o c e n tr o da c id a d e . Um Andabem Bompasseio
ô n ib u s de c a d a u m a d e s s a s e m p re s a s p a r te do te rm in a l a c a d a
3 0 m in u to s , n o s h o r á r io s in d ic a d o s na t a b e la a o la d o .
C a r lo s m o ra p r ó x im o a o te rm in a l d e ô n ib u s e tr a b a lh a n a c i ­
óhOOmin 6hl0m in
d a d e . C o m o n ão te m h o ra c e r t a p a ra c h e g a r a o tra b a lh o nem 6h30min 6h40min
p r e f e r ê n c ia p o r q u a lq u e r d a s e m p r e s a s , to m a s e m p re o p r i­
7h00min 7hl0m in
m e ir o ô n ib u s q u e s a i d o t e r m in a l. N e s s a s i t u a ç ã o , p o d e -s e
a f ir m a r q u e a p r o b a b ilid a d e d e C a r lo s v ia ja r num ô n ib u s da 7h30min 7h40min
e m p re s a A n d a b em é :

a ) um q u a rto da p ro b ab ilid a d e de e le v ia ja r num ô n ib u s da e m p re sa B o m p a s s e io .


b ) um te rç o da p ro b ab ilid a d e d e e le v ia ja r num ô n ib u s da e m p re sa B o m p a s s e io .
c) m etad e da p ro b a b ilid a d e de e le v ia ja r num ô n ib u s da e m p re sa B o m p a s s e io .
d ) du as v e z e s m a io r d o q u e a p ro b a b ilid a d e de e le v ia ja r num ô n ib u s da e m p re sa B o m p a s s e io .
e ) trê s v e z e s m a io r d o q u e a p ro b a b ilid a d e de e le v ia ja r num ô n ib u s da e m p re sa B o m p a s s e io .

1 10. (U . F. U b e r lâ n d ia -M G ) A o p re e n c h e r o fo rm u lá rio de in s c r iç ã o d o v e stib u la r de um a d e term in a d a u n iv ersi­


d a d e , d e n tr e o s 12 c u r s o s d if e r e n t e s o f e r e c i d o s , o c a n d id a to d e v e in f o r m a r o s 3 a o s q u a is e s t á se
can d id a ta n d o , in d ica n d o a ord em de p r e fe rê n c ia (p rim e ira , segu n d a e te rc e ira o p ç õ e s ). O n ú m ero d e m a­
n e ira s d ife re n te s em que o fo rm u lá rio pode s e r p ree n c h id o e a p ro b a b ilid a d e de que o c u r so de E n g e n h a ria
C iv il, um d os c u r so s o fe r e c id o s , fig u re c o m o um a d as o p ç õ e s em um fo rm u lá rio p re e n c h id o , a le a to ria m e n ­
te , sã o re s p e ctiv a m e n le ig u a is a:

a) I3 2 0 e — c) 1 320 e —
12 4

b) 220 e — d) 220 e —
4 12

( F E I - S P ) Para d e s lig a r-se um s iste m a de seg u ra n ça , devem s e r a c io n a d o s sim u lta n e a m en te 3 d e term in a d o s


b o tõ e s de um p ain el que p ossu i 5 b o tõ e s. Q u al a p ro b a b ilid a d e de d e s lig a r-se o s iste m a e sc o lh e n d o -s e a le a ­
to riam en te o s 3 b o tõ e s?

a) — O -L e) JL
24 10 4

b) —
16

112. (U . F. J u iz de F o r a -M G ) U m so ld a d o d o e sq u a d rã o a n tib o m b a s ten ta d e sa tiv a r um c e r to a rte fa to e x p lo siv o


q u e p ossu i 5 fio s e x p o sto s . Para d e s a tiv á -lo , o so ld a d o p re c is a c o rta r 2 fio s e s p e c ífic o s , um d e c a d a vez,
em u m a d e term in ad a ord em . S e c o rta r um fio erra d o ou na ord em e rra d a , o a rte fa to e x p lo d irá . S e o so ld a ­
do e s c o lh e r a le a to ria m e n te 2 fio s p ara c o rta r , num a d a te rm in a d a o rd em , a p ro b a b ilid a d e de o a rte fa to não
e x p lo d ir ao c o r tá -lo s é igu al a:

2 2 9
a) c) e)
~25~ 5 20
1 1
b) d)
20 10
113. ( U F - R N ) Para a c e s s a r o s iste m a de c o m p u ta d o re s da e m p re s a , ca d a fu n c io n á rio d ig ita su a s e n h a p e ss o a l,
fo rm ad a por 4 le tra s d is tin ta s do n o s so a lfa b e to (q u e p o ssu i 2 3 le tra s ), n u m a ord em p r e e s ta b e le c id a .
C e rta v ez, um fu n c io n á rio e sq u e c e u a re s p e ctiv a s en h a , le m b ra n d o a p e n a s que e la c o m e ç a v a c o m X e ter­
m inava co m F.
A p ro b a b ilid a d e de e le te r a c e rta d o a sen h a a o a c a s o , num a ú n ic a te n ta tiv a , é :

a) c)
326 253

b)
1 1
d)
529 420

173
T E S T E S D E V E S T IB U L A R E S

1 ! 4 . ÍU F - C E ) O ito p e ss o a s , sen d o 5 h o m e n s e 3 m u lh e res, s e rã o o rg a n iz a d a s em u m a fila . A p ro b a b ilid a d e de


as p e ss o a s do m esm o s e x o fic a r e m ju n ta s é :

1
a) c) —
28 28

1 d) -L
b)
18 18

115. (M a c k e n z ie - S P ) C o n s id e re to d o s o s n ú m e ro s d e 4 a lg a r is m o s d is tin to s q u e podem s e r fo rm a d o s u tiliz a n -


d o -s e I , 2 , 3 , 4 , 5 e 6 . E s c o lh id o a o a c a s o um d e s se s n ú m e ro s, a p ro b a b ilid a d e de e le c o n te r o a lg a rism o 3
e não c o n te r o a lg a r ism o s 5 é :

a, J - O — e) -L
15 15 12

b, -2 _ d) —
12 15

116. (M a c k e n z ie -S P ) 4 h o m en s e 4 m u lh e res devem o c u p a r o s 8 lu g a re s d e um b a n c o . A p ro b a b ilid a d e de que


n u n ca fiq u em lad o a la d o d u as p e ss o a s d o m e sm o s e x o é :

a) — O e)
56 16

b) 1 d) - L
32

117. ( U F - R S ) C ad a c a rte ia de um a c o le ç ã o é fo rm a d a p or s e is q u ad rad os c o lo rid o s , ju s ta p o s to s c o m o in d ic a a


fig u ra a b a ix o .

E m c a d a c a rte ia , d o is q u ad rad os fo ra m c o lo r id o s de a/ ul, d o is de verde e d o is de ro sa . A c o le ç ã o a p re s e n ta


to d a s as p o s s ib ilid a d e s d e d is tr ib u iç ã o d e s s a s c o r e s n a s c a r t e ia s n as c o n d iç õ e s c ita d a s e n ã o e x is te m
c a rte ia s co m a m esm a d is trib u iç ã o de c o r e s . R e tir a n d o -s e a o a c a s o u m a c a rte ia da c o le ç ã o , a p ro b a b ilid a d e
d e q u e so m en te um a c o lu n a a p re se n te o s q u a d ra d o s d e m esm a c o r é de:

a ) 6% d) 4 8 %
b) 3 6 % e) 90%
c) 40%

118. ( F u v e s t-S P ) Um a rq u iv o d e e s c r itó r io p o ssu i 4 g a v e ta s , ch a m a d a s a, b, c, d. E m c a d a g a v e ta c a b e m no


m á x im o 5 p astas. U m a s e c r e tá r ia gu a rd o u , a o a c a s o , 18 pastas n e sse a rq u iv o . Q u a l c a p ro b a b ilid a d e de
h av e r e x a ta m e n te 4 p a sta s na g a v e ta a?

3 1
a) d)
10 20

b)
1 e)
1
10 30
3
c)
20

174
T E S T E S D E V E S T IB U L A R E S

1 19 . ( U F - R S ) N o jo g o da M e g a S e n a s ã o s o rte a d o s s e is n ú m e ro s d is tin to s d en tre o s q u e a p a re c e m n a fig u ra .

01 02 03 04 05 06 07 08 09 10

11 12 13 14 15 16 18 19 20
-

21 22 23 24 25 26 27 28 29 30

31 32 33 34 35 36 37 38 39 40

41 42 43 44 45 46 47 48 49 50

51 52 53 54 55 56 57 58 59 60

C o n s id e re P a p ro b a b ilid a d e de q u e n en h u m n ú m ero s o rte a d o e m um c o n c u r s o s e ja s o rte a d o n o c o n c u r s o


s e g u in te . D e n tre as a lte rn a tiv a s a b a ix o , a m e lh o r a p r o x im a ç ã o para P é:

a) 9 0 % d) 60%
b) 80% e) 50%
c) 70%

120. ( U F F - R J ) E m u m a b a n d e ja há de/ p a s te is d os q u a is trê s s ã o d e c a rn e , trê s d e q u e ijo e q u a tro de c a m a rã o .


S e F a b ia n a re tirar, a le a to ria m e n te e se m r e p o s iç ã o , d o is p a s té is d e s sa b a n d e ja , a p ro b a b ilid a d e de o s d o is
p a s té is re tira d o s serem de c a m a rã o é :

3
a) d) i-
25 5
4
b) e) —
25 5
2
c)
I5

121. ( U F - P E ) E m um g ru p o de q u a tro d ep u ta d o s d o P p i e q u a tro d o P P 2 , é c o n h e c id o q u e ca d a um d os d e p u ta ­


d o s d o P P I p o ssu i um ú n ic o in im ig o p o lític o d éfitre o i^ d ep u ta d o s d o P P 2 . S e e s c o lh e rm o s n e sse g ru p o,
a le a to ria m e n te , um d ep u tad o do P P I e ou tro do P P 2 . p ara c o m p o r um a c o m is s ã o , q u al a p ro b a b ilid a d e de
n ão o b te rm o s in im ig o s p o lític o s ?

a, ± d)
4 \ 3
\
_l_
b, ± e)
3 4

c) —
2

122. ( U F - R S ) D e n tre um g ru p o fo rm a d o p o r d o is h o m e n s e q u a tro m u lh e res, trê s p e ss o a s s ã o e s c o lh id a s a o


a c a s o . A p ro b a b ilid a d e de q u e s e ja m e s c o lh id o s um h om em e du as m u lh eres é de:

a) 2 5 % d) 50%
b) 30% e) 60%
c) 33%

123. ( U F - P E ) F o rm an d o trê s p ares, a le a to ria m e n te , c o m Jo a q u im . P ed ro, C a r lo s . M a ria . Jo a n a e B e a tr iz , q u al a


p ro b a b ilid a d e de Jo a q u im e C a rlo s fo rm a rem um p a r?

a ) 0.1 d ) 0 .4
b ) 0 .2 e ) 0 .5
c ) 0 .3

175
T E S T E S D E V E S T IB U L A R E S

124. (P u c c a m p -S P ) Em um a e sc o la . 1 0 alu n os (6 ra p a z es e 4 ga ro ta s) apresen tam -se para c o m p o r a diretoria do


G rê m io Estud antil, que deverá ter o s segu in tes m em b ros: I p resid en te. I vice-p resid en te e 2 secretá rio s. O s
n om es d os can d id atos sã o co lo c a d o s em um a u m a , da qual serã o sorteados o s m em bros que co m p o rã o a d ireto ­
ria. A p robabilid ade de q u e, na equ ip e sortead a, o presidente ou o v ice-p resid en te seja m d o s e x o m a scu lin o é :

a, — a> J i
3 15
27
h) — e)
30

•'1
125. ( P U C -S P ) U m re p ó rter preten d e e n tre v is ta r a p e n a s 4 d o s in te g ra n te s de um c o n ju n to m u sic a l c o m p o s to
p or 7 rap azes e 5 g a ro ta s. A p ro b a b ilid a d e de q u e o g ru p o s e le c io n a d o para a e n tre v is ta te n h a p e lo m en os
um re p rese n ta n te de ca d a s e x o é :

76 29
a) d)
99 33

b)
26
e) 9l_
33 99
85
c)
99

126. (U . E . L o n d r in a -P R ) C o n s id e re um c u b o c su a s a re s ta s. A p ro b a b ilid a d e de e s c o lh e rm o s um par de a re sta s


d is tin ta s d e s se c u b o e e la s serem p a ra le la s e n tre si é:

1
a) d, —
33 33

b> - L O A_
66 II
c) —
11
127. (M a c k e n z ie -S P ) D o is p rêm ios igu ais sã o sortead os entre 6 pessoas, send o 4 h om ens e 2 m ulh eres. Supon do que
um a m esm a p e sso a não possa ganhar os 2 prêm ios, a probabilid ade de p elo m enos um hom em ser sorteado é:

a) A d) ü
6 14

b) L e) A
8 9
. 14
c) -----
15

128. ( P U C - R J ) D e sua turm a de 3 0 a lu n o s , é e s c o lh id a um a c o m is s ã o d e 3 re p rese n ta n tes. Q u a l a p ro b a b ilid a d e


de v o c ê fa z e r parte da c o m is s ã o ?

i i
a) d)
10 3
1 _2_
b) e)
12 9
5
c)
24

129. ( F G V - S P ) U m r e c ip ie n te c o n té m 4 b a la s de h o r te lã , 5 d e m o ra n g o e 3 d e a n is . S e d u a s b a la s fo re m
s o rte a d a s s u c e s s iv a m e n te e sem re p o s iç ã o , a p ro b a b ilid a d e d e que s e ja m de m e sm o s a b o r é :

a)
18
d) 2L
65 68

b)
19
e)
22
66 69
_20
c)
67

176
TESTES DE VESTIBULARES

130. ( M a c k e n z ie - S P ) E s c o lh id o s , a o a c a s o , d o is n ú m ero s d is tin to s d o c o n ju n to ( 1 , 2 , 3 , 4 , 5 , 6 . 7 , 8 , 9 . 1 0 } , a


p ro b a b ilid a d e de que o produto d e le s s e ja ím p a r é :

_3_
a) d) i .
4 9
J_
b) e) ±
2 5
2
c)
T

1 3 1 . ( C e f e t -M G ) U m a c a ix a c o n té m 2 5 b o la s ig u a is n u m erad as d e I a 2 5 . R e tir a n d o -s e d u as b o la s , a p r o b a b ili­


d ad e de se o b te r 2 n ú m ero s p rim o s é ig u a l, ou a p ro x im a d a m e n te ig u a l, a:

a) 12% c ) 15% e) 22%


b) 13% d ) 1 8%

1 3 2 . ( U F - S C ) E m um a c a ix a há 2 8 b o m b o n s. to d o s co m fo rm a , m assa e a sp e c to e x te rio r e x a ta m e n te ig u a is. D e s­


s e s b o m b o n s, 7 têm re c h e io de c o c o , 4 d e n o z es e 17 sã o re ch e a d o s com am ên d oas. S e re tira rm o s da c a ix a 3
b o m b o n s sim u ltan e am en te , a p ro b a b ilid a d e de se re tira r um b o m b o m de ca d a s a b o r é . a p ro xim a d a m e n te :

a ) 7 .5 % c ) 1 2 .5 % e ) 1 4 ,5 %
b) 11 % d ) 13%

1 3 3 . ( P U C -S P ) S e r ã o s o rte a d o s 4 p rêm io s ig u a is e n tre os 2 0 m e lh o re s a lu n o s d e um c o lé g io , d en tre o s q u a is


e s tã o T a le s e E u ler. S e ca d a a lu n o pode r e c e b e r a p e n a s um p rêm io , a p ro b a b ilid a d e de q u e T a le s ou E u le r
fa ça m parte d o g ru p o so rte a d o é :

a) -L d> -L
95 19

b) J - e) ü
19 95

o 4
19

134. ( P U C -S P ) C o n s id e re u m a fa m ília n u m e ro sa tal q u e:


• c a d a Filho d o s e x o m a s c u lin o tem um n ú m ero d e irm ã s igu al a o d o b ro do n ú m ero d e irm ã o s;
• c a d a Filho d o s e x o fe m in in o tem um n ú m ero çle irm ã s igu al a o d e irm ã o s a c r e s c id o d e 2 u n id ad es.
A o e s c o lh e r a o a c a s o 2 Filhos d e s sa fa m ília , a p ro b a b ilid a d e de e le s serem de s e x o s o p o s to s é :

11
a) J - d)
13 13
b) 20 11
e)
39 12

O i
12

135. ( U M C - S P ) A ta b e la a b a ix o fo r n e c e , p or s e x o e á re a e s c o lh id a , o n ú m ero de in s c r ito s em um v e stib u la r


para in g r e s s o n o c u rso su p e rio r:

Área
Sexo Biomédicas Exatas Humanas
M a s c u lin o 2 500 1 500 1 500
F e m in in o 1 500 1 000 200 0
E s c o lh id o , a o a c a s o , um d o s in s c r ito s e re p rese n ta n d o p or p{ a p ro b a b ilid a d e do e s c o lh id o s e r d o s e x o
m a s c u lin o e te r op tad o p or E x a ta s e p2 a p ro b a b ilid a d e d o e s c o lh id o s e r d o s e x o fe m in in o , sa b e n d o que
op tou p or B io m é d ic a s , p o d e -se c o n c lu ir q u e:

a ) pi = 0 . 6 e p2 = 0 .3 7 5 d ) p ( = 0 ,1 5 e p2 = 0 ,3 7 5
b ) p| = 0 , 6 e p 2 = 0 ,1 5 e ) pi = 0 ,3 7 5 e p2 = 0 ,1 5
c) pi = 0 ,1 5 e p2 = 0 .1 5

177
T E S T E S D E V E S T IB U L A R E S

136. (U E -R J)
P r o t é tic o s e d e n tis ta s d iz e m q u e a p r o c u ra p o r d e n te s p o s t iç o s n ã o a u m e n to u . A té d e c lin o u um
p o u qu in h o . N o B r a s il, seg u n d o a A s s o c ia ç ã o B r a s ile ir a de O d o n to lo g ia ( A B O ), há 1.4 m ilh ã o d e p e s­
s o a s se m nenhum den te na b o c a , e 8 0 % d e la s j á usam d en tad u ra. A ssu n to e n c e rra d o .

(A d a p ta d o d e: Veja. ou tu b ro d e 1 9 9 7 .)

C o n s id e re q u e a p o p u la çã o b ra s ile ira s e ja d e 1 6 0 m ilh õ e s de h a b ita n tes. E sc o lh e n d o a o a c a s o um d e s se s


h a b ita n te s , a p ro b a b ilid a d e de q u e e le n ão p o ssu a nen h um den te na b o c a e use d e n ta d u ra , de a c o rd o co m a
A B O . é d e:

a ) 0 ,2 8 %
b ) 0 ,5 6 %
c ) 0 ,7 0 %
d ) 0 ,8 0 %

137. ( U n ir io -R J) A O r g a n iz a ç ã o M u n d ial da S a ú d e ( O M S ) p e sq u iso u e c o n c lu iu que um c a s a l s a d io , em que os


d o is n ão s e ja m p a ren te s c o n s a n g ü ín e o s (p a re n te s em p rim eiro g ra u ), a o g e ra r um a c r ia n ç a , pode a p rese n ta r
o se g u in te q u ad ro p r o b a b ilís tic o em re la ç ã o a p ro b le m a s c o n g ê n ito s : s e x o m a s c u lin o tem 2 % de r is c o e
s e x o fe m in in o , 3 % . A p ro b a b ilid a d e de um c a s a l g e ra r um m e n in o c o m d o e n ç a c o n g ê n ita ou um a m en in a
sad ia é , em % . e x p re ss a por:

a ) 0 ,4 8 5 d ) 9 7 ,5
b ) 2 ,5 e) 99
c ) 4 9 ,5

138. ( E S P M - S P ) N a ú ltim a e ta p a de um p ro g ra m a de p rê m io s , o p a rtic ip a n te deve s o rte a r um a b o la de um a


urna que c o n te m I b o la preta e 2 b o la s b ra n c a s (d istin g u ív e is a p en a s p ela c o r ) e ga n h a se tira r a b o la preta.

P o rta n to , a p ro b a b ilid a d e d ele g a n h a r é . P ara " e s q u e n ta r " o p ro g ra m a , o a p re se n ta d o r prop õe q u e se

c o lo q u e u m a b o la v erm elh a na urna. q u e fu n c io n a ria c o m o “c o r in g a " , is to é , se o p a rtic ip a n te tira r a b o la


p reta , e le g a n h a , m a s, s e tira r a b o la v e rm e lh a , e le tem d ir e ito a um a nova retirad a (a g o ra sem a b o la ver­
m e lh a ) e gan h a se tira r a p reta. S e o p a rtic ip a n te a c e ita r e s s a p ro p o sta , sua c h a n c e d e g a n h a r o p rêm io:

a ) a u m e n ta em 20 % . d ) d im in u i d e 20 % .
b ) au m en ta em 4 0 % '. e ) n ã o se a lte ra .
c ) d im in u i d e 10 % .

139. (P U C -R J) A s cartas de um baralh o sã o am on toad as a leatoriam en te. Qual é a p robabilid ade de a ca rta de cim a
ser de cop as e a de b a ix o ta m b ém ? O baralh o é form ado por 5 2 ca rta s de 4 n aipes d iferen tes (1 3 de cad a n aipe).

a)
i d)
i
77 36

i 1
b) e)
25 45

1
c)
27

140 . ( F E I - S P ) N a in s p e ç ã o de q u a lid a d e de p ro d u çã o d e um tip o d e p e ça , a d o ta -se o s e g u in te p ro ce d im e n to : de


ca d a lo te c o m 20 p e ç a s p rod u z id a s sã o sep a ra d a s a le a to ria m e n te 2 p e ç a s ; d e p o is, e s s a s 2 p e ç a s sã o te sta ­
d as e . se p e lo m e n o s um a d e la s a p re s e n ta r alg u m d e fe ito , o lote é re je ita d o . S a b e n d o -s e que num d e te rm i­
n ad o lo te há 6 p e ç a s d e fe itu o s a s e 14 p e ç a s p e rfe ita s , q u al a p ro b a b ilid a d e d e s se lo te s e r a p rov a d o?

a, -L U) A
2 91

b, A e,
10 190
3
c) —
20

178
TESTES DE VESTIBULARES

1 4 1 . (U . E . L o n d r in a -P R ) C o n s id e re c o m o v erd a d eira s a s s e g u in te s in fo rm a ç õ e s:

1 ) O L o n d rin a E sp o rte C lu b e e s tá co m um tim e que g a n h a jo g o s c o m p ro b a b ilid a d e de 0 ,4 0 e m d ia s de


ch u v a e de 0 ,7 0 em d ia s sem ch u va.
2 ) A p ro b a b ilid a d e de um d ia d e ch u v a em L o n d rin a , no m ês de m a rço , é d e 0 .3 0 .

S e o tim e ganhou um jo g o em um dia de m a rço , em L o n d rin a , e n tà o a p ro b a b ilid a d e d e q u e n e ssa cid a d e


ten h a c h o v id o n aq u e le d ia c de:

a) 30% d ) 12 .3 4 8 %
b ) 8 7 .6 5 2 % e ) 8 0 .3 2 8 %
c) 1 9 .6 7 2 %

1 4 2 . (V u n e s p -S P ) Para um a p artid a d e fu te b o l, a p ro b a b ilid a d e de o jo g a d o r K n ào s e r e s c a la d o é 0 .2 e a p ro b a ­


b ilid a d e d e o jo g a d o r S s e r e s c a la d o é 0 .7 . S a b e n d o q u e a e s c a l a ç ã o de um d e le s ê in d e p e n d e n te da
e s c a la ç ã o d o ou tro , a p ro b a b ilid a d e de o s d o is jo g a d o r e s serem e s c a la d o s é:

a) 0 . 0 6 d ) 0 ,5 6
b ) 0 .1 4 e) 0 .7 2
c ) 0 .2 4

143. íU E - R J )

V E R E D A T RO PIC AL NANI

S u p o n h a h aver um a p ro b a b ilid a d e de 2 0 % para um a c a ix a d e M ic ro v la r ser fa ls ific a d a . Em d u a s c a ix a s , a


p ro b a b ilid a d e de p e lo m en os um a d e la s s e r Ç jlsa é ‘

a) 4 %
b) 16%
c ) 20%
d) 36%

144. (U . E . L o n d r in a -P R ) C o n tra c e rta d o e n ç a podem fcer a p lic a d a s a s v a cin a s I ou II. A v a cin a I fa lh a e m 10%
d os c a s o s e a v acin a II e m 2 0 % d os c a s o s , sen d o e s s e s e v e n to s to ta lm e n te in d e p e n d e n tes. N e ssa s c o n d i­
ç õ e s . s e to d o s o s h a b ita n te s d e um a c id a d e re c e b e ra m d o s e s ad eq u ad as d as d u as v a c in a s , a p ro b a b ilid a d e
de um in d iv íd u o não e s ta r im u n i/ ad o c o n tra a d o e n ç a é :

a) 30% d) 2%
b) 10% e) 1%
c ) 3%

145. (U . F. S ã o C a r lo s - S P ) G u s ta v o e sua irm ã C a ro lin e v ia ja ra m de fé r ia s para c id a d e s d is tin ta s . O s p a is re ­


c o m e n d a m q u e am b o s te le fo n e m q u a n d o c h e g a re m a o d e s tin o . A e x p e r iê n c ia e m fé r ia s a n te rio r e s m o stra
q u e nem s em p re G u s ta v o e C a ro lin e cu m p re m e s s e d e s e jo d os p a is. A p ro b a b ilid a d e de G u s ta v o te le fo n a r
é 0 . 6 e a p ro b a b ilid a d e de C a ro lin e te le fo n a r é 0 .8 . A p ro b a b ilid a d e de p e lo m e n o s um d o s filh o s c o n ta ta r
o s p a is é :

a) 0 .2 0 d ) 0 .8 6
b ) 0 .4 8 e ) 0 .9 2
c ) 0 .6 4

179
T E S T E S D E V E S T IB U L A R E S

146. ( U F - P E ) O s tim e s A. B e C p a rtic ip a m de um to rn e io . S u p o n h a q u e a s p ro b a b ilid a d e s de A ganhar e perd er


de B s ã o re s p e ctiv a m e n te 0 ,6 e 0 ,2 . e a s p r o b a b ilid a d e s d e A ganhar e perd er d e C s ã o re s p e ctiv a m e n te 0 ,1
e 0 ,6 . Jo g a n d o co m B e e m seg u id a co m C , q u al a p ro b a b ilid a d e de A em p a ta r o s d o is jo g o s ?

a ) 0 .5 c ) 0 .0 6 e ) 0 .0 3
b ) 0 .0 5 d ) 0 .0 4

147. ( U n ir io - R J ) E m u m a fá b r ic a de p a r a fu so s , a p ro b a b ilid a d e d e um p a ra fu so s e r p e r fe ito c de 9 6 % . S e re ti­


ra rm o s d a p ro d u ç ã o , a le a to ria m e n te , trê s p a r a fu so s , a p ro b a b ilid a d e de to d o s e le s se re m d e fe itu o s o s é
ig u al a:

a) 5~2 c) 5~4 e ) 5 -6
b ) 5 -3 d) 5 " 5

1 4 8 . ( U F - P E ) T r ê s d ad o s p e rfe ito s A, B e C têm su a s fa c e s n u m erad as da seg u in te fo rm a :

D ad o A: du as fa c e s n u m erad as co m 1 e q u a tro co m 5 ;
D ado B: s e is fa c e s n u m e ra d a s c o m 4 ;
D ad o C: q u a tro fa c e s n u m era d a s co m 2 e d u as c o m 6 .

L a n ç a n d o -s e d o is d e s se s d a d o s, d irem o s q u e é g a n h a d o r a q u e le que ap resen ta o m a io r n ú m ero na fa c e vol


lad a para c im a . D e p o sse d e s ta s in fo rm a ç õ e s, a n a lis e a s a firm a tiv a s a seg u ir:

1) O d ad o A ganha d o dado B co m p ro b a b ilid a d e - y .

2
2 ) O d ad o B g an h a d o d ad o C c o m p ro b a b ilid a d e — ■

3 ) O d ad o C gan h a do dado A co m p ro b a b ilid a d e y .

E s tá (ã o ) c o rr e ta (s ):

a) 1 e 2 , ap e n a s. b) I , a p e n a s. c) I, 2 e 3. d) I e 3 , a p e n a s. e ) 2 e 3 , a p e n a s.

1 4 9 . (E n e m -M E C ) U m a em presa de a lim en to s im prim iu em suas em b alagen s um ca rtã o de ap ostas do segu in te tipo:

Frente do cartão Verso do cartão


Como jogar:
1
- Inicie raspando apenas uma das
alternativas da linha de início (linha 1).
2 - Se achar uma bola de futebol, vá
para a linha 2 e raspe apenas uma
das alternativas. Continue raspando
3 dessa forma até o fim do jogo.
- Se encontrar um "X " em qualquer
uma das linhas, o jogo está
4 encerrado e você nâo terá direito ao
prêmio.
- Se você encontrar uma bola de
5 futebol em cada uma das linhas terá
direito ao prêmio.

C a d a c a rtã o de a p o sta s p ossu i 7 fig u ra s de b o la s de fu te b o l e 8 s in a is d e “ X ” d is trib u íd o s en tre os I5 e sp a ­


ç o s p o s s ív e is , de tal fo rm a q u e a p ro b a b ilid a d e d e um c lie n te g a n h a r o p rêm io n u n ca s e ja igu al a z e ro . Em
d e term in a d o c a rtã o e x is te m d u as b o la s na lin h a 4 e du as b o la s na lin h a 5 . C o m e s s e c a rtã o , a p ro b a b ilid a ­
de d e o c lie n te g a n h a r o p rêm io é :

a) — b) — c) — d) — e) ——
27 36 54 72 108

180
TESTES DE VESTIBULARES

O texto abaixo refere-se às questões 150 e 151:

U m a p o sta d o r tem trê s o p ç õ e s p ara p a rtic ip a r de c e rta m o d a lid a d e de jo g o , q u e c o n s is te n o s o rte io a le a tó ­


rio de um n ú m ero d en tre dez.

1 ? o p ç ã o : c o m p ra r trê s n ú m ero s para um ú n ic o s o rte io .


2 ? o p ç ã o : c o m p ra r d o is núm eros para um s o rte io e um n ú m ero para um seg u n d o s o rte io .
3 ? o p ç ã o : c o m p ra r um n ú m ero para ca d a s o rte io , num total de trê s s o rte io s.

1 5 0 . ( E n e m -M E C ) S e X, Y, Z re p rese n ta m a s p ro b a b ilid a d e s de o a p o sta d o r ganhar algum prêm io , e s c o lh e n d o ,


re s p e ctiv a m e n te , a U , a 2? ou a 3 " o p ç õ e s , é c o r r e to a firm a r q ue:

a) X < Y <Z d) X = Y > Z


b) X = Y =Z e) X > Y > Z
c) X > Y = Z

1 5 1 . ( E n e m -M E C ) E s c o lh e n d o -s e a 2 “ o p ç ã o , a p ro b a b ilid a d e d e o a p o sta d o r não ganhar em q u a lq u e r d o s so r­


te io s é igu al a:

a) 90 % c) 72% e) 65%
b) 8 1 % d) 7 0 %

1 5 2 . (U . F. S ã o C a r lo s - S P ) U m jo g o para du as p e ss o a s c o n s is te em um a urna co m 2 b o la s v e rm e lh a s e 1 azu l.


G an h a o jo g o q u em re tira r da urna a b o la a z u l. C a s o um jo g a d o r re tire um a b o la v e rm e lh a , e s s a v o lta para
a urna. e o o u tro jo g a d o r fa z sua re tira d a . O s jo g a d o r e s v ã o a lte rn a n d o su a s re tira d a s a té q u e s a ia a bola
azu l. T o d as as b o la s têm a m esm a p ro b a b ilid a d e de serem re tira d as. A p ro b a b ilid a d e d o p rim eiro a jo g a r
g a n h a r o jo g o , is to é , em um a d e su as re tira d a s p e g a r a b o la a z u l. v a le:

a, 1 c) —
3 2

b, —
5
d, —
5

1 5 3 . ( U F - P E ) U m c a s a l p la n e ja ter 4 filh o s. S u p o n d o igu al a c h a n c e de um filh o n a s c e r do s e x o m a s c u lin o ou


do se x o fe m in in o , q u al a p ro b a b ilid a d e d e o c a s a l v ir a ter. n o m ín im o , d o is filh o s d o s e x o m a s c u lin o ?

a ) 0 ,6 8 7 1 c ) 0 .6 8 7 3 e ) 0 ,6 8 7 5
b ) 0 .6 8 7 2 d) 0.6874^
f
1 5 4 . ( F E I - S P ) S a b e n d o -s e q u e no p r o c e s so d e m o n ta g e m de um d e term in a d o tip o de m á q u in a a p ro b a b ilid a d e
de o c o rr ê n c ia de algu m e rr o é 0 ,0 2 , qual a p ro b a b ilid a d e p de q u e a o m o n ta r 4 d e s sa s m á q u in a s o co rra m
e rro s em e x a ta m e n te 2 d as m o n ta g e n s?

a ) p = 0 ,0 4 c) p = 0 .0 2 V 0 ,9 8 2 e) p = 2 4 •0 ,0 2 2 • 0 .9 8 2
b ) p = 0 ,0 0 0 4 d ) p = 6 • 0 , 0 2 2 ■0 , 9 8 2

1 5 5 . ( U F - R S ) P ara c a d a um a d as 3 0 q u e stõ e s de um a prova o b je tiv a s ã o a p rese n ta d a s 5 a lte rn a tiv a s de re s p o s ­


ta s. d as q u a is s o m en te um a é c o rre ta .
C o n s id e re as a firm a ç õ e s re la tiv a s à p rova:

I) E x is te m no m á x im o 1 5 0 m a n e ira s d ife re n te s de re sp o n d e r à prova.


II) R e sp o n d e n d o a le a to ria m e n te , a p ro b a b ilid a d e de e rra r to d a s a s q u e stõ e s é ( 0 .8 ) 30.
III) R esp on d en d o ale a to ria m e n te, a p robab ilid ad e de exa ta m e n te 8 q u e stõ e s e sta rem c o rr e ta s é

^30\o.2) 8(O,8)22.
I8
A n a lisa n d o a s a firm a ç õ e s , c o n c lu ím o s q u e:

a ) ap en as III é v erd ad eira . d) a p e n a s 11 e III s ã o v e rd a d eira s.


b ) ap en as I e II sã o v e rd a d eira s. e) I. II e III sã o v erd ad eiras.
c ) ap en as I e III s ã o v e rd a d eira s.

181
Respostas
dos testes
1. d 32. b 63. d 94. d 125. e
2. c 33. d 64. d 95. a ' 126. e
3. d 34. c 65. a 96. d 127. c
4. h 35. e 66. e 97. e 128. a
5. a 36. c 67. c 98. c 129. b
6. b 37. c 68. b 99. d 130. d
7. d 38. a 69. d 100. b 131. a
8. d 39. c 70. c 101. c 132. e
9. b 40. b 71. e 102. b 133. d
10. e 41. e 72. e 103. c 134. b
11. c 42. e 73. e 104. b 135. d
12. c 43. b 74. c 105. c 136. c
13. b 44. b 75. b 106. b 137. c
14. d 45. e 76. d 107. e 138. e
15. e 46. b 77. e 108. c 139. a
16. a 47. c 78. b 109. d 140. c
17. d 48. e 79. a 110. c 141. c
18. c 49. d 80. e 111. c 142. d
19. b 50. e 81. e 112. b 143. d
20. b 51. b 82. a 113. d 144. d
21. d 52. d 83. b 114. a 145. e
22. b 53. d 84. c 115. d 146. c
23. b 54. d 85. e 116. e 147. e
24. a 55. b 86. c 117. c 148. c
25. d 56. e 87. b 118. a 149. c
26. d 57. e 88. d 119. e 150. e
27. c 58. b 89. e 120. c 151. c
28. e 59. a 90. a 121. a 152. d
29. d 60. a 91. e 122. e 153. e
30. c 61. d 92. c 123. b 154. d
31. c 62. e 93. e 124. d 155. d

182
Significado das siglas de vestibulares
A c a f e -S C — A s s o c ia ç ã o C a ta rin e n s e d a s F u n d a ç õ e s E d u c a c io n a is , S a n ta C a ta rin a
A F A -S P — A ca d e m ia da F o rç a A é re a , S ã o P a u lo
A m a n -R J — A c a d e m ia M ilita r de A g u lh a s N e g ra s. R io d e Ja n e ir o
C e fe t-M G — C e n tro F e d e ra l de E d u c a ç ã o T e c n o ló g ic a de M in a s G e ra is
C e fe t - P R — C e n tro F e d e ra l de E d u c a ç ã o T e c n o ló g ic a d o P aran á
C e s e s p -P E — C e n tro de E stu d o s S u p e rio r e s d o E sta d o de P e rn a m b u co
C e s g r a n r io -R J — C e n tro de S e le ç ã o d e C a n d id a to s a o E n s in o S u p e rio r d o G ran d e R io , R io d e Ja n e ir o
C e s u b r a -D F — C e n tro de E n s in o S u p e rio r U n ific a d o d e B r a s ília . D is trito F ed era l
C e su p a -P A — C e n tro U n iv e rs itá rio d o Pará
C o v e s t-P E — C o m is s ã o d e V e s tib u la re s de P e rn a m b u co
E C M - A L — E s c o la d e C iê n c ia s da S a ú d e de A la g o a s
E E M - S P — E s c o la de E n g e n h a ria M a u á , S ã o P au lo
E f e i-M G — E s c o la F e d e ra l de E n g e n h a ria d e Ita ju b á , M in a s G e ra is
E fo a -M G — E s c o la d e F a r m á c ia e O d o n to lo g ia d e A lfe n a s , M in a s G e ra is
E s a l-M G — E s c o la S u p e rio r d e A g ric u ltu ra de L a v ra s, M in a s G e ra is
E s P C E x - S P — E s c o la P re p a ra tó ria d e C a d e te s d o E x é r c ito , S ã o P au lo
E T F - R J — E s c o la T é c n ic a F e d e ra l do R io d e Ja n e ir o
F a a p -S P — F u n d a ç ã o A rm an d o Á lv a re s P e n te a d o , S ã o P au lo
F C A -P A — F a cu ld a d e de C iê n c ia s da A d m in is tra ç ã o , Pará
F C M -M G — F a cu ld a d e de C iê n c ia s M é d ic a s d e M in a s G e ra is
F C M S C - S P — F a cu ld a d e de C iê n c ia s M é d ic a s da S a n ta C a s a de S ã o P au lo
F E I - S P — F a cu ld a d e de E n g e n h a ria In d u stria l. S ã o P au lo
F e s p -S P — F a cu ld a d e de E n g e n h a ria de S ã o P au lo
F G V -S P — F u n d a ç ã o G e tú lio V argas. S ã o Pau lo
F IS F S -S P — F a c u ld a d e s In te g ra d a s d e S a n ta F é do S u l, S ã o Pau lo
F u n e c -M G — F u n d a çã o E d u c a c io n a l de C a ra tin g a , M in a s G e ra is
F u n re i-M G — F u n d a çã o d e E n s in o S u p e rio r de S ã o J o ã o dei R e i, M in a s G e ra is
F u rg -R S — F u n d a çã o U n iv ersid a d e d o R io G ra n d e. R io G ra n d e do S u l
F U R -R N — F u n d a çã o U n iv ersid a d e R e g io n a l d o R io G ra n d e d o N orte
F u v e s t-S P — F u n d a çã o para o V e stib u la r da U n iv ersid a d e d e S ã o P au lo
I B M E C - S P — In stitu to B r a s ile ir o d e M e rc a d o de C s p ita i$ ,^ 5 ã o P au lo
1 M E -R J — In stitu to M ilita r de E n g e n h a ria . R io de Ja n e ir o
I M E S - S P — C e n tro U n iv e rs itá rio M u n ic ip a l de S ã o C a e ta n o do S u l, S ã o P au lo
IT A -S P — In stitu to T e c n o ló g ic o d e A e ro n á u tic a . S ã o P au lo
P U C -B A — P o n tifíc ia U n iv ersid a d e C a tó lic a da B a h ia
P u c c a m p -S P — P o n tifíc ia U n iv ersid a d e C a tó lic a d e C a m p in a s , S ã o P au lo
P U C -M G — P o n tifíc ia U n iv ersid a d e C a tó lic a de M in a s G e ra is
P U C -P R — P o n tifíc ia U n iv ersid a d e C a tó lic a do P aran á
P U C - R J — P o n tifíc ia U n iv ersid a d e C a tó lic a do R io de Ja n e ir o
P U C -R S — P o n tifíc ia U n iv ersid a d e C a tó lic a do R io G ra n d e d o S u l
P U C - S P — P o n tifíc ia U n iv ersid a d e C a tó lic a de S ã o P au lo
U. C . B r a s ília - D F — U n iv ersid ad e C a tó lic a de B r a s ília . D is trito F ed era l
U C D B - M S — U n iv ersid ad e C a tó lic a D om B o s c o . M a to G ro s s o d o S u l
U C -G O — U n iv ersid ad e C a tó lic a d e G o iá s
U C -M G — U n iv ersid ad e C a tó lic a d e M in a s G e ra is
U c s a l- B A — U n iv ersid ad e C a tó lic a d e S a lv a d o r. B a h ia
U C S - R S — U n iv ersid ad e de C a x ia s do S u l. R io G ra n d e d o S u l
U d e s c -S C — U n iv ersid ad e do E s ta d o de S a n ta C a ta rin a
U E -C E — U n iv ersid ad e E stad u al d o C e a rá
U E F S - B A — U n iv ersid ad e E stad u a l de F e ira de S a n ta n a . B a h ia
U E -M A — U n iv ersid ad e E stad u al d o M a ra n h ã o
U E -M G — U n iv ersid ad e E sta d u a l de M in a s G e ra is
SIGLAS DE VESTIBULARES

U E -P B — U n iv ersid a d e E stad u al da P a ra íb a
U E -P I — U n iv ersid a d e E sta d u a l d o P ia u í
U E -R J — U n iv ersid a d e E stad u al d o R io de Ja n e ir o
U e s b -B A — U n iv ersid a d e E sta d u a l d o S u d o e s te B a ia n o . B a h ia
U F -A C — U n iv ersid a d e F ed era l do A cre
U F -A L — U n iv ersid a d e F e d e ra l de A la g o a s
U F -A M — U n iv ersid a d e F e d e ra l d o A m a z o n a s
U F -B A — U n iv ersid a d e F ed era l da B a h ia
U F -C E — U n iv ersid a d e F e d e ra l d o C e a rá
U F -E S — U n iv ersid a d e F e d e ra l d o E s p írito S a n to
U F F - R J — U n iv ersid a d e F ed era l F lu m in e n s e . R io d e Ja n e ir o
U F -G O — U n iv ersid a d e F ed era l d e G o iá s
U F -M A — U n iv ersid a d e Fed eral d o M a ra n h ã o
U F -M G — U n iv ersid a d e F e d e ra l d e M in a s G e ra is
U F -M T — U n iv ersid a d e F e d e ra l do M a to G ro s s o
U F -P A — U n iv ersid a d e F ed era l d o Pará
U F -P B — U n iv ersid a d e F ed era l da P a ra íb a
U F -P E — U n iv ersid a d e F ed era l de P e rn a m b u co
U F -P I — U n iv ersid a d e F e d e ra l d o P ia u í
U F -P R — U n iv ersid a d e F e d e ra l do P aran á
U F -R J — U n iv ersid a d e F e d e ra l d o R io de Ja n e ir o
U F -R N — U n iv ersid a d e F ed era l do R io G ra n d e d o N orte
' R - R J — U n iv ersid a d e F e d e ra l R u ra l d o R io de Ja n e ir o
U F - R S — U n iv ersid a d e F e d e ra l do R io G ra n d e d o Sul
U F -S C — U n iv ersid a d e F ed era l de S a n ta C a ta rin a
U F - S E — U n iv ersid a d e F ed era l de S erg ip e
U lb r a -D F — U n iv ersid a d e L u tera n a do B r a s il. D is trito F ed era l
U lb r a -R S — U n iv ersid a d e L u te ra n a d o B r a s il. R io G ra n d e do S u l
U M C -S P — U n iv ersid a d e de M ogi das C ru z e s . S ã o P au lo
U m e s p -S P — U n iv ersid a d e M e to d ista d e S ã o P au lo
U n am a-P A — U n iv ersid a d e da A m a z ô n ia . P ará
U n B -D F — U n iv ersid a d e de B r a s ília . D is trito F ed era l
U n e b -B A — U n iv ersid a d e do E sta d o da B a h ia
U n esp a r — U n iv ersid a d e E stad u al do P aran á
U n ic a m p -S P — U n iv ersid a d e E stad u al de C a m p in a s . S ã o P au lo
U n ic a p -P E — U n iv ersid a d e C a tó lic a de P e rn a m b u co
U n ife n a s -M G — U n iv ersid a d e de A lfe n a s , M in a s G e ra is
U n ife s p -S P — U n iv ersid a d e F e d e ra l de S ã o P a u lo
U n ifo r -C E — U n iv ersid a d e de F o rta le z a . C e a rá
U n im e p -S P — U n iv ersid a d e M e to d ista de P ir a c ic a b a . S ã o P au lo
U n im e s -S P — U n iv ersid a d e M e tro p o lita n a de S a n to s . S ã o P au lo
U n io e s te -P R — U n iv ersid a d e E stad u al do O e s te d o Paraná
U n ip -S P — U n iv ersid a d e P a u lista O b je tiv o . S ã o P au lo
U n ir io - R J — U n iv ersid a d e d o R io de Ja n e ir o
U n irp -S P — C e n tro U n iv ersitá rio d o R io P re to . S ã o P au lo
U n is a -S P — U n iv ersid a d e de S a n to A m a ro . S ã o P au lo
U n is in o s -R S — U n iv ersid a d e do V ale do R io d o s S in o s . R io G ra n d e d o S u l
U n ita u -S P — U n iv ersid a d e de T a u b a té . S ã o P au lo
U n iu b e -M G — U n iv ersid a d e de U b e ra b a . M in a s G e ra is
U ni v a le -M G — U n iv ersid a d e d o V ale d o R io D o c e , M in a s G e ra is
U P E - P E — U n iv ersid a d e do E s ta d o de P e rn a m b u co
U S F - S P — U n iv ersid a d e S ã o F r a n c is c o . S ã o P au lo
U S J T - S P — U n iv ersid a d e S ã o Ju d a s T a d eu . S ã o P au lo
U T P -P R — U n iv ersid a d e Tu iu ti do P aran á
V u n e s p -S P — F u n d a çã o para o V e stib u la r da U n iv ersid a d e E sta d u a l P a u lista

184
Fundamentos de Matemática Elementar é uma coleção
consagrada ao longo dos anos por oferecer ao estudante o
mais completo conteúdo de Matemática elementar. Os vo­
lumes estão organizados da seguinte forma:

•volume 1 — conjuntos, funções


• volume 2 — logaritmos
• volume 3 — trigonometria
• volume 4 — seqüências, matrizes, determinantes,
sistem as
• volum e 5 — c o m b in a tó ria , p ro b ab ilid ad e
• volum e 6 — com p lexo s, p olinôm ios, eq u ações
• volum e 7 — g eo m etria a n a lítica
• volum e 8 — lim ites, d eriv ad as, noções de in teg ral
• volum e 9 — g eo m etria plana
• volum e 10 — g eo m etria esp acial
• volum e 11 — m a tem á tica co m e rcia l, m a tem á tica
fin a n ce ira , esta tística d escritiv a

A coleção atende a alunos do ensino médio que procu­


ram uma formação mais aprofundada, estudantes em fase
pré-vestibular e também universitários que necessitam
rever a Matemática elementar.
Os volumes contêm teoria e exercícios de aplicação, além
de uma seção de testes de vestibulares, acompanhados de
respostas. Há ainda uma série de artigos sobre história da
Matemática relacionados aos temas abordados.
Na presente edição, a seção de testes de vestibulares foi
atualizada, apresentando novos testes selecionados a partir
dos melhores vestibulares do país.

Você também pode gostar